Sei sulla pagina 1di 87

TRUSTS & ESTATES

1. Introduction: Freedom of Disposition


A. The Power to Transmit Property at Death
 Question for the Course: Who takes a decedent’s property at his or her death?
1. Freedom of A. The Power to Transfer Property at Death
Disposition & the  Freedom of Disposition/Dead Hand Control: The organizing principle of the American law of donative
Dead Hand transfers is freedom of disposition. Property owners have nearly unrestricted rights to dispose of their
property at their death as they please. RS3d Property 10.1.
 “(c) American law does not grant courts any general authority to question the wisdom, fairness, or
reasonableness of the donor’s decisions . . . . The main function of the law in this field is to facilitate rather
than to regulate. . . . [but] American law curtails freedom of disposition…to the extent that the donor attempts
to make a disposition or achieve a purpose that is prohibited or restricted by an overriding rule of law.” RS3d
Prop. 10.1. {McL}
1. Arguments in support—testamentary freedom: decedent’s property; beneficiary has no right to property; given
that a decedent can completely disinherit a beneficiary, the decedent should be able to condition or restrict an intended
beneficiary’s inheritance.
2. Arguments against: deceased donor cannot change her mind or respond to changed circumstances
B. Valid Restrictions
 Americans are largely free to impose whatever conditions they want [on a gift to a beneficiary], and their
plans can often be imposed for as long as they want. Ray Madoff.
C. Invalid Restrictions: Freedom of disposition is subject only to wealth transfer taxation and a handful of policy
limitations—i.e., spousal rights, creditors’ rights, unreasonable restraints on alienation or marriage, provisions
promoting divorce, impermissible racial restrictions, provisions encouraging illegal activity, RAP.
 Absolute restraints on marriage: A condition that the beneficiary not marry anyone, is general or total, and,
at least in the case of a first marriage, is against public policy and void. Shapira.
 Partial Restraint on Marriage: A partial restraint on marriage which imposes only reasonable restrictions
on the right to marry is valid, and not contrary to public policy. Shapira. *also does not violate constitutional
rights of beneficiary*
o Reasonableness: A restraint unreasonably limits the transferee’s opportunity to marry if a marriage
by the restraint is not likely to occur. The likelihood of a marriage is a factual question, to be
answered from the circumstances of a particular case. Shapira.
 Maddox (finding an unreasonable restraint on marriage when there were only 5 or 6
eligible men in the community in the “horse and buggy” days)
 Shapira (reasonable no when judicial knowledge of insffucient number of eligible young
ladies and son could travel by plane and freeway, and communicate by phone.)
o Requirement to Marry within a Religion: Gifts conditioned upon the beneficiary’s marrying
within a particular religious class or faith are generally reasonable. Shapira.
o Temporal Requirement: There must be a reasonable grace period. Shapira (finding a seven year
time limit for a 21-year old to be reasonable)
 Religion Requirement: Testamentary gifts conditioned upon the religious faith of the beneficiary are invalid.
Shapira (citing Drace).
 Encouraging Separation/Divorce: A bequest or devise conditioned on the beneficiary’s obtaining a
separation or divorce from a spouse is invalid. Shapira (citing Fineman)
o A provision that encourages separation or divorce is invalid, but a provision that is meant to provide
support in the event of separation or divorce is valid.
 Disrupting Familial Relationships: Conditions that disrupt or discourage familial relationships are invalid.
 Destruction of Property: see pg. 14 (“a well-ordered society cannot tolerate” waste)
 Void for Uncertainty: i.e., gift conditioned on marry “social equal” or not marrying a “common woman,”
“woman of the same class,” “prudent woman” – ALL void for vagueness
Shapira v. Union National Bank: A father’s testamentary gifts to his son required him to be married within seven years of the father’s death to
“a Jewish girl whose both parents were Jewish.” Otherwise, the estate would “gift over” to the State of Israel. The court held that judicial
enforcement of the condition would not violate the son’s 14th Amendment right to marry because the court was being asked to enforce the
testator’s restriction upon his son’s inheritance, not to enforce any restriction upon the son’s right to marry. The court also held that the condition
did not violate public policy as a restraint on marriage because the condition was a partial, rather than a total, restraint on marriage (a ban on a
type of marriage, not on marrying). The court was also motivated by the depth of T’s conviction to the Jewish faith.
 Provision should not have been made: risk of embittering son to Jewish faith; detrimental effects on siblings’ relationship; not being
able to revise for changed circumstances; costly litigation
 Solution: Trust; Trustee has discretion
B. The Mechanics of Succession
1. Probate & A. Macro Issue
Nonprobate Property  Who gets the decedent’s property upon death? Depends on whether property is NONPROBATE OR
1/87
PROBATE
B. Probate vs. Nonprobate Property
 Will disposes of probate property, but not nonprobate property.
C. Probate Property
 Probate property is the default
 If decedent has will, probate property will pass through it
 If decedent does NOT have will, probate property will pass via intestacy
D. Nonprobate Property: Cannot be passed by will or through intestacy
 Instrument other than a will (“will substitute”)
o Joint tenancy property (joint tenants with rights of survivorship / Tenants by the Entirety)
 The dead tenant’s rights terminate at death and the surviving tenant has sole ownership.
 Not devisable
 Can be used for real estate, bank accounts, etc.
o Life Insurance
 passed only by the provisions of the policy—to the beneficiary indicated on the form;
proven by death certificate; death benefit cannot be given to anyone other than the
beneficiary.
o Property held under Pay-on-Death (POD) & Transfer-on-Death (TOD) Contracts
 banks accounts, mutual funds
 Property goes to the designated surviving beneficiary
o Property held in Inter Vivos Trust (UTATA trusts and testamentary trusts go through probate)
 Assets are put into a revocable trust during life and are passed on according to the terms
of the trust.

3. Probate A. Three Functions


Administration  (1) Provides evidence of transfer of title; (2) Protects D’s creditors; (3) Ensures D’s assets pass to intended
beneficiaries {McL}
B. Administration of Probate
 Primary (or domiciliary) jurisdiction: the probate court in the county where the decedent was domiciled at
time of death has jurisdiction over all the decedent’s personal property, and over the decedent’s real property
located within that jurisdiction.
 Ancillary Jurisdiction: Decedent owned real property in another jurisdiction.
 Appointment of “Personal Representative”—fiduciary
o Duties: Collect, inventory, and manage decedent’s assets; give notice and pay creditors (including
taxes), distribute assets
o PR may purchase a bond
 usually waived in family situations
 required in court-appointed administration
o If person dies “testate” and PR named in will, the PR is the executor
 “devises” real property to “devisees”
 “bequeaths” personal property to “legatees”
 “Letters Testamentary”: document from probate court authorizing an individual as an
executor
o If person dies “intestate” or PR not named in will, the PR is the administrator
 Real property “descends” to “heirs”
 Personal property “distributed” to “next-of-kin”
 “Letters of Administration”: document from probate court authorizing an individual as the
administrator
 All interested parties get notice—all potential intestate heirs, all beneficiaries, creditors
 Collection and inventory of decedent’s assets

2/87
 PR must pay outstanding debts (including taxes)
o Federal estate tax return (2018 Federal Estate Tax Exclusion Amount—$11.2 million)
 9 months to file
 only filed if it exceeds estate tax exclusion amount
 Exclusion can be transferred to surviving spouse, but the federal estate tax return must be
filed
o State Estate Tax Return
o Decedent’s Final Federal Income Tax Return
o Decedent’s Final State Income Tax Return
o (Fiduciary) Annual Federal Income Tax Return for Estate
o (Fiduciary) Annual State Income Tax Return for Estate

4. Is Probate A. Avoiding Probate


Necessary?  Probate can be time consuming and costly
 Most estates can avoid probate because they are small
o Small Estate Administration Statute
 affidavit of survivorship
o Informal Family Settlement
 Use a combination of a pour-over will and a revocable trust to avoid probate
B. Administering a Small Estate: Short-term nonclaim statutes—when you probate a will, creditors get notice and
have a short time to assert a claim (probate protects against unknown creditors)
o Car—transfer by affidavit (and possibly by death certificate) at DMV
o small accounts—transfer by affidavit (and possibly by death certificate) at bank
o joint checking account—surviving joint tenants takes all (nonprobate asset)
o life insurance policy—nonprobate asset
o employer pensions plan with designated beneficiary—nonprobate asset
C. Typical Estate Plan for Decedent with More than a Modest Estate

2. Intestacy: An Estate Plan by Default


A. An Estate Plan by Default
1. Advantages of a Will 1. Advantages of a Will vs. Intestacy {McL}

3/87
versus Intestacy  Name Executor and Alternates:
o PR gathers asset, pay debts and taxes, and distributes remaining assets to beneficiary.
o If decedent dies “intestate” the court appoints a PR for her.
o Court will give preference to family members (SS/C/Ps/siblings), but if none are available/able to
qualify, the court could appoint a creditor
o Much better to have will and name Executor with alternates.
 Name Guardian of Minor Children and Alternates:
o If person dies “intestate with minor children and other parent not alive” the court appoint.
o Court will give preference to family member, but decedent may have strong feelings about which
brother/sister, etc. is best suited to raise her children.
o For many, this is the most important reason to have a will
 Waive Bond of Executor (if bond required under state law)
o Bond: Insurance Policy that protects beneficiaries if PR absconds with assets
o Estate pays for this—it’s best avoid the expense
o Some states, including Utah require bond only in certain circumstances
 Control Disposition of Assets
o Distribution of probate assets of D who dies intestate governed by statute of descent and
distribution (aka law of intestate succession or intestacy)
 Anticipate changed circumstances
 Avoid Intestate Succession
2. The Purpose of 2. The Purpose of Intestacy Statutes: Distribution of Probate Assets of D who dies Intestate governed by statute of
Intestacy Statutes descent and distribution (aka law of intestate succession or intestacy statute) {McL}
 Primary Objective: To carry out probable intent of typical intestate decedent.
 Secondary Policy Considerations: Protect economic health of D’s family; textual simplicity; ease of
administration
3. Applicable Law & A. Applicable Law
the Uniform Probate  The law of the state where a decedent was domiciled at death governs the disposition of the decedent’s
Code personal property
 The law of the state where the decedent’s real property is located governs the disposition of real property
B. UPC Text
 UPC 2-101. Intestate Estate
 (a) Any part of a decedent’s estate not effectively disposed of by will passes by intestate succession to the
decedent’s heirs as prescribed in this [code], except as modified by the decedent’s will.
 (b) A decedent by will may expressly exclude or limit the right of an individual or class to succeed to property
of the decedent passing by intestate succession. If that individual or a member of that class survives the
decedent, the share of the decedent’s intestate estate to which that individual or class would have succeeded
passes as if that individual or each member of that class had disclaimed his [or her] intestate share.
 UPC 2-102. Share of Spouse
 The intestate share of a decedent’s surviving spouse is:
o (1) the entire intestate estate if:
 (A) no descendant or parent of the decedent survives the decedent; or
 (B) all of the decedent’s surviving descendants are also descendants of the surviving
spouse and there is no other descendant of the surviving spouse who survives the
decedent;
o (2) the first [$300,000], plus three-fourths of any balance of the intestate estate, if no descendant of
the decedent survives the decedent, but a parent of the decedent survives the decedent;
o (3) the first [$225,000], plus one-half of any balance of the intestate estate, if all of the decedent’s
surviving descendants are also descendants of the surviving spouse and the surviving spouse has
one or more surviving descendants who are not descendants of the decedent;
o (4) the first [$150,000], plus one-half of any balance of the intestate estate, if one or more of the
decedent’s surviving descendants are not descendants of the surviving spouse.
 UPC 2-103. Share of Heirs Other Than Surviving Spouse
 (a) Any part of the intestate estate not passing to a decedent’s surviving spouse under Section 2-102, or the
entire intestate estate if there is no surviving spouse, passes in the following order to the individuals who
survive the decedent:
o (1) to the decedent’s descendants by representation;
o (2) if there is no surviving descendant, to the decedent’s parents equally if both survive, or to the
surviving parent if only one survives;
o (3) if there is no surviving descendant or parent, to the descendants of the decedent’s parents or
either of them by representation;
o (4) if there is no surviving descendant, parent, or descendant of a parent, but the decedent is
survived on both the paternal and maternal sides by one or more grandparents or descendants of
grandparents:
 (A) half to the decedent’s paternal grandparents equally if both survive, to the surviving

4/87
paternal grandparent if only one survives, or to the descendants of the decedent’s paternal
grandparents or either of them if both are deceased, the descendants taking by
representation; and
 (B) half to the decedent’s maternal grandparents equally if both survive, to the surviving
maternal grandparent if only one survives, or to the descendants of the decedent’s
maternal grandparents or either of them if both are deceased, the descendants taking by
representation;
o (5) if there is no surviving descendant, parent, or descendant of a parent, but the decedent is
survived by one or more grandparents or descendants of grandparents on the paternal but not the
maternal side, or on the maternal but not the paternal side, to the decedent’s relatives on the side
with one or more surviving members in the manner described in paragraph (4).
 (b) If there is no taker under subsection (a), but the decedent has:
o (1) one deceased spouse who has one or more descendants who survive the decedent, the estate or
part thereof passes to that spouse’s descendants by representation; or
o (2) more than one deceased spouse who has one or more descendants who survive the decedent, an
equal share of the estate or part thereof passes to each set of descendants by representation.
 UPC 2-105. No Taker
 If there is no taker under the provisions of this [article], the intestate estate passes to the state.
C. UPC Comments
 The UPC favors the surviving spouse
 The decedent’s property escheats to the state much sooner
 Problems: The cash numbers are arbitrary; if children are minors, there will have to be guardianship
proceedings—the surviving spouse will likely become the guardian; there could be resentment in case of
remarriage and among surviving children. {McL}
o Better Solution: Statutory Trust: Deceased spouse’s assets go into a statutory trust; trust income to
SS; trust principal to SS and DS’s descendants for support, health, education; At SS’s death, assets
to DS’s descendants, into further trust or custodial account for minors. {McL}
UPC Approach Who takes? How Much?
(Chart 1) 1. Surviving spouse 100% if no issue or parents; or
100% if all decedent’s issue are also issue of
surviving spouse & surviving spouse has no
other issue; or
$300K + 75% of rest if no issue but
surviving parent; or
$225K + 50% of rest if all issue are also
issue of surviving spouse & surviving
spouse has other issue
$150K + 50% of rest if one or more issue
not of surviving spouse
Any property not passing to surviving spouse passes as follows:
2. Issue Equally (by representation)
3. Parents Equally, or all to the survivor
4. Issue of parents Equally (by representation)
5. Grandparents/issue 50% to paternal grandparents or survivor;
50% to maternal grandparents or survivor;
otherwise to their issue equally (by
representation);
If no surviving grandparents or issue on one
side, all to the other side
6. Issue of predeceased spouse(s) Equally (by representation)
7. Escheat to state 100%
UPC Approach Surviving spouses’ share depends on family situation: Surviving spouse’s share
(Chart 2) Decedent has no surviving issue or parents 100%
All Decedent’s issue are issue of the surviving spouse, and surviving 100%
spouse has no other issue
Decedent has no other issue but parent $300K + 75% of rest
All Decedent’s issue are issue of surviving spouse, but surviving spouse $225K + 50% of rest
has other issue
Not all Decedent’s issue are issue of surviving spouse $150K + 50% of rest
Who takes any property not going to surviving spouse? How is property taken?
Issue Equally (by representation)
Parents Equally, or all to the survivor
Issue of parents Equally (by representation)
Grandparents/issue 50% to paternal grandparents or survivor;
50% to maternal grandparents or survivor;
5/87
otherwise to their issue equally (by
representation);
If no surviving grandparents or issue on one
side, all to the other side
Issue of predeceased spouse(s) Equally (by representation)
Escheat to state 100%
4. Heirship & the  Heir: a person cannot become an heir until she survives the decedent
Expectancy of an Heir  Heir apparent: a person that is alive has no heirs—only heirs apparent.
Apparent
B. The Basic Structure of Intestate Succession
 In all states, after the spouse’s share, descendants take to the exclusion of ancestors and collaterals
 In nearly all states, intestacy favor only spouses and blood relations
 Across states, there is much variation in the size of the surviving spouse’s share
 Across states, there are competing methods for implementing the principle of representation by which the descendants of a predeceased
child take the child’s share
1. Surviving Spouse A. Basic Concepts
 Most persons want everything to go to the surviving spouse, thus excluding parents, siblings, and even
children.
 In most states, the surviving spouse receives at least a one-half share of the decedent’s estate
 Conduit theory: the typical intestate decedent would want to provide financial security to the surviving
spouse, and that the surviving spouse will pass any remainder to the couple’s children.
B. Traditional Intestate Distribution Scheme
 Surviving spouse’s share depends on how many of decedent’s issue survived decedent
o One Surviving Child: Spouse typically gets 50%.
o >1 Surviving Child: Spouse gets 33%.
o No Surviving Issue: Spouse gets 50%.
 Elective Share
o Policy: Balance: protect surviving spouse from being disinherited AND freedom of disposition
C. UPC Approach
 “Omitted Spouse” Statute: If a pre-marriage will disinherits the spouse, the UPC presumes the decedent
meant to leave everything to spouse, but did not get around to changing the will.
 See UPC 2-102, infra.
 Stepparents: The UPC is more concerned about the surviving spouse properly taking care of the deceased
spouse’s issue where not all of the deceased spouse’s issue are issue of the surviving spouse. In that case, the
surviving spouse takes less and the issue take more.
Who Qualifies— D. Who Qualifies—Survival Requirements
Survival  To be eligible to receive property from a decedent, a taker must “survive” the decedent.
Requirements  If the claimant fails to meet the survival requirement, the claimant is treated as if he or she predeceased the
decedent.
1. Common Law
 To qualify as an heir, the party had to prove by a preponderance of the evidence that he or she survived the
decedent by a millisecond. Whether a person survived the decedent is a question of fact.
 With the advent of the Industrial Revolution and the development of machines such as cars, trains, and
airplanes that substantially increased the potential for simultaneous death scenarios, the preponderance of the
evidence standard came under increasing criticism.
2. Uniform Simultaneous Death Act (USDA)
 The USDA codified the common law rule. The act provided that where “there is no sufficient evidence” as to
who survived whom, the party claiming a right to take is to be treated as having predeceased the defendant.
o *Now there is a 120-hour survival requirement to be proven by clear and convincing evidence
 Preponderance of the Evidence Standard: Survivorship is a fact which must be proven by the
preponderance of the evidence by the party whose claim depends on survivorship. Janus.
 It is not necessary to determine the exact moment of the survivor’s death or by how long the survivor
survived the decedent. Janus.
 Competing policies
o Avoid unnecessary probate of the same asset when two people die simultaneously
o Carry out the probably intent of the typical intestate decedent
 Problems with USDA
o (1) High costs of litigation (low burden of proof and high stakes—i.e., Campbell (wife and husband
in drowning accident))
 A higher standard of proof (i.e., clear and convincing evidence) would prevent litigation
o (2) Unfairness (all of the couple’s property ends up on one side of the family)
o (3) Encourages unseemly behavior (families suing families after deaths)
 Now in many states, a person must survive 120 hours or 5 days after decedent in order to
take
 Determining Time of Death

6/87
o Common Law: A person is death when there is irreversible cessation of circulatory and respiratory
functions—but artificial life support systems made this standard unworkable.
o Modern Trend: Death occurs when there is irreversible cessation of total brain activity.
 The Clear & Convincing Evidence Standard: In some states, to qualify as a survivor, a claimant must
prove by clear and convincing evidence that he or she survived the decedent.
 UPC 120-hour Approach: The UPC requires that to qualify as a taker, the taker must prove by clear and
convincing evidence that he or she survived the decedent by at least 120 hours. UPC 2-104; 2-702.
 Mechanics of the Survival Requirement: (1) Did the claimant actually survive the decedent? (2) Did the
claimant legally survive the decedent?
o Analyze first spouse to die: Thomas died first. (1) Can Martha prove that she actually survived
Thomas? (2) Can Martha prove that she legally survived Thomas? If not, Martha is treated as if she
predeceased Thomas. All of Thomas’s probate property passes to Thomas’s heirs.
o Analyze second spouse to die: Who takes Martha’s property? Ask whether (1) Thomas actually
survived Martha, and (2) legally survived Martha. If not, Martha has no surviving spouse. Her
probate property passes to her heirs.
 Wills & Nonprobate Instruments: The statutory survival requirement is a default rule that applies if the
written instrument does not have its own express survival requirement. If the written instrument has an
express survival requirement, it applies.
Janus v. Tarasewicz: Stanley and Theresa Janus, husband and wife, died
after ingesting Tylenol capsules which had been laced with cyanide by an
unknown perpetrator prior to its sale in sores. Stanley was pronounced dead
shortly after he was admitted to the hospital. Theresa was place on life
support systems for almost two days before being pronounced dead. Stanley
had a life insurance policy that name Theresa the primary beneficiary and his
mother, Alojza Janus (plaintiff) as the contingent beneficiary. The life
insurance company paid the policy to Tarasewicz, the administrator of
Theresa’s estate. Alojza sued, claiming there was no sufficient evidence that
Theresa survived Stanley. On appeal, the court held there was sufficient
evidence to prove that Theresa survived Stanley because efforts to preserved
Theresa’s life continued after more intensive efforts on Stanley’s behalf had failed, there was evidence that Stanley died on the way to the
hospital, and there was evidence that Theresa was not brain dead until the next day.
 Holding seems contrary to the probable intent of the decedent
2. Intestate Share of  By Representation: The surviving issue of the predeceased relative step up and represent the predeceased
Descendants relative.
English Per Stirpes  Vertical Equality—treats each line of descent equally—but not horizontal equality
 Step 1: Always divide the decedent’s property at the first generation (among the decedent’s children), even if
everyone at that generation is dead.
 Step 2: One share is given to each party who is alive, and one share is given to each party who is dead but
survived by issue.
 Step 3: The shares for each party who is dead but survived by issue drop by bloodline. Each share drops only
to the issue of the predeceased party.

Modern Per Stirpes  Horizontal equality at first generation where there is a live taker only; not vertical equality
 Step 1: Always divide at the first generation where there is a live taker.
 Step 2: One share is given to each party who is alive, and one share is given to each party who is dead but
survived by issue.
 Step 3: The shares for the dead parties survived by issue drop by bloodline (Repeating Step 1)
 Modern Per Stirpes Twist (FN 27 pg. 82 DUKE)

7/87
Modern Per Stirpes  Horizontal equality at all levels—treats each generation—but not vertical equality
Twist  Step 1: Always divide at the first generation where there is a live taker.
 Step 2: One share is given to each party who is alive, and one share is given to each party who is dead but
survived by issue.
 Step 3: The shares for the dead parties survived by issue drop by bloodline BUT flip back to English Per
Stirpes
Per Capita at Each  UPC Section 2-103. (b) [Decedent’s Descendants.] If, under Section 2-103(1), a decedent’s intestate
Generation estate or a part thereof passes “by representation” to the decedent’s descendants, the estate or part thereof
is divided into as many equal shares as there are (i) surviving descendants in the generation nearest to
the decedent which contains one or more surviving descendants and (ii) deceased descendants in the
same generation who left surviving descendants, if any. Each surviving descendant in the nearest
generation is allocated one share. The remaining shares, if any, are combined and then divided in the
same manner among the surviving descendants of the deceased descendants as if the surviving
descendants who were allocated a share and their surviving descendants had predeceased the decedent.
 Step 1: Always divide at the first generation where there is a live taker.
 Step 2: One share is given to each party who is alive, and one share is given to each party who is dead but
survived by issue
 Step 3: Pool the shares for the dead parties survived by issue. Add them together and dived the total equally
among the eligible takers at the next
 Horizontal Equality: All descendants at a generation who take will take equally.

3. Ancestors, a. Ancestors: If an intestate decedent is survived by a descendant, the decedent’s ancestors and collaterals do not take.
Collaterals, & Others In about half of the states, if there is no descendant, after deducting the spouse’s share, the rest of the intestate’s
property is distributed to the decedent’s parents. In the remaining states the spouse takes to the exclusion of the
decedent’s parents.
b. Other Ancestors & Collaterals
 Collateral kindred: all persons who are related by blood to the decedent but who are not descendants
(children, grandchildren) or ancestors (parents, grandparents) of decedent. {McL}
 Consanguinity: Relationship by blood {McL}
 First-line collaterals: descendants of the decedent’s parents, other than the decedent and the decedent’s
descendants
 Second-line collaterals: descendants of the decedent’s grandparents, other than the decedent’s parents and
their descendants
c. Three Approaches
 parentelic system: the intestate estate passes to grandparents and their descendants, and if none to great-
grandparents and their descendants, and if none to great-great-grandparents and their descendants, and so on
down each line descended from an ancestor until an heir is found. In distributing the property, the per stirpes,
moderns per stirpes, or per capita at each generation doctrines apply (or something else, depending on the
intestate succession scheme).

8/87
 degree-of-relationship system: One counts the degrees of relationship between the decedent and the relative
—those with the closest (lower) degree take to the exclusion of those of a more remote (higher) degree. To
determine the degree of relationship, use the Table of Consanguinity, or count from the decedent up to the
closest common ancestor, and then down to the live relative
 degree-of-relationship with a parentelic tiebreaker approach: The first step is to determine the degree of
relationship of the possible takers. Those of a closer (lower) degree take to the exclusion of those of a higher,
more remote degree. Then, if there are multiple takers sharing the lowest degree of relationship, under the
parentelic tiebreaker, those in the closer parentelic/collateral lines take to the exclusion of those in the more
remote parentelic/collateral lines.
 Under UPC, there is no intestate succession beyond second-line collaterals. UPC § 2-103. The intestate estate
passes to the state. UPC § 2-105.
c. Laughing Heirs: Heirs who are so far removed from the decedent that they likely didn’t know him and suffer no
sense of bereavement upon learning of his death.
d. Stepchildren & In-Laws: Under the UPC, stepchildren take if there are no grandparents, issue of grandparents, or
any closer heir, and before the state takes.
e. Half-Bloods (Stepsiblings)
 Common Law: Only whole-blooded relatives are entitled to inherit.
 UPC & Modern Trend Majority: A relative of the half-blood is treated the same as a relative of the whole
blood.
 Modern Minority
o In a few states—i.e., FL, TX, VA—a half-blood is given a one-half share
 Create one share for each half blood. Create two shares for each whole blood. Allocate
one share to each half blood and two shares to each whole blood.
 Each half-blood’s share should equal half of each whole-blood’s share.
o In a few other states, a half-blood takes only when there are no whole-blood relatives of the same
degree
Table of
Consanguinity

4. Disinheritance by  Common Law: To disinherit someone, the testator must devise his entire state to other persons. If the
Negative Will decedent’s will expresses an intent to disinherit the heir, but some or all of the decedent’s property is
distributed through intestacy and the heir in question qualifies to receive a share, the heir takes despite the
decedent’s clear intent.
 UPC & Modern Trend: Authorizes a negative will by way of an express disinheritance provision. The
barred heir is treated as if he disclaimed his intestate share, which means he is treated as having predeceased
the decedent. UPC 2-101(b).
o When a testator expressly excludes an individual in his will, the individual is excluded from taking
under both testate and intestate succession, unless the testator expressly specifies a contrary
intention. Estate of Samuelson.
9/87
o Disinheritance of either an individual or a class must be expressed and cannot be implied. Estate of
Samuelson.
 When a testator disinherits all heirs, he or she leaves no surviving spouse or kindred, and, as a consequence,
an escheat is triggered. Estate of Melon.
o Rationale: The Nevada legislature determined that testamentary freedom has primacy over the
policy disfavoring escheats. {McL}
Estate of Samuelson (ND): Ernest was the son of Harry and Hulda Samuelson. Eleanor West was the son of Harry Samuelson and Jane Grant.
Eleanor had two granddaughters, Amanda and Robin West. Ernest Samuelson’s will stated: “I give, devise, and bequeath all the rest, residue and
remainder of my estate to my mother, Hulda Samuelson.” Paragraph V provided: “I have intentionally failed to provide for my half sister,
Eleanor West.” Ernest died, survived by no descendants, nor spouse. Hulda, Harry, and Eleanor all predeceased him. Amanda and Robin
survived. The court held that Ernest’s intent to exclude Eleanor, as expressed in his will, did not also preclude Eleanor’s heirs from inheritance.
Estate of Melton: In 1975, William Melton executed a formal will, devising most of his estate to his parents and small portions to his brother
and two of his cousins, Terry and Jerry. He also indicated that his daughter was to receive nothing. In 1979, Melton executed a handwritten
codicil on the back of one of the 1975 will forms that provided his friend, Susie Kelleher, should receive a small portion of his estate (both will
forms and the codicil are hereinafter referred to as “the 1975 will”). In 1995, Melton sent a handwritten letter to Kelleher. It reads: “I am on the
way home from Mom’s funeral. Mom died from an auto accident so I thought I had better leave something in writing so that you Alberta
Kelleher will receive my entire estate. I do not want my brother Larry J. Melton or Vicki Palm or any of my other relatives to have one penny of
my estate.” Kelleher predeceased Melton. Melton was survived by a daughter, Vicki Palm and two half-sisters. The court held the disinheritance
clause contained in the 1995 letter was enforceable even though Kelleher predeceased Melton, thereby causing his devise to her to lapse, and
Melton’s estate to pass through intestate succession. The court also held that, because Melton disinherited all of his heirs in the 1995 letter, an
escheat was triggered.
C. Transfer to Children
 The basic intestate distribution scheme gives a decedent’s probate property first to his or her surviving spouse and second to his or her
descendants/issue. Often, the decedent’s issue receive a share even if there is a surviving spouse.
 Inheriting “from and through”: A parent-child relationships establishes that one can inherit from and through the other party.
o Ex: A child can inherit from a parent if the parent dies intestate, and a parent can inherit from a child if the child dies intestate.
If a child’s parent dies, and thereafter the parent’s mother (the child’s grandmother) dies intestate, the child can inherit through
the deceased parent. Likewise, a parent can inherit not only from a child, but also through a predeceased child under the
appropriate circumstances.
1. Adopted Children
a. Formal Adoption a. Formal Adoption
 Transplantation Theory of Adoption: “rebirth” into a completely different relationship. {McL}
 Adopted Children: In some states, an adopted child inherits only from and through adoptive parents and
their relatives. The child can no longer inherit from and through the natural parents. See Hall; UPC 2-119(a)
(Duke, pg. 95).
o Policy: Adopted children should be no better off than nonadopted children
o Because an adopted child has no right to inherit from the estate of a natural parent who dies
intestate, it follows that the same child may not inherit through the natural parent by way of
representation.
 In others, an adopted child inherits from both adoptive parents and genetic parents and their relatives.
 Stepparent Adoption Exception: (1) The adopting stepparent can inherit from and through the adopted child
and the adopted child can inherit from and through the adopting stepparent. (2) The natural parent loses his or
her right to inherit from and through the child, but the child retains the right to inherit from and through the
natural parent. UPC 2-119(b)(2) (Duke, pg. 95).
o *Insert Pic on CT, pg. 40.
 Adoption by relative of parent: Where a child is adopted by a relative of either natural/genetic parent, or the
spouse or surviving spouse of a relative, the child retains the right to inherit from and through both
natural/genetic parents. UPC 2-119(b) (Duke, pg. 95)
 Post-death Adoption: Where a child is adopted after the death of both natural/genetic parents, the child
retains the right to inherit through both natural/genetic parents. UPC 2-119(d) (Duke, pg. 95)
b. Adult Adoption b. Adult Adoption
 Majority: Adopted adults are treated the same as adopted minors for purposes of intestate succession.
 Reasons people adopt adults: (1) graft somebody into the ancestor’s family tree—i.e., when
grandparents/parents create long term trusts benefiting their descendants/issue/heirs—& (2) to prevent claims
against an estate by collateral relatives.
 Policy Issues:
o Child adoptions are usually public, nonconsensual, and involve a great commitment; Adult
adoptions are usually secret, consensual, and involve less commitment (more like marriages).
o Adult adoptions give rise to strategic behavior (adopt several adults to increase the share).

10/87
c. Adoption & c. Adoption & Wills & Trusts: Whether an adopted child qualifies as a “child” or “heir” for purposes of a will, trust,
Wills & Trusts or other written instrument is a question of the intent of the decedent.
 Modern Trend & General Rule: A minor adopted by A is presumptively included in a gift by T to A’s
“children,” “issue,” or “heirs.” See Minary.
 Adults: The cases are split on whether an adult adoptee is included in a class gift by someone other than the
adoptive parent or is barred by the stranger-to-the-adoption rule.
o Barred: Adoption of an adult for the purpose of bringing that person under the provisions of a
preexisting testamentary instrument when he clearly was not intended to be so covered should not
be permitted. Minary.
 An adopted child is entitled to take under the will or trust of the adoptive parent just like a biological child
 stranger-to-the-adoption rule (common law): the adopted child is presumptively barred from taking,
whatever generic word is used, if the donor was not the adoptive parent.
Hall v. Vallandingham: Earl J. Vallandingham died in 1956, survived by his widow, Elizabeth, and their four children. Two years later,
Elizabeth married Jim Walter Killgore, who adopted the children. In 1983, twenty-five years after the adoption of Earl’s children by Killgore,
Earl’s brother, William Jr., died childless, unmarried, and intestate. His sole heirs were his surviving brothers and sisters and the children of
brothers and sisters who predeceased him. The trial court determined that the four natural children of Earl, because of their adoption by their
adoptive father, Jim Walter Killgore, were not entitled to inherit from William M. Vallandingham Jr. On appeal, the court affirmed, holding that
Maryland law had eliminated the adopted child’s right to inherit from the natural parent and by so doing abrogated the right to inherit through
the natural parent by way of representation.

Minary v. Citizens Fidelity Bank & Trust Co.: Amelia Minary died, leaving a testamentary trust, which (1) devised the income from the trust
to her husband and three sons, James, Thomas, and Alfred for lifetimes and (2) at death of last surviving income beneficiary, remainder to “my
then surviving heirs.” Alfred married Myra and adopted her as his child. The trust terminated upon Alfred’s death without biological
descendants. He was survived by Myra and Thomas’s two children. The Court held
that Alfred’s adoption of his wife did not make her eligible to inherit under the
provisions of his mother’s will—she was not a “then surviving heir.”

2. Posthumous 2. Posthumous Child: Child conceived before, but born after, the parent’s death. {McL}
Children  General Rule: Courts have established a rebuttable presumption that the normal period of gestation is 280
days. Therefore, there is a rebuttable presumption that the deceased husband is the natural father if the child
is born within 280 days after the husband dies.
o For the purposes of inheritance or of determining property rights, if it is to a child’s advantage—for
purposes of inheriting from and through the natural father—to be treated as in being from the time
of conception rather than from the time of birth, the child will be so treated if born alive.
 Uniform Parentage Act § 204: Establishes rebuttable presumption that a child born to a woman within 300
days after the death of her husband is a child of that husband.
11/87
3. Nonmarital 3. Nonmarital Children: Children born outside a valid marriage. {McL}
Children  Common Law: A nonmarital child could not inherit from either natural parent, and neither natural parent
could inherit from or through the child—the child’s property escheated to the king or other overlord.
 Modern Trend/UPC: All states now permit inheritance by a nonmarital child from and through the child’s
biological mother.
a. Paternity Issues: Most states allow paternity to be established by evidence of the subsequent marriage of the
parents, acknowledgment of the father, an adjudication during the life of the father, or clear and convincing proof after
his death (use DNA evidence).
4. Reproductive 4. Reproductive Technology: Medical science now offers the making of a baby without coitus.
Technology
a. Posthumously a. Posthumously Conceived Children under Intestate Succession
Conceived Children  Posthumously Conceived Child: Child both conceived and born after the father’s or mother’s death. {McL}
 Intestacy—case law: Limited circumstances may exist in which posthumously conceived children may enjoy
the inheritance rights of “issue” under MA intestacy law: (1) PCC’s legal representation must establish
genetic relationship; (2) PCC’s legal representation must show that D affirmatively consented to posthumous
conception and support of any resulting child. Time limitations may preclude commencing a claim for
succession rights on behalf of a posthumously conceived child. In any action brought to establish such
inheritance rights, notice must be given to all interested parties. Woodward. {McL}
o Rationale: (1) Best Interests of Posthumously Conceived Children. (2) Orderly Administration of
Estates—Protection of existing heirs/creditors, Finality. (3) Reproductive Rights of Decedents.
Woodward. {McL}
 States Statutes:
o FL: PCC inherits from D only if expressly provided in D’s will. {McL}
o CA: PCC inherits from D only if: (1) D consented in signed and dated writing and designated
person to control the use of the genetic material; (2) notice to PR w/in 4 months of D’s death of
possibility; and (3) PCC in utero within 2 years of D’s death and not D’s clone. {McL}
o LA: PCC inherits from deceased P if born to SS within 3 years of D’s death and D authorized SS to
use gametes. {McL}
 UPC: PCC inherits from deceased P if: (1) P consented in signed writing or consent by clear and convincing
evidence, and (2) child is in utero not later than 36 months after P’s death, or is born not later than 45 months
after P’s death. {McL}
 Restatement (Third) Prop. Donative Transfers 2.5 cmt. 1: PCC must be born within reasonable time after
the D’s death in circumstances indicating that D would have approved of child’s right to inherit. Clear case:
PCC produced artificial insemination of the D’s widow with his frozen sperm. {McL}
 Social Security Benefits & State Inheritance Law: SCOTUS has confirmed that a posthumously conceived
child is eligible for Social Security survivor’s benefits only if the child would inherit from the predeceased
parent under state law. Astrue.
b. Posthumously b. Posthumously Conceived Children & Long-Term Trusts: Whether a posthumously conceived child qualifies as a
Conceived Children “child,” “issue,” “descendant,” or “heir” for purposes of a will or trust is a question of the intent of the decedent—not a
& Long-Term question of intestate rules.
Trusts  Where a governing instrument is silent, children born of this new biotechnology with the consent of their
parent are entitled to the same rights for all purposes as those of a natural child. In re Martin B.
 Unless the language or circumstances indicate that the transferor had a different intention, a child of assisted
reproduction must be treated for class-gift purposes as a child of a person who consented to function as a
parent to the child and who functioned in that capacity or was prevented from doing so by an event such as
death or incapacity. In re Martin B.
 Cons: Potential for strategic behavior (have many PCC); The amount others in the class get smaller share
Woodward v. Comm’r of Social Security: In January, 1993, Lauren Woodward and Warren Woodward were informed that the Warren
(husband) had leukemia. Advised that the husband’s leukemia treatment might leave him sterile, the Woodwards arranged for a quantity of the
husband’s semen to be medically withdrawn and preserved, in a process commonly known as “sperm banking.” The husband died in 1993, and
the wife was appointed administratrix of his estate. In 1995, the wife gave birth to twin girls. The children were conceived through artificial
insemination using the husband’s preserved semen. The wife applied for two forms of Social Security survivor benefits: “child’s” benefits. The
Social Security Administration rejected the wife’s claims on the ground that she had not established that the twins were the husband’s “children”
within the meaning of the Act because they “are not entitled to inherit from [the husband] under the Massachusetts intestacy and paternity laws.”
The Court held that the children could not enjoy the inheritance rights of natural children under Massachusetts’s law of intestate succession
because the fact that the husband is the genetic father of the wife’s children is not sufficient to establish that the husband is the children’s legal
father for purposes of the distribution of his intestate property.
In re Martin B.: Grantor created trusts: “During the life of Grantor’s wife, principal to
Grantor’s wife and Grantor’s issue in Trustee’s discretion. At Wife’s death, remainder
to Grantor’s then living Issue by representation.” Grantor (who was a life income
beneficiary of the trusts) died in July 2001, survived by his wife Abigail and son
Lindsay, but predeceased by his son James. Before dying, James deposited a sample of
his semen at a laboratory with instructions that it be cryopreserved and that, in the event
of his death, it be held subject to the directions of his wife Nancy. Although at his death
James had no children, three years later Nancy underwent in vitro fertilization with his
cryopreserved semen and gave birth in 2004 to a boy (James Mitchell) and then to
12/87
another boy in 2006 (Warren). The Court held that, for these instruments, the term “issue” and “descendants” include children conceived by
means of in vitro fertilization with the cryopreserved semen of the grantor’s son because the Grantor had intended all members of his bloodline
to receive their share.
5. Advancements &
Hotchpot
a. Advancements at a. Advancements at Common Law: An advancement is an inter vivos gift made by the decedent to the heir—i.e., a
Common Law prepayment At common law, any lifetime gift by the decedent to a child was presumed to be an advancement of the
child’s intestate share. To avoid application of the doctrine, the child had the burden of establishing that the transfer was
intended as an absolute gift that was not to be counted against the child’s share of the estate.
 Rationale: A parent would want equal distribution among children
 Only applies in the context of intestacy
Child Predeceased Parent After Advancement: When a parent makes an advancement to a child and the child
predeceases the parent, the amount of the advancement is deducted from the shares of the child’s descendants if other
children of the parent survive.
b. Hotchpot b. Hotchpot
 Hotchpot Calculation: All advancements are added back into the parent’s probate intestate estate to create
the “hotchpot.” Then the hotchpot is divided equally among the decedent’s heirs. Any advancement received
by a child is counted against the child’s share of the hotchpot. The child actually receives from the parent’s
estate only their share of the hotchpot minus any advancement the child has received.
o (1) Hotchpot = [advancements + estate]; (2) Share Per Child = [Hotchpot / # children]; (3) Child
One’s Share = [Share Per Child – advancements to Child One]
 Advancement Exceeds Share: If one child has received an advancement that is greater than the amount she
is to receive under the hotchpot calculation (the “Share Per Child”), the child does not have to give back a
portion of this amount. The child will stay out of the hotchpot (don’t add that child’s advancement into the
hotchpot and don’t count the child as a child), and the decedent’s Hotchpot will be equally divided among the
other children.
 Child Predeceases: advancement doctrine still applies
o Example: pg. 127 (Duke)
c. Advancements in c. Advancements in Modern Law
Modern Law  Largely because of the difficulty of proving the donor’s intent, many states have reversed the common law
presumption of advancement. In these states, a lifetime gift is presumed not to be an advancement unless it is
shown to have been intended as such.
 Writing Requirement: Some states and UPC 2-109(a) require that the intention to make an advancement be
declared in a writing signed by the parent or child.
Donee Predeceases: Under the UPC, if the donee child predeceases the donor parent, the advancement is not
taken into account in determining the share of the child’s descendants (unless the writing expressly provides
so).
6. Guardianship & 6. Guardianship & Conservatorship of Minors: A minor has neither the legal capacity to manage property nor the
Conservatorship of legal power to make most choices about how and where to live. Clients with young children should be advised to
Minors provide for the possibility that their children might be orphaned.
a. Guardian of the a. Guardian of the Person of Minor Child
Person of Minor  A guardian of the person has responsibility for the minor child’s custody and care.
Child  If both parents die while a child is a minor, the court will appoint a guardian of the person, usually from
among the nearest of relatives.
o Court usually appoints close family member.
 For a parent with a minor child, one of the principal reasons for making a will is to designate a guardian of
the person for the child.
 No ability to deal with minor’s property UNLESS otherwise designated.
b. Property b. Property Management Options
Management  A guardian of the person has no authority to deal with the child’s property.
Options Several alternatives for property management are available: (1) guardianship of the property; (2)
conservatorship; (3) custodianship; and (4) trusteeship.
1. Guardianship of the Property.
 A guardian of the property, who does not have title to the ward’s property, usually cannot change investments
without a court order.
 The guardian has the duty of preserving the specific property left to the minor and delivering it to the ward at
age 18, unless the court approves a sale, lease, or mortgage.
 The guardian ordinarily can use only the income from the property to support the ward; the guardian needs
court approval to go into principal.
 Guardianship for a minor’s property is like going through a continuous probate until the child reaches the age
of majority. It should be avoided.
 Ends at 18
2. Conservatorship
 The guardian of the property has been renamed the conservator and given “title as trustee” to the protected
person’s property along with investment powers similar to those of a trustee. Appointment and supervision by

13/87
a court is still required, but a conservator has far more flexible powers than a guardian, and only one trip to
the courthouse annually for an accounting may be necessary.
 Ends at 18
 Should be avoided.
3. Custodianship
 A custodian is a person who is given property to hold for the benefit of a minor under the Uniform Transfers
to Minors Act (UTMA) or its predecessor, the Uniform Gifts to Minors Act (UGMA).
 The custodian has the right to manage the property and to reinvest it. However, the custodian is a fiduciary
and is subject to “the standard of care that would be observed by a prudent person dealing with property of
another.”
 The custodian is not under the supervision of the court, and no accounting to the court annually or at the end
of the custodianship is necessary, but an interested party may require on if she wishes.
 Terminates when minor turns 18 or 21—property goes to child
4. Trusteeship
 Best option for a significant amount of money (has higher up-front costs)
 A trust is the most flexible of all property management systems.
 A trust can postpone possession until the donor thinks the child is competent to manage the property.
D. Bars to Succession
 The doctrines in this section address the issue of whether there are situations where an otherwise eligible taker is nevertheless barred
from taking.
1. The Slayer Rule A. Homicide: Where a party who otherwise is entitled to take from a decedent kills the decedent, the equitable
principle that one should not profit from one’s own wrongdoing argues against permitting the killer from taking.
 Manslaughter: Voluntary manslaughter comes within the scope of the homicide doctrine, but involuntary
manslaughter does not.
B. Judicial Approaches—No Slayer Statute: If the jurisdiction does not have a statute addressing the issue of a slayer
taking by decent or distribution from the person he has killed, the courts are split:
 (1) The legal title passed to the slayer and may be retained by him in spite of his crime.
 (2) The legal title will not pass to the slayer because of the equitable principle that no one should be permitted
to profit by his own fraud, or take advantage and profit as a result of his own wrong or crime.
 (3) The legal title passes to the slayer, but equity holds him to be a constructive trustee 1 (to prevent unjust
enrichment) for the heirs or next of kin of the decedent. Mahoney.
C. Slayer Statutes—Variation Across States: Nearly every state has enacted a statute dealing with the rights of a
killer in the estate of a victim, but the details of these statutes vary considerably and often leave gaps to be resolved by
courts.
 Which assets are covered (Does statute bar slayer from taking victim’s probate and nonprobate assets?)
o UPC: The slayer statute applies to all types of property: nonprobate, probate testate, and probate
intestate. UPC § 2-803. Applies to both nonprobate and probate assets. Adopted by Utah.
o Statute Covers Probate Property Only: Some statutes expressly cover probate property only.
 Constructive Trust: One should argue that a constructive trust should be imposed on the
nonprobate property based on the equity principle that one should not benefit from one’s
own wrongdoing. The constructive trust would be in favor of those who would have taken
if the killer had predeceased the victim.
 Cardozo: “A constructive trust is the formula through which the conscience of
equity finds expression. When property has been acquired in such
circumstances that the holder of the legal title may not in good conscience
retain the beneficial interest, equity converts him into a trustee.” {McL}
 Who takes if the slayer does not?
o Majority: The prevailing view is that the killer is treated as having predeceased the victim. UPC 2-
803 provides that the killer is treated as having disclaimed the property. Under UPC 2-1106, a
disclaimant is treated as having died immediately before the victim.
o Killer’s Issue: Some states extend the bar by statute to the killer’s descendants. Other states limit
the right of the killer’s descendants to take by case law.
 If there is no bar, the killer is treated as if he predeceased the victim. Thus property passes
under intestacy pursuant to the per stirpes et al. doctrines.
 Is a criminal conviction required? UPC 2-803(g) and most states provide that a final criminal conviction of
a felonious and intentional killing is conclusive. Acquittal, however, is not dispositive of the acquitted
individual’s status as a slayer. In the absence of a conviction the court must determine whether, under the civil
standard of preponderance of evidence rather than the criminal standard of beyond a reasonable doubt, the
individual would be found criminally accountable for the killing. If so, the individual is barred.
 “Can a donor opt out of the slayer rules?”: Majority: No. Wisconsin: Yes. Louisiana: Maybe.

1 Constructive Trust: If a defendant is unjustly enriched by the acquisition of title to identifiable property at the expense of the claimant or
in violation of the claimant’s rights, the defendant may be declared a constructive trustee, for the benefit of the claimant, of the property in
question and its traceable product. The sole duty of the constructive trustee is to convey the property to its rightful claimant.

14/87
In re Estate of Mahoney: The wife was convicted of manslaughter in the death of her husband. The husband died intestate, and the wife
claimed her intestate share. Vermont had no homicide statute, yet the court said that it would be inequitable (unjust enrichment) to permit the
wife to profit from her own wrongdoing and adopted the constructive trust approach to the issue to ensure that the killer did not profit from her
own wrongdoing.
2. Disclaimer A. Disclaimer: A disclaimer is an heir or devisee’s refusal to take the property (usually to reduce taxes or to keep
property from creditors).
 Conceptual Approach—Treat as if Predeceased: If a party disclaims, the disclaimant is treated as if she
predeceased the decedent. The property in question is then distributed as if the disclaimant predeceased the
decedent. The property is distributed to the next eligible taker under the various rules governing who takes in
the event a taker predeceases the decedent (i.e., will, intestate succession).
 Mechanical Approach—Focus on Disclaimed Interest: Only the disclaimed interest passes as if the
disclaiming party predeceased the decedent, so calculate the disclaimed party’s share before giving effect to
the disclaimer. The disclaimer does not affect the size of the share going to the next-in-line takers.
B. Common Law/Statutory Law
 Common Law: (1) Intestate: An intestate successor could not prevent title from passing to him. If the heir
refused to accept the inheritance, the common law treated the heir’s renunciation as if title passed to the heir
and then from the heir to the next intestate successor. (2) Testate: If a person tied testate, the devisee could
refuse to accept the devise, thereby preventing title from pass to the devisee.
 Modern Trend/UPC: Almost all states have enacted disclaimer legislation that treats the disclaimant as
having died before the decedent or before the time of distribution. See UPC 2-1105 to -1106.
C. Benefits of Disclaiming: A party may disclaim a testamentary gift to: (1) redistribute property; (2) avoid gift tax
consequences; (3) avoid ordinary creditors.
1. Redistribute Property: Disclaimers can be used to adjust who takes and how much they take after the death of the
decedent.
 Strategic Disclaimer under the UPC: A strategic disclaimer is not possible because only “the disclaimed
interest” passes to the descendants of a disclaimant who take by representation. See UPC 2-1106(b)(3) (*Add
pic in Duke, at 141)
2. Avoiding Taxes: If one accepts the property and then gives it to the next taker in line, gift tax consequences to the
transfer may result. If, however, one disclaims and the legal effect is simply to pass the property in question to the next
taker in line, the disclaimer has no gift tax consequence.
 Majority: Most state disclaimer statutes require that a disclaimer be made within 9 months of the creation of
the interest being disclaimed.
 Minority/UPC: There is no time limit
3. Avoiding Creditors: If an heir or devisee is facing creditors’ claims, such that any inheritance or devise would go
directly to the creditors, the heir or devisee can elect to disclaim the property in question to avoid the property going to
the creditors.
 Avoid Ordinary Creditors: If A dislcaims her interests in O’s estate, most cases have held that A’s ordinary
creditors cannot reach the disclaimed property. So long as the disclaimer was made prior to the filing of a
bankruptcy petition, the federal courts will respect the state law relation-back doctrine for claims against a
bankrupt debtor.
o Bankruptcy Exception: If, however, a bankruptcy petition is filed before the debtor disclaims,
courts almost invariably hold that the disclaimer is ineffective under federal bankruptcy law.
o Minority: An insolvent debtor who is not already in bankruptcy may not use a disclaimer to avoid
his creditors.
 Federal Government as Creditor Exception: Where the federal government is a creditor of the disclaimant
(for tax purposes—federal tax lien—or under Medicaid reimbursement provisions), the property disclaimed
often is subject to the claim of the federal government (no relation back). Dry v. U.S.
o Post-Mortem Tax Planning with Disclaimers: Example in PP. Son should only take $ 11.2
million by disclaiming $3.8 million. The $3.8 million would pass to the mother with no estate tax at
the father’s death. However, there can be no legally binding agreement between mother and son.

 Relation Back--Majority/UPC: Most disclaimer statutes provide that a disclaimer relates back for all
purposes to the date of the decedent’s death. In an intestate estate, the disclaimer “takes effect . . . as of the
15/87
time of the intestate’s death.” UPC 2-1106(b).
3. Wills: Formalities & Forms
 Whether a will has been properly executed is a function of two variables: (1) the jurisdiction’s statutory Wills Act formalities, and (2) the
jurisdiction’s judicial philosophy as to what degree of compliance with the Wills Acts formalities is acceptable.
A. Execution of Wills
 Three types of wills: (1) attested wills; (2) notarized wills; and (3) holographic wills.
 Every state probate code includes a provision, knowns as the Wills Act, which prescribes rules for making a valid will.
1. Executing a Valid 1. Model Execution Ceremony: Almost all states have a statute—such as UPC § 2-506—that recognizes as valid a
Attested Will—Will will executed with the formalities required either by the state where the testator was domiciled at death, by the state
Formalities where the will was executed, or by the state where the testator was domiciled when the will was executed.
 Example of Model Execution Ceremony: Duke, at 168-69
 Typical Attested Will: “I, _______, hereby make my will, and I revoke all other wills and codicils that I have
previously made.” {McL}
2. Safeguarding a Will—Majority: A lawyer who has drawn a will or other estate planning documents for a client
may offer to retain the executed originals of the documents subject to the client’s instructions.

16/87
A. The Core A. General Statutory Requirements: There are three core formalities for the making of an attested will: (1) writing;
Formalities (2) signature; and (3) attestation. {McL}
B. {McL} UPC § 2-502(a). Execution; Witnessed Wills. (a) [Witnessed Wills.] Except as otherwise provided in
subsection (b) and in Sections 2-503, 2-506, and 2-513, a will must be:
 (1) in writing;
 (2) signed by the testator or in the testator’s name by some other individual in the testator’s conscious
presence and by the testator’s direction;2 and . . .
 (3) . . . (A) signed by at least two individuals, each of whom signed within a reasonable time after the
individual witnessed3 either the signing of the will as described in paragraph (2) or the testator’s
acknowledgment of that signature or acknowledgement of the will.
C. Interested Witnesses: Historically, there has been a requirement there must be at least two disinterested witnesses.
If there are not, the scope of the remedy has evolved over time, with jurisdictions split over which remedy is
appropriate.
 Interested Witness: A witness that is a beneficiary of the will.
 Only applies to the witnesses that are interested—not to the disinterested witnesses (For example, a statute
requires 2 witnesses. 4 witness the will. 2 are disinterested. 2 are interested. The will is valid because of the 2
disinterested witnesses).
 Disinterested Beneficiary: When one witness receives a bequest under the will, but is actually adversely
affected by the admission of the will to probate, the witness has nothing to gain by the admission of the will,
and the interested witness statute is satisfied. Morea.
1. Invalidate Will (Common Law/Minority of States): In England, an interested witness was not permitted to testify
in court, hence a will attested by an interested witness could not be proved in probate.
2. Void Interested Witness’s Gift (Slim Majority): The purging statute allowed a will attested by an interested witness
to be admitted to probate, but it voided (purged) any bequest to the interested witness.
 Rebuttable Presumption of Misconduct
3. Purging Approach—Void Excess (Slim Majority): The witness forfeits only the excess benefit afforded to the
witness by the will.
 Rebuttable Presumption of Misconduct: A devise to a witness triggers a rebuttable presumption of undue
influence, duress, or fraud. If the interested witness cannot rebut the presumption, apply the purging
approach.
4. Abolish the Doctrine (UPC/Minority): Does not require that any of the witnesses be disinterested. UPC 2-505(b)
provides that a will is valid even if witnessed by an interested person and without purging the interested witness of his
devise.
 Criticism of Purging Statutes: Purging statutes do not prevent fraud because schemers are cleverer and find
a way not to be a witness. Consequently, the statutes are generally applied to innocently interested witnesses.
 Good Attorneys: Still use disinterested witnesses. Purpose of the UPC is to avoid the harsh result of purging
innocent beneficiaries.
B. Functions Served  Evidentiary: The formalities serve an evidentiary function by providing the probate court clear and
by Formalities convincing evidence of the authenticity and finality of the will without the benefit of live testimony from the
testator. It establishes that the document truly reflects the testator’s last will and testament.
 Protective: The formalities serve a protective function by protecting the testator from being unduly
influenced or otherwise coerced at the time of executing her will, and lack of capacity.
o Modern: Genuineness of testator’s signature.
 Ritualistic/Cautionary: The formalities serve a ritualistic function by impressing upon the testator the
solemnity of making a will.
 Channeling: The formalities serve a channeling function by encouraging individuals to consult with an
attorney to draft and supervise the execution of their wills, thereby facilitating the probate of the will and
decreasing administrative costs—i.e., standardize the form of wills; judicial economy; efficiency for testators.
Stevens v. Casdorph: Two months prior to his death, Casdorph took Mr. Miller
to a bank to execute his will. Mr. Miller signed the will in front of a bank
employee/public notary, who then took the will to two other bank employees for
the purpose of having each of them sign the will as witnesses. Mr. Miller did not
accompany the notary to the employees’ work area, and the employees testified
during their depositions that they did not actually see the decedent place his
signature on the will. The will left the bulk of Mr. Miller’s estate to Casdorph
(nephew). After Mr. Miller’s death, the Stevenses (nieces) challenged the will
claiming improper execution under West Virginia law (“[T]he signature shall be
made or the will acknowledged by him in the presence of at least two competent
witnesses, present at the same time; and such witnesses shall subscribe the will in
the presence of the testator, and of each other.”). On appeal, the court applied
strict compliance and invalidated the will because the two witnesses were not
there at the same time when he signed the will, and they did not subscribe the will in the presence of Mr. Miller, and of each other.

2 Elderly or blind person


3 “actually observed”—telephonic acknowledgement by testator to witnesses does not count
17/87
Estate of Morea: Of the three witnesses to the decedent’s will, two took under the will, and the other was disinterested. Because there were not
two disinterested witnesses, an issue arose under the interested witness doctrine. One of the witnesses who took under the will was decedent’s
son, whose legacy under the will was less than his intestate share. The second was decedent’s friend. Under the New York interested witness
statute, a party who is both a witness and a beneficiary takes the lesser of his intestate share or his legacy under the will. The court held that the
will was valid because there were at least two attesting witnesses who had nothing to gain by the admission of the will to probate: (1) the son
was actually adversely affected by the admission of the will because his intestate share would be greater than his bequest and (2) the third
witness received nothing from the will.
In re Will of Ranney: The testator’s lawyer had meant to include at the
end of the will a one-step self-proving affidavit, but he mistakenly used
the language of a two-step affidavit instead. The court, noting that the will
had not been first executed in strict compliance with the Will’s Act, held
that will may be probated with clear and convincing evidence that it
substantially complies with the Wills Act.
In re Estate of Hall: The testator (Jim) and his wife (Betty) went to their
attorney’s office, modified a draft of their joint will, and then signed it on
the advice of the attorney that it would constitute a valid will until they
signed the final version. Jim and Betty executed the joint will, the
attorney signed it (there was nobody in the office at the time—no
witnesses), and then Jim and Betty returned home where they destroyed
their original wills. Jim died before executing the final version. Under the
harmless error doctrine, the court validated the will, despite it not being attested, on the ground there was
clear and convincing evidence that Jim intended the document to be his will: (1) The joint will specifically
revoked all previous wills; (2) Jim directed Betty to destroy all previous wills; and (3) Betty testified her
lawyer told them the joint will was to stand as a will until they executed another one. The court dispensed
with the attestation requirement.
In re Probate of Will and Codicil of Macool: Louise and Elmer Macool were
married. Louise’s first will left all her property to Elmer, with her 7 stepchildren as
contingent beneficiaries. After Elmer dies, she went to her attorney and gave him a
handwritten note, instructing him to draft a new will for her that would give her two
nieces the same share as her stepchildren. Her attorney prepared a “rough” will, but
before she had a chance to see it, Louise died (an hour after the meeting). One of the
nieces offered the draft for probate. The appellate court held that the plaintiff failed to
establish, by clear and convincing evidence, that Louise intended the document
denoted by Calloway as a “rough” draft to be her last and binding will: Because of
her untimely death she had no opportunity to read, review, clear up any ambiguity, or
express her final assent to the “rough” draft. The court was not willing to dispense
with the attestation and signature requirements.
2. Notarized Wills A. UPC 2-502(a). Execution; Notarized Wills. (a) [Notarized Wills.] Except as otherwise provided in subsection (b)
and in Sections 2-503, 2-506, and 2-513, a will must be:
 (1) in writing;
 (2) signed by the testator or in the testator’s name by some other individual in the testator’s conscious
presence and by the testator’s direction; and . . .
 (3) . . . (B) acknowledged by the testator before a notary public or other individual authorized by law to take
acknowledgements.
B. Rationale: (1) a single notary can serve the functions underlying the Wills Act (i.e., evidentiary, cautionary,
channeling, protective) as well as a pair of witnesses and (2) a notarized will would almost always be upheld under the
harmless error doctrine.
C. Adopted: UPC 2-502 is a recent innovation and only Colorado and North Dakota have adopted it. Utah has not.
3. Holographic Wills A. Holographic Will: A holographic will must be (1) in Testator’s handwriting & (2) signed by the testator. {McL} It
need not be attested by witnesses. (need holographic will statute; need testamentary intent)
o Ritual: No ritual or ceremony—i.e., letter, writing in notebook
o Evidentiary: Handwriting is excellent evidence of genuineness.
o Protective: No witnesses to testify testator intended document to be will.
o Channeling: No channeling—testator’s have no legal training, can’t spell, doesn’t have
understanding of grammar.
 Disfavored by Courts—Issues: The absence of witnesses raises a number of concerns: (1) whether the
testator had capacity when the will was executed; (2) increased potential for fraud and undue influence; (2)
whether the decedent rafter had requisite “testamentary intent”; (4) how to resolve conflicts between multiple
wills; and (5) higher costs of administration.
 Holographic Will States (2012): Duke, at 197.

18/87
 UPC Approach: The UPC recognizes holographic wills.
o UPC § 2-502(b). Holographic Wills. (b) [Holographic Wills.] A will that does not comply with
subsection (a) is valid as a holographic will, whether or not witnessed, if the signature and material
portions4 of the document are in the testator’s handwriting.
B. Requirements: Most states compensate for the concerns surrounding holographic wills by adding additional
requirements, such as . . .
 Writing: HW must be in writing.
 Signed: HW must be signed. Anything the testator intends as his or her signature qualifies as a valid
signature. Only the testator can sign a HW.
 Dated: Some states require that the HW will be dated. The UPC does not.
 Handwritten: HW must be in the testator’s handwriting. The jurisdictions are split over how much must be
in the testator’s handwriting. See First Generation, Second Generation, and Third Generation Holographic
Will Statutes below.
 Testamentary Intent: The document must express testamentary intent.
a. Testamentary a. Testamentary Intent: Testamentary intent is the intent that the document constitutes the person’s last will and
Intent testament—the intent that this document be probated as the decedent’s will. {CT}
 The test is not whether the decedent intended the document to be his will, but whether the decedent intended
the document to have legal effect on his death. Deeds, mortgages, letters, powers of attorney, agreements,
checks, notes, etc., have all been held to be, in legal effect, wills. In re Kimmel’s Estate.
 Examples: Words such as “save this” support a finding of testamentary intent. {CT}
1. 1st Generation: The document must be entirely handwritten and the words much show a testamentary intent. {CT}
2. 2nd Generation: If the jurisdiction requires only that the material provisions be in the testator’s handwriting, a sub-
issue is whether testamentary intent is a “material provision.” If it is, testamentary intent must be discernible
exclusively from the handwritten portions of the document.
 Example: In re Estate of Gonzalez.
3. 3rd Generation: Testamentary intent can be derived from the handwritten material, the non-handwritten provisions,
or other extrinsic evidence. (See “Extrinsic Evidence infra)
b. Preprinted Will a. Preprinted Wills Form: When a person obtains a preprinted will form, completes the form by hand, and then signs
Forms it, but does not have it attested by two witnesses, it is not entitled to probate as a formal will. However, it may be
entitled to probate as a holographic will if enough of the text was handwritten by defendant (See “Signature and
Handwriting” infra).
 Example: In re Estate of Gonzalez.
c. Signature & 1. Signature: In almost all states that permit holographs, the testator may sign the will at the end, at the beginning, or
Handwriting anywhere else on the face of the document. But if the holograph is not signed at the end, there may be doubt whether
the testator intended his name to be a signature
2. Handwriting: The jurisdictions are split over how much of the HW must be in the testator’s handwriting. The
statutes fall roughly into three generations.
(a) First Generation: “entirely written, signed, and dated.” HW must be “entirely written, signed, and dated” in the
testator’s handwriting. {McL}
 “entirely signed”: no stamps (i.e., family seal), no minor additions of another person (i.e., banker)
 “entirely dated”: If the day, year, or month of the date was omitted, the dating of the instrument was
insufficient to allow probate (i.e. May 1965).
(b) Second Generation (1969 UPC): “material provisions.” Only “the signature and the material provisions” of the
will must be in the testator’s handwriting. {McL}
 Material Provisions: The material provisions are those that affect the disposition of the testator’s property:
the “who gets what,” any administrative provisions (i.e., appointment of a personal representative or

4 i.e., property and beneficiaries


19/87
guardian), and maybe testamentary intent.
 surplusage theory: the handwritten portion of the instrument should be given effect as a holographic will if it
makes sense without the text not written by the testator.
 Testamentary Intent: Some courts (i.e., In Estate of Muder--*Duke, at 204) have been willing to look
beyond the handwriting (to the preprinted words) to make sense of a disposition or to find testamentary intent
where part of a disposition (such as “I give”) or the language necessary to establish testamentary intent (such
as “last will and testament”) is not handwritten. Other courts do not look to the preprinted words for context
(example: In re Will of Ferree *Duke, at 205).
o Example: In re Estate of Gonzalez.
o Partially Preprinted Will Form: “I give to my wife all my earthly possessions. T’s signature.”
Valid holographic will. In re Estate of Muder.
o Entirely Preprinted Will Form: Not a valid attested will; Not a valid holographic will. In re Will
of Ferree (NJ) {McL}

(c) Third Generation (1990 UPC): “material portions” and extrinsic evidence allowed. The UPC requires only that
the signature and “material portions” be handwritten. UPC § 2-502(b). {McL} However, under the UPC harmless error
doctrine, a holographic will that otherwise would fail for want of material provisions in the testator’s handwriting still
might be valid as long as the document is signed as there is clear and convincing evidence that the testator intended the
document to be his or her will. UPC § 2-503. {McL}
 Immaterial Portions: “I give, devise, and bequeath”
 Material Portions: property and beneficiaries.
 Example: Gonzalez (court moves to third generation)
 Testamentary Intent: “Intent that a document constitutes the testator’s will can be established by extrinsic
evidence, including, for holographic wills, portions of the document that are not in the testator’s
handwriting.”
d. Testamentary  Extrinsic evidence may provide factual support for the trial court’s determination of testamentary intent. In re
Intent—Extrinsic Estate of Kuralt.
Evidence  UPC § 2-502(c) [Extrinsic Evidence]. Intent that a document constitute the testator’s will can be established
by extrinsic evidence, including, for holographic wills, portions of the document that are not in the testator’s
handwriting.
In re Kimmel’s Estate: The decedent handwrote a letter to two of his children. The letter talked about some family matters and the weather.
Near the end of the letter he wrote, “I have some very valuable papers I want you to keep fore me so if enny thing hapens all the scock money in
the 2 Bank liberty lones Post office stamps and my hoome on Horner St goes to George Darl & Irvin Kepp this letter lock it up it may help you
out . . . . Father.” The decedent mailed the letter and died suddenly that afternoon. Some of his children offered the letter for probate. On appeal,
the court held that the letter was testamentary in characters because the Testator was sick at the time and he wrote “if enny thing hapens” and
“kepp this letter lock it up it may help you out.” The court also held the signature on the letter (“Father”) was in compliance with the Wills Act
because it was the method he always used in signing such letters.
In re Estate of Gonzalez (ME): Decedent filled out a preprinted will form, giving his estate to three of his five children. Nobody saw him fill
out the form, but sister-in-law saw him sign it. The form was not signed by any witnesses. The decedent showed the document to his brother and
Elisabeth and told them he intended to rewrite it more neatly on a second blank commercially printed form will. The brother, his wife, and their
mother signed the blank form as witnesses. The decedent suddenly died. The first document was offered for probate. The court admitted the
document to probate as a valid holographic will because his handwritten words could be read in the context of the preprinted words.
Specifically, the printed phrase, “BE IT KNOWN that I ____, a resident of ____, County of ____, in the State of _____, being of sound mind, do
make and declare this to be my Last Will and Testament expressly revoking all prior Wills and Codicils at any time made ,” is an introductory
phrase that may be preprinted, and, when filled in the by the testator’s writing, can become a valid statement of testamentary intent in a
holographic will.
In re Estate of Kuralt (MT): Charles Kuralt was married to Suzanne Baird, but he also had a long-term (30-year) and intimate relationship
with Pat Baker. In 1989, Kuralt executed a valid holographic will bequeathing all his property in Montana to Pat. In 1994, he executed a formal
will that left all his property to Suzanne and their children. In 1997, Kuralt began the process of transferring inter vivos the 90 acres of the
Montana property to Pat, but he was hospitalized and died before finishing the process. While in the hospital, he handwrote and signed a
20/87
document that he sent to Pat. The document provided in pertinent part, “I’ll have the lawyer visit the hospital to be sure you inherit the rest of
the place in MT if it comes to that.” Kuralt did not have his lawyer do so. After Kuralt died, Pat offered the will for probate. On appeal, the
Court held that there was substantial credible evidence that the 1997 letter expressed Kuralt’s intent to effect a posthumous transfer of his
Montana property to Shannon because (1) Kuralt had transferred other property in Montana to Pat for no real consideration; (2) Kuralt and
Shannon had a close, personal relationship, which included significant financial support; (3) Kuralt wrote the letter with possible death looming;
and (4) Kuralt used the term “inherit.” Thus, the 1997 letter was a codicil to Kuralt’s 1994 formal will.
*{McL} does not believe Kuralt was decided correctly*
4. Executing a Valid
Attested Will—
Judicial Philosophy
1. Strict A. Strict Compliance Rule (Majority/Traditional Rule): For a will to be admitted to probate, it must be in strict
Compliance compliance with the formal requirements of the applicable Wills Act. If there is any deficiency in the execution ceremony,
the document is not a valid will.
 Example—Testamentary Intent: The creation of a valid will requires concurrently both (1) testamentary intent
and (2) a written instrument, executed in the manner provided by West Virginia law. Mere intent by a testator to
execute a written will is insufficient. Stevens.
 Example: The Wills Act mandated a testator to sign or acknowledge his signature “in the presence of two
witnesses present at the same time.” The decedent acknowledged his signature on his will before two witnesses,
one at a time. Because the decedent did not acknowledge the signature in the presence of two witnesses present
at the same time, the court refused to admit the will to probate—even though the court acknowledged it had no
doubt the testator intended the document to be his will. In Re Groffman.
B. Typical Formalities: The Wills Act formalities vary from jurisdiction to jurisdiction, but some common examples are
(1) Writing; (2) Signature; (3) Signing by Another; (4) Witnesses; (5) Presence; (6) Order of Signing; (7) Writing below
Signature;(8) Delayed Attestation; (9) Videotaped wills; and (10) Electronic Wills.
 (1) Writing: To have a valid will, there must be a writing.
o Nuncupative/Oral Wills: Some states permit nuncupative/oral wills under certain circumstances. A
nuncupative will is one created under peril of death.
 (2) Signature: The writing must be signed. A signature is anything the testator intends as her signature—even if
it is not the person’s full name.
o A mark: Any mark, even an “X,” may qualify as the testator’s signature if that is what the testator
intends as her signature.
 (3) Signing by Another: The will may be signed by someone other than the testator, as long as the person signs
the testator’s name, in the testator’s presence, and at the testator’s express direction.
 (4) Witnesses: Most jurisdictions require that the testator sign or acknowledge in the presence of at least two
witnesses, who are present at the same time. They must sign the will.
o Acknowledgement: If a witness acknowledges his/her signature on a will in the physical presence of
the other subscribing witness and the testator, then the will is properly witnessed within the terms of
West Virginia law. Wade.
o Disinterested: See infra Subsection D.
o Age Requirement: Some states require witnesses to be 18. UPC does not. All states require witnesses
to be competent: (1) able to tell the truth; (2) able to testify in court; (3) able to perceive and remember
events accurately; and (4) able to respond to questions posed on cross-examination.
 Bottom Line: Use Adults.
 (5) Presence: The testator must sign in the presence of the witnesses, and, under the traditional approach, the
witnesses must sign in the presence of the testator. There are two approaches to the presence requirement:
o Line of Sight (England & some States): The testator does not actually have to see the witnesses sign
but must be able to see them were the testator to look. See Groffman; Stevens.
 Blind Testator Exception: The test is whether the testator would have been able to see the
witnesses sign from where the testator was standing or sitting if the testator had the power of
sight.
o Conscious Presence (Some States): The witness is in the presence of the testator if the testator,
through sight, hearing, or general consciousness of events, comprehends that the witness is in the act
of signing. This approach is much broader than the line of sight approach.
o Modern Trend/UPC: The modern trend and UPC approach is to abolish the requirement that the
witness sign in the presence of the testator. Under such statutes however, the testator must sign or
acknowledge in the (conscious) presence of the witness. See UPC § 2-502(a)(3).
 (6) Order of Signing
 (7) Writing below Signature
 (8) Delayed Attestation
 (9) Videotaped Wills: No court has upheld a videotaped will.
 (10) Electronic Wills: Nevada permits electronic wills executed under very strict requirements. Otherwise it is
presumed that an electronic will would not satisfy the traditional Wills Act formalities unless the jurisdiction
were to apply substantial compliance or harmless error.
 (11) Uniform Electronic Wills Act:
Section 4 – Execution – “Text Record” that is signed by the testator
Section 11 – Revocation – Subsequent Writing (including electronic), Physical (“revocatory”) act.
21/87
2. Substantial A. Substantial Compliance Doctrine: Under substantial compliance, even if a will is not executed in strict compliance
Compliance with the jurisdiction’s Wills Act formalities, the court can admit the will to be probated if (1) clear and convincing
evidence shows that the testator intended this document to constitute his or her last will and testament, and (2) clear and
convincing evidence shows that the will substantially complies with the statutory Wills Act formalities. UPC 2-503 (1990)
 Problem: Courts were hesitant to follow the substantial compliance doctrine
 Example: In re Will of Ranney
 “Studied Disregard”: The substantial compliance doctrine does not allow for the studied disregard of the
formalities still required by statute—i.e., will not attested by two witnesses, but notarized, not in substantial
compliance. In re Will of Ferree.
 1 of 2 witnesses: A will signed by only one witness when two are required can never be in substantial
compliance with the state’s Wills Act. Smith v. Smith.
 Criticism: Professor Langbein—the father of substantial compliance in the U.S.—later criticized it. After
studying its use in Australia, Langbein concluded that the courts put too much emphasis on the second prong of
the test (clear and convincing evidence shows that the will substantially complies with the statutory Wills Act
formalities). He advocated for harmless error instead.
3. Harmless Error A. Harmless Error/Dispensing Power: Under the harmless error approach, if a will is not executed in strict compliance
*preferred with the jurisdiction’s Wills Act formalities, the court is empowered to probate the will if clear and convincing evidence
shows that the decedent intended the document to constitute his or her last will and testament. UPC 2-503 (1997).
 {McL} UPC § 2-503. Harmless Error. Although a document or writing added upon a document was not
executed in compliance with Section 2-502, the document or writing is treated as if it had been executed in
compliance with that section if the proponent of the document or writing establishes by clear and convincing
evidence that the decedent intended the document or writing to constitute:
o (1) the decedent’s will,
o (2) a partial or complete revocation of the will,
o (3) an addition to or an alteration of the will, or
o (4) a partial or complete revival of his [or her] formerly revoked will or of a formerly revoked portion
of the will.
 Comment: By way of dispensing power, this new section allows the probate court to excuse a harmless error in
complying with the formal requirements for executing or revoking a will . . . .
B. Scope: The harmless error doctrine authorizes a court to “dispense” with those Wills Act formalities that it deems
appropriate as long as there is clear and convincing evidence the decedent intended the document to be her will. Many
have argued that (1) the witness requirement is the least important (and thus easiest to dispense with) and (2) the writing
requirement is the most important and the one that can almost never be dispensed with.
 Attestation: Harmless error rule may be invoked to excuse a defect in attestation.
o If two individuals do not properly witness the document, [UPC 2-503] provides that the document may
still be treated as if it had been executed if the proponent of the document establishes by clear and
convincing evidence that the decedent intended the document to be the decedent’s will. In re Estate of
Hall.
o Attestation is the requirement most easily dispensed with because it makes only a modest contribution
to the purposes of the will formalities.
 Writing: The lack of writing cannot be excused as harmless. RS3d Prop. The UPC does not allow for dispensing
the writing requirement (because it applies to a document or writing)
 Signature: Among the defects in execution that can be excused, the lack of a signature is the hardest to excuse.
This is because it raises doubt as to authenticity and finality.
o Two circumstances: Dropping dead in the act of signing; Switched wills case
C. Intent: For the harmless error doctrine to apply, a party must show by clear and convincing evidence that the decedent
intended this particular document to be her final will, even if there is clear and convincing evidence that the decedent
intended a certain disposition (i.e., to include a certain person in her estate plan). Macool.
 Review & Assent: For an unsigned writing to be admitted into probate as a will under the harmless error rule,
the proponent of the writing must prove, by clear and convincing evidence, that: (1) the decedent actually
reviewed the document in question; and (2) thereafter gave his or her final assent to it. Macool. {McL}
D. Switched Wills Case: When a husband and wife sign each other’s wills on accident, the signature and attestation
requirement can be dispensed under the harmless error rule. This is because there is clear and convincing evidence that the
husband and wife intended to sign the property will.

22/87
B. Revocation of Wills
A. Ambulatory: A will is ambulatory, subject to modification or revocation (in whole or in part) by the testator at any time prior to death.
B. Revocability of wills: Wills are executed inter vivos but are not effective until death. {CT} If the testator changes her mind after executing a
will, the testator can revoke it, replace it, or amend it at any time. A will can be revoked (1) by act, (2) by writing, (3) by common law presumption
of physical act revocation, or (4) by operation of law. {CT}
1. Revocation by A. Revocation by Writing or by Physical Act: All states permit revocation of a will (1) by a subsequent writing
Writing or by Physical executed with Wills Act formalities, and (2) by a physical act such as destroying, obliterating, or burning the will.
Act  No Revocation: If a duly executed will is not revoked in accordance with the applicable statute, the will must
be admitted to probate.
B. Oral Declaration: An oral declaration that a will is revoked, without more, is not enough to revoke the will.
C. {McL} UPC § 2-507. Revocation by Writing or by Act. (a) A will or any part thereof is revoked:
 (1) by executing a subsequent will5 that revokes the previous will or part expressly or by inconsistency; or
 (2) by performing a revocatory act on the will,6 if the testator performed the act with the intent and for the
purpose of revoking the will or part or if another individual performed the act in the testator’s conscious
presence and by the testator’s direction. For purposes of this paragraph, “revocatory act on the will” includes
burning, tearing, canceling, obliterating, or destroying the will or any part of it. A burning, tearing, or
canceling is a “revocatory act on the will,” whether or not the burn, tear, or cancellation touched any of the
words on the will.
a. Revocation by A. Express Revocation: A will be may revoked by a subsequent writing expressing the intent to revoke, but only if the
Writing subsequent writing qualifies as a valid will. {CT} The subsequent will must be executed with Wills Act formalities and
qualify as either an attested will or a holographic will. UPC § 2-507(a)(1). A subsequent will can revoke a prior will
expressly or implicitly by inconsistency.
1. Express Revocation: Express revocation is clear when there is a clear and express statement of the intent to revoke
the prior will. A properly executed instrument that does no more than express the intent to revoke a prior will is a valid
will.
 Express-Revocation Clause—Example: “I, Wilt Chamberlin, hereby make my will, and I revoke all other
wills and codicils that I have previously made.” {McL}
B. Revocation by Inconsistency (Implied Revocation): Revocation by inconsistency occurs when the subsequent will
disposes of the decedent’s property in a way that is inconsistent with the prior will. Because the later will controls over
the prior will, the prior will is deemed revoked to the extent of any inconsistencies. The question is whether the testator
intended the subsequent will to replace a prior will in whole or in part, or if instead he intended the subsequent will to
supplement the prior will.
 Modern View/UPC: A subsequent “will” that does not expressly revoke a prior will, but makes a complete
disposition of the testator’s estate is presumed to replace the prior will and revoke it by inconsistency. UPC §
2-507(c). {McL} A subsequent “will” that does not make a complete disposition of the testator’s estate is not
presumed to revoke the prior will and is instead viewed as a codicil, and it supersedes the earlier will to the
extent of any inconsistency. UPC § 2-507(d). {McL} Any property not disposed of under the codicil is
disposed of in accordance with the prior will. § 2-507(d). {McL}

5 UPC § 1-201(57) defines a “will” to include a codicil and any testamentary instrument that merely appoints an executor or revokes or
revises another will. In a state that recognizes holographic wills, a holograph can revoke an attested will—a principle that is implicit in Estate
of Kuralt. {McL}
6 Words of cancellation must be on the will—not on a folder in which the will is placed, or on the front or back cover of the will.
23/87
C. Wills vs. Codicil: A testamentary instrument (i.e., a will) that supplements, rather than replaces, an earlier will is
called a codicil; the later codicil supersedes the earlier will to the extent of inconsistency between them. The prior will
still stands and is valid to the extent it is not revoked by codicil.
a. Codicils—Execution: A codicil is a will and must be executed in accordance with the Wills Act formalities.
 Exception—Codicils to Holographic Wills: Handwritten amendments (interlineations) to a holographic will
constitute a valid holographic codicil, even if the interlineations do not qualify as a valid holographic will in
their own write. {CT}
b. Mixed Wills and Codicils: Holographic codicils to attested wills are valid, and attested codicils to holographic wills
are valid.
c. Revocation of a Codicil/Will: Revocation of a codicil does not revoke the will. RS3d Prop. § 4.1. cmt. m.
Revocation of a will revokes all of its codicils unless it is clear that the T intended the codicils to operate independently
of the will. {McL}
D. Writing as Revocation by Act—Cancellation
 Common Law: The act of writing can qualify as revocation by act. The written words must be so placed as
to physically affect the written portion of the will, not merely on blank parts of the paper on which the will is
written. Thompson.
 UPC: “A will or any part thereof is revoked: . . . (2) by performing a revocatory act on the will, 7 if the testator
performed the act with the intent and for the purpose of revoking the will . . . . ‘[R]evocatory act on the will’
includes . . . canceling . . . the will or any part of it. A . . . canceling is a ‘revocatory act on the will,’ whether
or not the . . . cancellation touched any of the words on the will.” UPC § 2-507(2).burning, tearing, or
canceling is a ‘revocatory act on the will,’ whether or not the burn, tear, or cancellation touched any of the
words on the will. UPC § 2-507(2).
 Example—”VOID”: Writing “void” across the will does not qualify as a valid revocation by writing because
it does not qualify as a valid will (it is not signed). But the act of writing “VOID” across the will does qualify
as a destructive act. {CT}
a. Harmless Error: Even if a writing does not qualify as an act of cancellation under UPC § 2-507(a)(2), it may
qualify as a revocation by subsequent writing under the harmless error rule of § 2-503 (the court may dispense with one
of formalities). The words of cancellation must be “on the will.” Otherwise, the words must qualify under UPC § 2-
507(a)(1).
 Handwritten, non-holographic (attested) wills are not excluded from the harmless error doctrine. Stoker.
b. Revocation by A. Revocation by Act: To effect revocation of a duly executed will, in any of the methods prescribed by statute, two
Act things are necessary: (1) The doing of one of the acts specified (i.e., burning, tearing, cancelling, obliterating), (2)
accompanied by the intent to revoke—the animo revocandi. Proof of either, without proof of the other, is insufficient.
Thompson.
B. Revocatory Acts
 Words of Cancellation: “Void”; “Revoked”
 Common Law—Destructive Act Must Affect Writing: Revocation of a will by cancellation within the
meaning of the statute, contemplates marks or lines across the written parts of the instrument, or a physical
defacement, or some mutilation of the writing itself, with the intent to revoke. If written words are used for
the purpose, they must be so placed as to physically affect the written portion of the will, not merely on blank
parts of the paper on which the will is written. Thompson.
o Essential Component: The revocatory act must destroy an essential component of the will (i.e.,
testator’s signature)
 UPC—Destructive Act Must Affect Some Part of Will: The last sentence of UPC § 2-507 allows for
revocation by a physical act of cancellation whether or not the cancellation touches any of the words of the
will.
o Harmless Error: Even if a writing does not qualify as an act of cancellation under § 2-507(a)(2), it
may qualify as a revocation by subsequent writing under the harmless error rule of § 2-503 (the
court may dispense with one of formalities). The words of cancellation must be “on the will.”
Otherwise, the words must qualify under § 2-507(a)(1).
o Handwritten, non-holographic (attested) wills are not excluded from the harmless error doctrine.
Stoker.
C. Revocation by Another: The act may be performed by another if performed in the testator’s presence and at the
testator’s direction. UPC § 2-507(a)(2).
D. Botched Revocation: Argue for constructive trust—if there was clear and convincing evidence the testator intended
to revoke the will. {McL}
E. Destruction of Copy: The revocatory act must be performed on the will. Performing the act on another document or
on an unexecuted copy of the will is insufficient. If, however, the act was performed on the wrong document due to
wrongdoing or mistake, a constructive trust may be imposed on the devisees of the will in favor of the persons who
would have taken had the will been revoked. Alternatively, if the intent to revoke is proved by clear and convincing
evidence, the failure to perform the act on the will, accompanied by performance of the act on a copy that the testator
mistakes for the will, may be excused as a harmless error. RS3d Prop. 4.1 cmt. f.
F. Express Revocation is Much Better: I __________ hereby make my will, and I revoke all other wills and codicils
that I have previously made.

7 Words of cancellation must be on the will—not on a folder in which the will is placed, or on the front or back cover of the will.
24/87
3. Harmless Error and Revocation by Physical Act: The Harmless Error Doctrine does not apply to revocation by
physical act under the UPC and State Legislatures. The Restatement is in favor of it. {McL}
G. Partial Revocations by Physical Act: The jurisdictions are split over whether partial revocation by physical act is
permitted (i.e., interlineations, strike-outs, obliterations).
 Whole Revocation vs. Partial Revocation: “Mutilate” means something less than total destruction. Mere
mutilation of a will would not, of itself, take from a will all legal force. A mutilation, however, which takes
from the instrument an element essential to its validity, would have the effect to revoke it. Funk.
o UPC: “intent and for the purpose of revoking . . . part thereof.” UPC 2-507.
 Presumption that Decedent Performed the Act: Because decedent was in possession of the will, there is a
presumption that decedent performed the act with the intent and for the purpose of revocation. Corroborating
evidence. In re Byrne’s Will.
 Erasures in a Holographic Will: So we say that erasures in a holographic will, made by the hand of testator
himself, is legal revocation of such portions as are so erased, since it is in the manner required for a will of
that kind to be executed; and new portions written into such will by the same hand, to take the place of
erasures, or new portions otherwise written therein by the same hand, being thereby executed in the manner
which justifies the validity of a holographic will originally, so long as the signature of the testator remains in
such manner as to make it manifest that it is intended as a signature, do not in any sense invalidate the will or
affect its finality, since it is then a complete new holographic will, and needs no re-execution or republication
before witnesses, because it did not originally demand execution and publication before them. La Rue v. Lee.
See also Schumacher.
 Examples: Horst v. Horst; Hansel v. Head
1. Traditional (Minority) Approach: Partial revocations by physical act are not allowed. They instead ignore the act in
question and give effect to the will as originally written. {McL}
 Rationale: (1) Greater potential for fraud; (2) Effects new gifts without will execution formalities
2. Modern (Majority/UPC § 2-507) Approach: Partial revocations by physical act are allowed. {McL} They probate
the will with the marks, ignoring the language that has been obliterated.
 Rationale: (1) carry out testator’s intent; (2) Potential for fraud is no greater than that for wholly revoking a
will; (3) Wholly revoking a will also effects new gifts without will execution.
 How to Treat the Revoked Gift: (1) Majority: The revoked gift may fall to the residuary (but nowhere else);
(2) Minority: The revoked gift may pass through intestacy only; and (3) UPC: The will is given effect as it
reads with the partial revocation by act regardless of where that means the gift goes. {CT}
c. Revocation by 1. Lost Wills: (1) If the will is in the T’s possession/control before death and (2) the will is not found among T’s
Presumption of personal effects at death a presumption arises that the T destroyed the will with the intent to revoke it. {McL} See
Physical Act Harrison; Estate of Schumacher. If the presumption is not overcome, the will is deemed revoked. If the presumption is
Revocation rebutted, the will is deemed “lost,” and extrinsic evidence is admitted to prove its terms. The contents of a lost but
unrevoked will can be proved by a photocopy, a digital copy, the drafter’s notes or recollections, or by other clear and
convincing evidence.
 Standard of Proof for Overcoming Presumption of Revocation: Split: Older cases required clear and
convincing evidence. The more recent trend (and the Restatement) is to require only a preponderance of the
evidence. If the presumption is rebutted, the will is entitled to probate if its contents can be proved.
 A will that is not properly revoked is still valid even if it is lost (i.e., the person who had custody of it cannot
find it).
2. Duplicate Originals: Duplicate originals are multiple originals of the same will, each one properly executed. Often
the attorney keeps one duplicate original, and the testator takes the other duplicate original home with her.
 i. Option 1—Revokes all Duplicate Originals: If the presumption doctrine applies to one duplicate original,
it applies to all duplicate originals. See Harrison.
 ii. Option 2—Does Not Revoke All Duplicate Originals: The presumption doctrine does not revoke the
duplicate originals unless none of the duplicate originals are found following the testator’s death. {CT}
Thompson v. Royall (VA): Mrs. Kroll (the testatrix) had her attorney take out her will and codicil and revoke them. To do so, on the back of the
manuscript cover to the will, the attorney handwrote, in the presence of the testatrix and another, the following notation: “This will null and
void.” Mrs. Kroll then signed the notation. Mrs. Kroll died three
weeks later. The will and codicil were offered for probate. The court
held that the writing did not qualify as revocation by writing because
the notification did not qualify as a valid will (not attested because
not witnessed; not holographic because the material provisions were
not in Mrs. Kroll’s handwriting). The court held that the writing did
not qualify as revocation by act (cancellation) because the
handwriting did not touch any portions of the will.
In re Estate of Stoker: Stoker (the decedent) created a pour-over
will and trust in 1977 that benefited his then-girlfriend. They broke
up. Stoker urinated on, then burned copies of the 1997 will and trust.
In 2005, Stoker had a friend handwrite a document that said, “I,
Steve Stoker revoke my 1997 trust … [Ex-girlfriend] to get nothing.
Everything is to go to my kids … [who] are to have power of attorney
over everything I own.” The document failed as an attested will
because it was not signed by witnesses and failed as a holographic

25/87
will because it was not in Stoker’s handwriting. The court held that there was clear and convincing evidence, under California’s harmless error
doctrine, that Stoker intended the 2005 document to constitute his will because it provided that all his property would go to his children, that the
1997 trust was revoked, that his ex-girlfriend was to receive nothing, and that his children were to have power of attorney over everything.
Extrinsic evidence supported that conclusion: Stoker’s friend testified that Stoker told her the document was “my last will and testament.”
Stoker revoked the 1997 trust because the 2005 will expressly and unequivocally provided that the 1997 trust was revoked and that Stoker’s
children were to receive everything. The 1997 will was revoked by inconsistency.
Harrison v. Bird: Ms. Speer (the testatrix) executed duplicate wills (two identical original wills), leaving one with her attorney and taking the
other home with her. Thereafter, the testatrix called the attorney and advised him that she wanted to revoke her will. The attorney tore the will
into pieces in the presence of his secretary and mailed the pieces to the testatrix. The court held that the attorney’s act of tearing one of the
duplicate originals into pieces was not a valid revocation by act because it was not done in the presence of Ms. Speer. However, the court ruled
that, because the pieces of the will that were mailed to Ms. Speer were not found after her death, there was a presumption that Ms. Speer
destroyed her will and thus revoked it. Because the challenger did not present sufficient evidence to rebut the presumption that Ms. Speer
destroyed her will with the intent to revoke it, the revocation was valid and Ms. Speer died intestate. (If the pieces had been found in Daisy’s
possession it would have still been valid under UPC § 2-507 because there was no subsequent will and there was not revocatory act performed
by Daisy or in her conscious presence.)
 What if the pieces had been find in file marked revoke will?
o Revocatory act must be “on the will”—not in a folder with “revoked will” written on it
o No common law presumption—will was found in her possession.
o UPC § 2-503: There was a writing but still no signature or attestation. Doesn’t work.
In re Byrne’s Will: Upon Byrne’s (the testatrix) death, her will was removed from her safety deposit box. The will had lines drawn through the
names of Byrne’s brothers, wherever their names appeared in the will, and through provisions relating to the brothers. A letter was also attached
to the codicil addressed to the executor stating that the executor and Byrne’s three sisters were to divide her estate equally, while making no
bequest to the two brothers. The Court held that the marks clearly indicated Byrne’s intended to strike out those provisions of her will which
made provision to her brothers. It also was found in her safety deposit box—presumption that the testator performed the revocatory act with
intent to do so—and the letter corroborated the revocatory act.
In re Estate of Funk: After Funk (the decedent) executed her will, she handwrote numerous notes on the typed document including changes in
beneficiaries and certain bequests. She drew lines through her bequests to FUPC (a church) and Purdue University. In addition, page four was
missing and a portion of page five had been cut out and then reattached by stapling. The Court held that Funk did not revoke her last will and
testament by mutilation before her death because Funk did not subtract from her will any essential part. Instead, her numerous marginal notes
and handwritten changes to the will, at most, indicated an intent to change her will.
Horst v. Horst (OH): One copy of Mary Horst’s (testator’s) will had substantial markings in the margins and within the body of the text on the
first page. There was an “X” over about ten lines of text on that page, and the “X” itself was marked out. The words “the amount of $500” were
blackened in ITEM II. The words “This Will is correct” was legible at the top of the page. Other writings, and multiple signatures by Mary were
also legible between and around the typewritten lines. On the second page, there were no markings surrounding or within the text, but there is
marked-out writing surrounding Mary’s final signature. The Court held that testator’s conduct was insufficient to revoke the entire will because
she left almost all of the text on the first page visible, and left the second page, including the date and her signature, entirely intact. She also left
one copy of the will completely unmarked. At most, Mary’s markings could indicate an intention to change portions of the will but not to revoke
the will entirely—the words “This Will is correct” supports its validity.
Hansel v. Head: Earl and Ernestine Head prepared a valid joint (handwritten, attested) will. The will offered for probate read in pertinent part:
“Also, that all insurance, checking account, Savings account, and all mortgage balances due us, be made payable to Glinda
Summerlin Autery, and that after after [sic] all funeral expenses, and our current debts are paid in full, that one half, of the
balance of our cash assets be divided equally between, Jerry Clinton Summerlin, Donald Harold Summerlin, and James
Albert Poole, our nephews. The remaining one half to be divided equally between the children of, Daniel T. Head, Sr.,
“VOID Earl L. Head Ernestine S. Head Birdie Ruth Schoening, Glinda Summerlin Autery, and Joseph Byron Summerlin.
“EXCEPTIONS TO ABOVE:
Any Monies, Stocks & etc. in the name of Ernestine S. Head only to be divided equally between the following: Earl L.
Head, Joe B. Summerlin, Glinda S. Autery, Donald H. Summerlin & Jerry C. Summerlin.”
The “EXCEPTIONS” clause was written in a different ink from that used in the main part of the will, indicating it was written later on. The term
“VOID” was printed in the margin and was also printed in a different ink. Correcting fluid had been used to cover an undecipherable name, and
the Heads’ names had been written on top of the correcting fluid. The court held that the “EXCEPTIONS” clause was a nullity because it was
unwitnessed. The court also held that the word “VOID” indicated that the revocatory intent was directed only at one name, rather than at the
entire will. Therefore, the attempted partial revocation did not effect a revocation on the entire will. The court held the will should be admitted
to probate and the share allocable to the person whose name was obliterated should be disposed of through intestacy.
La Rue v. Lee: Griffith (decedent) handwrote a will with a pencil. There were erasures on the will. There was a separate writing, signed by the
testator but not dated, which stated: “I may chang my will.” It then proceeds to set forth what change might be made, ending with these words:
“If I dont make theas changes my will must stand as I hav it now Roat.” The court upheld the will and the changes made to it because the
writing showed testamentary intent.
Estate of Schumacher: Schumacher (decedent) executed a holographic will, which contained a clause devising share of Meyers Land & Cattle
stock to Schumacher’s cousins. Later, Schumacher met with his attorney to create a typed will. His attorney’s copy of the will included lines
crossing out the names of two of the cousins as remainder devisees of all the shares of the stock. The attorney testified Schumacher told him to
cross out the cousins’ names. The attorney drafted a typed will pursuant to his instructions and transmitted it to Schumacher. Schumacher died
without ever executing the will. The original holographic will (with lines crossing out the names of the cousins) was found at his secretary’s
house and was tendered to the probate court. On appeal, the court held that it could reasonably infer that the will was in decedent’s possession at
the time of his death, even though it was at his secretary’s house because Schumacher gave the box of his behalf to organize and keep on his
behalf. The court also held that there was evidence to prove that decedent made the cross-outs with the intent and purpose to revoke the devise
of stock to the cousins because of decedent’s statements to his attorney.

26/87
2. Dependent Relative A. Doctrine of Dependent Relative Revocation (DRR): When a T revokes a Will, in whole or in part, upon the
Revocation condition (or assumption) that another disposition will be valid, and the other disposition fails, the revocation is
canceled if probating the revoked will or part would carry out the T’s intent more closely than letting the property pass
by intestacy. {McL}
 *Doctrine is applied to carry out T’s presumed intent* {McL}
 Common Scenario: A testator who destroys a prior will under a belief that a new will is valid but, in fact, for
some reason unknown to the testator, the new will is invalid.
 Classic Example: LaCroix v. Senecal.
 *can apply to revive just one provision in an earlier will, or to revive a will in whole.
1. Revocation by Act: When the testator revokes a gift by act (valid revocation) on the belief that a new will or codicil
is valid, but the new will or codicil is not valid (mistake of law), the original will stands if that is what the testator
would have wanted if she had known the new will or codicil was invalid. {CT}
 DRR in Jurisdiction in which Partial Revocation by Physical Act Not Allowed: In such jurisdiction,
partial revocation can only occur by subsequent writing—either by holograph or by attested document. The
next question is whether the addition/modification to the original will is a valid codicil—either by holograph,
or by attested document.
o How to Wholly Revoke Will: Cross out T’s signature or witness’s signature or writing “VOID”
across the whole thing.
 DRR in Partial Revocation by Physical Act Jurisdiction:
o Valid partial revocation by physical act under the jurisdiction?
o Valid addition/modification to the original will—i.e., valid holographic or attested codicil?
o Carpenter (doesn’t really apply)
o UPC § 2-507
o Ruel
o Houghten
2. Revocation by Writing: A part of a will can be revoked by markings added to the original instrument following its
execution as long as such addition its itself duly executed. In such case, the obliteration is a valid partial revocation of
the original will, and DDR does not apply. In re Will of Litwack.
 A valid partial revocation by subsequent writing must follow the will formalities.
 The new addition by subsequent writing (a codicil) must also follow the will formalities.
3. Holographic Wills
 DRR in 2nd Generation Holographic Will State: The material provisions must be in the testator’s
handwriting, not in print. Estate of Phifer. If the material provisions are not, the will is invalid and the
original will is admitted to probate. Estate of Phifer.
 DRR in 3rd Generation Holographic Will State: The material portions must be in the testator’s
handwriting, not in print. Phifer would not have been valid here either because the material portions are not
included.
4. A Few Scenarios
 Same Disposition in Subsequent Codicil: When a testator repeats the same dispositive plan in a new will,
revocation of the old one by the new is deemed inseparably related to and dependent upon the legal
effectiveness of the new. LaCroix.
 Substitution of a Larger Bequest for a Smaller Bequest: The inquiry should always be: “What would the
testator have desired had he been informed of the true situation,” that is to say, what would he have wished
his legatees to have, had he known that the attempted alteration could not be given effect and that they could
take only either the amount which he originally gave them or nothing. On this question evidence of the
testator’s pertinent oral declarations and of the circumstances under which the alteration was made is
admissible. Ruel.
o In Ruel, the only evidence the court had before it was mathematics (Reduction from $500 to $100).
However, if the reduction is only 50 percent, you need more extrinsic evidence because you can’t
tell from the math.
 In re Houghten’s Estate (in a state where partial revocation by physical act is permitted).
 DRR Does Not Apply—Intestate Succession Would Bring About Result of Altered Will: Where (invalid)
Will 2 indicates that the testator wants a beneficiary to have certain property and intestacy would accomplish
the same result, DRR does not apply. Estate of Lyles (in a state where partial revocation is permitted).

27/87
LaCroix v. Senecal (CT): Dupre (testator) left a will dated March 26, 1951,
and a codicil thereto dated April 10, 1951. Item five of the will read, in
pertinent part as follows: “All the rest, residue and remainder of my
property . . . I give, devise, and bequeath . . . one-half to Aurea Senecal.” The
codicil revoked Item 5 but still devised one-half of the residue to Aurea. The
end of the codicil confirmed the will “in all respects except as altered by this
Codicil.” However, the codicil was witnessed by Aurea’s husband (interested,
necessary witness—prohibited by purging statute) (no residue of the residue
clause applied). The Court held that the devise to Aurea under Item 5 of the
original will stood under the doctrine of dependent relative revocation
because it was clear that her intention to revoke the will was conditioned
upon the execution of a codicil which would be effective to continue the
same disposition of her residuary estate. Because the codicil was void, the
conditional intention of Dupre to revoke the will was rendered inoperative, and the gift to Aurea under the will continued in effect. (no residue
of residue applied)
Estate of Phifer (CA—State that Recognizes Holographic Wills): Phifer (decedent) made holographic additions, deletions and interlineations
on several copies of his formal, witnessed will and initialed them. He initialed the changes. He sent the copies out to the legatees and devisees
named in the formal will. He also composed a handwritten letter explaining and clarifying the construction of certain provisions of the formal
will and the changes he made. A copy of the will with the holographic changes was admitted for probate. On appeal, the court held that the
changes were a nullity, despite Phifer’s clear intent. The court held that the handwriting was wholly unintelligible, devoid of testamentary intent,
and without substance, except insofar as it was explained by incorporating the typewritten provisions of the formal will to which it related, and
so was not a valid holographic will under California law. The court held that decedent’s notation that the will was “amended by handwritten
initial” was not a valid holographic codicil because it only incorporated the handwritten material which derived meaning only from its relation
to the formal will. The court held that the decedent’s letter evidenced testamentary intent, if at all, solely to the extent it was conjoined with the
foregoing invalid attempts at modification of the formal will and should not have been admitted to probate.
In re Will of Litwack (NY): Litwack (testatrix) executed a formal, attested will devising her apartment to her son. After the will was executed,
she obliterated the paragraph, handwrote “delete 2/05/03,” and signed next to it. The obliteration was witnessed by two friends who duly signed
their names in her presence. The court held that the obliteration was a valid partial revocation of the original will because it was executed with
the formalities required for the execution of a will.
Carpenter v. Wynn (KY): Carpenter (the testator) executed his last will and testament and signed his name in the presence of two attesting
witnesses. The will as originally drawn devised to the two sons $ 500 each. A few days after this will was drawn, Carpenter summoned the
draftsman, who was also an attesting witness, and informed him that he desired to raise the specific bequests he made to his two sons from $
500 to $ 700. In the presence of this attesting witness the testator struck out the word “five” in the two bequests and substituted the word
“seven” in lieu thereof. Carpenter did not re-sign the will. The altered will was submitted for probate. On appeal, the court applied the DDR and
held that the ENTIRE original will was not revoked by the testator because of the alteration that raised the amount of the specific bequest: It was
clear that Carpenter intended his sons to have $500 each at all events, and that he did not mean that, if they could not receive $700, the old will
was to be invalid.
Ruel v. Hardy (NH): A codicil to Smith’s (testatrix’s will) recited that she had been appointed the executrix of the estate of Almina A.
Dinsmore and provided: “Should there be any money left me from said . . . Estate I desire it to be used as follows: . . . Third: To Frank W.
Persons, and to Walter E. Parsons and Mrs. Lilla A. Smith all of Salisbury Heights, N.H., I give five hundred dollars to each one.” However, at
some point the word “five” had been crossed out and the word “one” had been written under it. The court held that the word “one” was not part
of the codicil because there was no proof of its attestation. The court then applied the doctrine of dependent relative revocation and held that
handwritten changes in the will codicil left Frank, Walter, and Lilla with nothing under the will because the reduction of 80% of the legacy
tended to show a preference on the testatrix’s part that the legatees should have nothing rather than the full sum (there was no other evidence).
In re Houghten’s Estate (MI): Houghton (decedent) executed an attested will. When his will was found after his death, it was discovered that
Houghton had struck the clause, which bequeathed the residue of his estate to his brother-in-law and made specific bequests to two other
persons. The changes indicated that his brother-in-law was to receive nothing. On appeal, the court held that the will as altered could not be
probated because Houghton had failed to comply with the statutory requirements. The court did not doubt that decedent intended to revoke the
devise and bequest of the residue to his brother-in-law. As such, because the means employed by decedent to revoke his will satisfied the statute,
the doctrine of dependent relative revocation did not apply. However, decedent’s attempt to devise certain property to others failed because he
did not properly execute a new will. Thus, his estate had to be administered as if he were intestate because his original will was completely
revoked.
Estate of Lyles (CA): Lyles (the testator) died, leaving her niece as executor. Lyles’ will, as originally executed, bequeathed 40 acres to Howell
and 100 acres to her niece. When the will was found in the home of Lyles, after her death, the devise to the Howell had been scribbled out. The
figure 100 had been marked out and changed to 140. On appeal, the court found that the attempt to make the change was sufficient to work a
partial revocation because the doctrine of dependent relative revocation was not applicable under the circumstances. Where the markings
indicated that the testator wanted her niece to have all her land and intestacy would accomplish the same result, the doctrine was not relevant:
While DDR presumed that a testator would prefer the revival of the original disposition over intestacy, here Lyles would have chosen intestacy
over revival.
3. Revival of Revoked A. Revival: The question of revival typically arises under the following facts: Testator executes will 1. Subsequently,
Wills testator executes will 2, which revokes will 1 by an express clause or by inconsistency. Later, testator revokes will 2. If
the doctrine of revival applies, the previously revoked will 1 is valid without having to be re-executed. {McL}
1. Minority Rule: If Will 1 is revoked by Will 2, Will 1 cannot be revived unless it is re-executed or republished. See
Alburn. {McL}
2. Majority (UPC) Rule: A majority of states holds that upon revocation of will 2, will 1 is revived if the testator so
intends. {McL} If all the testator has to do revive Will1 is to intend to revive Will 1 when he or she revokes Will 2, the

28/87
key is how the testator revoke Will 2. If the testator revoked Will 2 by act, the courts will take any evidence of the
testator’s intent to revive Will 1 (presuming no revival of Will 1 where Will 2 is revoked by act; presuming revival
where Codicil is revoked by act). If, however, the testator revoked Will 2 by Will 3, the intent to revive Will 1 must be
set forth in Will 3.
3. UPC § 2-509 (1990). Revival of Revoked Will: Duke, at 238.
 (a) [Revocation of Will 2 by Act: Presumption Against Revival of Will 1] If a subsequent will (will 2) that
wholly revoked a previous will (will 1) is thereafter revoked by a revocatory act under Section 2-507(a)(2),
the previous will (will 1) remains revoked unless it is revived. The previous will is revived if it is evident
from the circumstances of the revocation of the subsequent will or from the testator’s contemporary or
subsequent declarations that the testator intended the previous will to take effect as executed.
 (b) [Revocation of Codicil by Physical Act: Presumption in Favor of Revival] If a subsequent will
(codicil) that partly revoked a previous will (will 1) 8 is thereafter revoked by a revocatory act under Section
2-507(a)(2), a revoked part of the previous will (will 1) is revived unless it is evident from the circumstances
of the revocation of the subsequent will (codicil) or from the testator’s contemporary or subsequent
declarations that the testator did not intend the revoked part to take effect as executed.
 (c) [Revocation of Will 2/codicil by Will 3: No Presumptions] If a subsequent will (will 2/codicil) that
revoked a previous will (will 1) in whole or in part is thereafter revoked by another, later will (will 3/codicil)
the previous will (will 1) remains revoked in whole or in part, unless it or its revoked part is revived. The
previous will (will 1) or its revoked part is revived to the extent it appears from the terms of the later will
(will 3/codicil) that the testator intended the previous will (will 1) to take effect.
B. CHECK FOR DDR ISSUE AS WELL
In re Estate of Alburn (WI): The trial court had found that Alburn (the decedent)
had first executed the Milwaukee will. Later she executed the Kankakee will.
Before her death she tore up the Kankakee will, mistakenly believing that by doing
so she would revive the Milwaukee will. On appeal, the court held that there was
uncontroverted testimony from Alburn’s sister-in-law that Alburn had told her she
intended the Milwaukee will to stand, that Alburn had indicated that she did not
wish to die intestate, that she took no steps after destroying the Kankakee will to
make another will, and that both wills had similar provisions not providing for
next-of-kin. On this basis it was not against the great weight and preponderance of
the evidence that Alburn destroyed the Kankakee will in the mistaken belief that
she was reviving the Milwaukee will. Thus, the doctrine of dependent relative
revocation applied to support the probate of the Kankakee will.
4. Revocation by
Operation of Law
a. Divorce A. Revocation by Operation of Law—Divorce:
 Majority: In all but a few states, statutes provide that a divorce revokes any provision in decedent’s will for
the decedent’s divorced spouse.
o Issues: (1) Does the statute apply to nonprobate assets as well as probate assets? (2) Does the
statute revoke bequests to relatives of the ex-spouse in addition to revoking bequests to the ex-
spouse. {McL}
 Minority: In the remaining states, revocation occurs only if the divorce is accompanied by a property
settlement.
 UPC § 2-804 (1990, rev. 1997):
 . . . (b) [Revocation Upon Divorce.] Except as provided by the express terms of a governing instrument,9 a
court order, or a contract relating to the division of the marital estate made between the divorced individuals
before or after the marriage, divorce, or annulment, the divorce or annulment of a marriage:
o (1) revokes any revocable
 (A) disposition or appointment of property made by a divorced individual to his [or her]
former spouse in a governing instrument and any disposition or appointment created by
law or in a governing instrument to a relative of the divorced individual’s former spouse,
 (B) provision in a governing instrument conferring a general or nongeneral power of
appointment on the divorced individual’s former spouse or on a relative of the divorced
individual’s former spouse, and
 (C) nomination in a governing instrument, nominating a divorced individual’s former
spouse or a relative of the divorced individual’s former spouse to serve in any fiduciary
or representative capacity, including a personal representative, executor, trustee,
conservator, agent, or guardian; and
o (2) severs the interests of the former spouses in property held by them at the time of the divorce or
annulment as joint tenants with the right of survivorship [or as community property with the
right of survivorship], transforming the interests of the former spouses into equal tenancies in
common . . . .

8 Codicil
9 “governing instrument”: a deed, will, trust, insurance, or annuity policy, account with POD designation, pension plan, or other such
nonprobate transfer.
29/87
 (d) [Effect of Revocation.] Provisions of a governing instrument are given effect as if the former spouse and
relatives of the former spouse disclaimed all provisions revoked by this section or, in the case of a revoked
nomination in a fiduciary or representative capacity, as if the former spouse and relatives of the former
spouse died immediately before the divorce or annulment . . . .
 (f) [No Revocation for Other Change of Circumstances.] No change of circumstances other than as described
in this section and in Section 2-803 effects a revocation.
 Applies to nonprobate transfers as well as to wills.
B. Beneficiaries Affected
 Example 1—”Relating to the Spouse” Statute (Factual Inquiry): If the devise was primarily motivated by
the marriage, it “relates to” the marriage, and so is revoked by the statute. If the devise was motivated by
some other reason (i.e., close personal relationship), the devise is not revoked by statute. This is a fact-based
inquiry to determine the decedent’s intent. The court may look to the four corners of the will, extrinsic
evidence, the situation of the testator and his relations with the parties, and other surrounding circumstances.
Friedman.
 Example 2—UPC: The Uniform Probate Code voids any revocable disposition, or appointment, created by
law, or in a governing instrument “to a relative of the divorced individual’s former spouse.” It revokes the
interest of such relative as a matter of law. Estate of Marchwick.
 Drafting: If a testator wants to leave assets to a family member of a divorced spouse, include express
language: “Notwithstanding any divorce revocation statute . . .”
C. Scope: The UPC approach is to apply the revocation by operation of law not only to wills, but also to will
substitutes—life insurance, joint tenancy, pension plans, and other nonprobate arrangements. The majority approach is
to apply the revocation by operation of law doctrine only to wills.
b. Marriage  Traditional Rule/Minority: A premarital will is revoked upon marriage
 Modern Rule/Majority: A premarital will remains valid in spite of a subsequent marriage, but a surviving
pretermitted spouse is entitled to an intestate share of the deceased spouse’s estate, unless it appears from the
will that the omission was intentional or the pretermitted spouse is provide for in the will or by a will
substitute. This kind of statute in effect revokes the will to the extent of the pretermitted spouse’s intestate
share. if a pretermitted spouse cannot take an intestate share because one of the exceptions applies, an elective
or forced share of the decedent spouse’s estate, which is available to all surviving spouses in separate
property states whether intentionally or unintentionally disinherited, may be available.
o See UPC § 2-301 (Duke, at 563).
c. Birth of Children  Common Law Rule/Minority: Marriage followed by birth of children revokes a will executed before
marriage.
 Majority: Almost all states have pretermitted child statutes, which give a child born after the execution of a
parent’s will, and not mentioned in the will, a share in the parent’s estate. Some pretermitted child statutes
include children born before the execution of the will as well as children born thereafter. A pretermitted child
statute, if applicable, results in a revocation of the parent’s will to the extent of the share given to the child
under the statute.
o See UPC § 2-302 (Duke, at 566).
Friedman v. Hannan (MD): Hannan (the decedent) married Zelinski. The two had no children. They were divorced and entered into a property
settlement agreement. Hannan died. During the marriage, Hannan had executed a will in which he named some of Zelinksi’s family members. A
Maryland statute directs that provisions in a will “relating to the spouse” be revoked upon divorce from that person.” On appeal, the Court held
that the automatic revocation provision is not limited to bequests to a former spouse and may include bequests to a former spouse’s family
members: A court should utilize the terms of the will and circumstances surrounding its execution to determine whether a bequest “relates to the
spouse” within the meaning of the statute. The Court also held that the trial court could infer that bequests made to the wife’s family were made
primarily because of the marriage. There was no evidence of some independent reason for the bequest in the will itself or the circumstances
existing at the time of execution. Even though the provision named the wife’s family members, it was not error to find that the decedent was
group-minded in making the residuary bequest. The bequest failed as a result of the divorce. The decedent did not know the wife’s family before
marrying her, and during a large portion of the marriage he was employed at sea.
Estate of Marchwick (MT): Marchwick (decedent) executed her last will and testament, and an unfunded pour-over revocable living trust.
Marchwick’s trust expressed her intent to distribute a portion of her estate to her then husband (Burgess). Marchwich’s trust distributed the
remainder of her estate as follows: 60 percent to her daughter from a previous marriage, and the remaining 40 percent divided equally among
Marchwick’s stepchildren (Burgess children). Marchwick and Burgess divorced. Marchwick died. The court held that the UPC operated to
revoke the stepchildren’s interests as a matter of law because the stepchildren were biological children of Marchwick’s former spouse.
C. Components of a Will
 It is possible for documents other than a will to determine who takes the testator’s estate. Two doctrines that can have this effect by
permitting extrinsic evidence to resolve the identity of persons or property are (1) incorporation by reference and (2) acts of independent
significance.
1. Integration A. Doctrine of Integration: Under the doctrine of integration, all papers that are present at the time of execution and
are intended to be part of the will are treated as part of the will (integrated into the will).
 Holographic Wills: A court generally adheres strictly to statutory provisions regarding the execution,
interpretation, and probate of wills. This rule is said to be especially true in the case of holographic wills.
Thus, where the instrument offered consists of more than one sheet of paper, it must be made clearly apparent
the testator intended that together they should constitute the last will and testament of the testator. In re Estate
of Rigsby.
 Fasten All Documents Together: The lawyers can avoid litigation by ensuring the will is fastened together
30/87
before the testator signs and by having the testator sign or initial each numbered page of the will for
identification.
C. RUN THIS THROUGH THE INCORPORATION BY REFERENCE & ACTS OF INDEPENDENT
SIGNFICANCE ANALYSES.
In re Estate of Rigsby (OK): Following Rigsby’s death, two handwritten pages, dated the same date, were found folded together. The first page
clearly expressed testamentary intent, was signed at the bottom, and had approximately 2.5 inches of blank space at the bottom of the page. The
second page was a list of personal property with the name of an individual after it (some of which conflicted with the gifts of the first page). The
second page was initialed and dated but, standing alone, did not express testamentary intent; nor was it signed at the bottom. Neither page
referenced the other page. On appeal, the Court admitted the first page to probate but not the second page. The Court held that it did not clearly
appear that the second page was intended to be included with the first because (1) the first page did not make any mention of a list such as the
second offered page; and (2) the second page conflicted in part with the first page.
2. Republication by A. Doctrine of Republication by Codicil: A codicil is a will that merely amends an existing will. {CT} A will is
Codicil treated as if it were executed when its most recent codicil was executed, whether or not the codicil expressly
republishes the prior will, unless the effect of so treating it would be inconsistent with the testator’s intent. RS3d.
 Applies only to a prior validly executed will (cannot “republish by codicil” an instrument that was not
executed with required formalities) {McL}
 Publication of a Will: Occurs when a testator conveys to witnesses, by words or by action, that a document
is a testator’s will.
3. Incorporation by
Reference
a. Existing Writings A. Incorporation by Reference: Allows for a writing that was in existence but not present at the time of execution and
that was not itself executed with testamentary formalities to be absorbed into the testator’s will.
 Applies to incorporate instruments that were not executed with required formalities into a will {McL}
B. Requirements: A valid will can incorporate by reference a document that was not executed with Wills Act
formalities, thereby giving effect to the intent expressed in the incorporated document, as long as (1) the will manifests
the intent to incorporate by reference; (2) the will describes the document sufficiently to permit its identification; and
(3) the document being incorporated was in existence when the will was executed (think snapshot). UPC § 2-510
(“Any writing in existence when a will is executed may be incorporated by reference if the language of the will
manifests this intent and describes the writing sufficiently to permit its identification.”). See Clark. {McL}
C. Incorporation by Reference vs. Integration: If the document is physically present when the testator signs the will,
you can use integration to give effect to the document. If the document is in existence but not physically present when
the will is executed, you can try to use incorporation by reference.
D. RUN THIS THROUGH THE ACTS OF INDEPENDENT SIGNFICANCE & INTEGRATION ANALYSES
E. Snapshot: The courts strictly apply the requirement that the document has to be in existence at the time that the will
was executed. If the document changes over time, only the document as it existed at the time the will was executed is
incorporated by reference (unless the will is re-executed by codicil).
 Extrinsic Evidence: Courts look at extrinsic evidence to determine whether the writing was in existence at
the time.
b. Subsequent A. UPC § 2-513. Separate Writing Identifying Devise of Certain Types of Tangible Personal Property. Whether or
Writings & not the provisions relating to holographic wills apply, a will may refer to a written statement or list to dispose of
Tangible Personal items of tangible personal property not otherwise specifically disposed of by the will, other than money. To be
Property admissible under this section as evidence of the intended disposition, the writing must be signed by the testator and
must describe the items and the devisees with reasonable certainty. The writing may be referred to as one to be in
existence at the time of the testator’s death; it may be prepared before or after the execution of the will; it may be
altered by the testator after its preparation; and it may be a writing that has no significance apart from its effect on the
dispositions made by the will.
B. UPC’s Tangible Personal Property List (Slim Majority): In a break from traditional law, the UPC allows a
testator to dispose of tangible personal property by a separate writing, even if prepared after the execution of the
testator’s will, provided that the will makes reference to the separate writing. UPC § 2-513 (The Clark Court did this as
well). The UPC allows the testator to reserve the power to make and then continue revising a list of bequests of tangible
personal property without additional testamentary formalities.
 Value Limit: With the exception of a few states, there is no value limit.
 Tangible Personal Property: Capable of being handled, has physical substance. Not documents of title,
stock, cash.
 Rationale: Most people think they can write a list of how TPP should be distributed; Would require constant
changing of the will if the statute did not exist. People change their minds.

31/87
Clark v. Greenhalge (MA): Nesmith’s (testator’s) 1977 will named
Greenhalge as executor and principal beneficiary and provide that he
was to receive all her tangible personal property except for those items
designated “by a memorandum left by me and known to him, or in
accordance with my known wishes.” Thereafter Nesmith created a
memorandum and a notebook in which she made entries giving certain
items of tangible personal property to certain beneficiaries. The
memorandum was created in 1972 and amended in 1976. The notebook
was titled “List to be given Nesmith 1979” and contained an entry
giving a painting to Clark. Nesmith also told Clark of her intent to leave
her the painting. Nesmith executed two codicils to her will in 1980.
Upon Nesmith’s death, Greenhalge refused to give Clark the painting.
On appeal, the court held that the language of the will was broad enough to include both the memorandum and the notebook, and that the will
described the notebook with reasonable certainty. Further, the notebook was in existence at the time of the execution (and maybe the notebook
entry??) of the codicils. The court also held that the will gave the testator the right to alter and amend the bequests of tangible personal
property in her will, without having to amend it formally. Therefore, the notebook was incorporated by reference into the will, and Clark
was entitled to the painting.
4. Acts of Independent A. UPC § 2-512. Events of Significant Independence. A will may dispose of property by reference to acts and events
Significance (Doctrine that have significance apart from their effect upon the dispositions made by the will, whether they occur before or after
of Non-testamentary the execution of the will or before or after the testator’s death. The execution or revocation of another individual’s will
Acts) is such an event. {McL}
B. Acts of Independent Significance: If the beneficiary of property designations is identified by reference to acts or
events that have a lifetime motive and significance apart from their effect on the will, the gift will be upheld under the
doctrine of acts of independent significant (a.k.a. the doctrine of nontestamentary acts).
 Example—Leaving Property to People That Will Take Under Another’s Will: A testator may intend for
his property to go to the same persons who are named in another person’s will as devisees and legatees, and
the gift of the testator’s property by his will can be upheld on the ground of “independent significance.”
Klein.
o “Where A leaves property to such persons as may take the property of B under B’s will, the
disposition of B’s property is a fact of independent significance. B’s disposition of his own property
has significance quite apart from the effect which it may have on the disposition of A’s property.
Accordingly . . . where A leaves property to such persons as may take under the will of B, whether
B survives A or predeceases him, the disposition of A’s estate is valid and the persons who take
under B’s will are entitled to A’s estate.” Klein.
 B’s will does not have to be in existence at the time of A’s death.
 B may survive A. The residue would be held until B’s death.
 If B references A’s will, it might not be an act of independent significance.
 Example: “I give $1,000 to each of my son-in-laws.” Marriage is act of independent significance.
 Example: “I give the car I own at my death to . . .” “I give $10,000 to each employee in my employ at the
time of my death.” Acts of independent significance because you do things not for will purposes but for other
purposes.
 Example: “I give all of the stuff in my garage to my brother, Bob.” Putting stuff in garage can be act of
independent significance (i.e., storing and using items)
 Example: “I leave $10,000 to each of the persons I will identify in a letter I will leave for my executor.” NOT
act of independent significance—serves only the purpose of controlling who takes under the will.
B. Extrinsic Evidence: Under the doctrine of independent significance, a court may refer to extrinsic evidence to
identify the persons who are to take under the will. In re Will of Tipler.
C. RUN THIS THROUGH THE INCORPORATION BY REFERENCE & INTEGRATION ANALYSES
First Nat’l Bank v. Klein (AL): Leslie (the testatrix) executed a will devising one-third of her residuary estate to a son. Leslie later executed a
codicil which changed the distribution to the residuary legatees and the residuary beneficiaries of the son’s estate, should the son predecease her.
The son predeceased Leslie. Under the terms of the son’s will, his residuary estate was given to a trust benefitting his second wife. On appeal,
the court held that Leslie could leave part of her estate to those who would take under her son’s will, whether the son predeceased her or not.
Leslie’s codicil—by the words, “residuary legatees and residuary beneficiaries of his estate under his last will and testament”—designated those
who were the recipients of the son’s bounty, and the language was sufficiently certain to identify the class intended. Thus, the codicil was not
void for uncertainty, but was valid by reason of facts of independent significance. Moreover, it did not depend for validity upon the son’s choice
of beneficiary having been communicated to Leslie.
In re Will of Tipler: Tipler (the testatrix) executed a formal will that left her property to her husband and two days later executed a holographic
codicil to her will leaving her property to be distributed in accordance to her husband’s will if he should have predeceased her. At this time the
husband had not executed his will. The husband predeceased Tipler. Tipler’s heirs contended that the codicil was invalid because it incorporated
a document not yet in existence when the codicil was executed. On appeal, the court found that the doctrine of facts of independent significance
was satisfied because her husband’s will had independent significance of distributing his estate and was not written with the intention of
distributing Tipler’s estate.
4. Wills: Capacity & Contests
 Introduction: The traditional method of opting out of intestacy is to execute a will. One requirement for creating a valid will is that the
testator have testamentary capacity. Even if a testator has testamentary capacity, if the testator suffers from a defect in capacity (i.e.,
insane delusions, undue influence, duress, fraud), a will may be contested. {CT}

32/87
 If a will is set aside for lack of capacity, insane delusion, or undue influence, the will is set aside, and the probate property passed by a
previous will or by intestate succession. Watch out for when a wrongdoer also marries the testator because the surviving spouse can take
under intestate success.
A. Capacity to Make a Will
A. Capacity to Make a Will: “An individual 18 or more years of age who is of sound mind may make a will.” UCC § 2-501. {McL} A contestant
can disprove sound mind by disproving general mental capacity or by showing an insane delusion.
1. Mental Capacity A. Introduction: The first requirement for a valid will is the testator must have testamentary capacity. {CT}
1. Policy Justifications
2. Temporal Application: The testator must have the requisite capacity at the time he or she performs a testamentary
act—executes or revokes a will. {CT}
 Lucid Interval: If a person who usually lacks testamentary capacity executes a will during a lucid moment,
the will is valid even though the testator lacked capacity for some period of time before and/or after executing
the will. {CT}
B. Requirements: The testator must be capable of knowing and understanding in a general way [1] the nature and
extent of his or her property, [2] the natural objects of his or her bounty,10 and [3] the disposition that he or she is
making of that property, and must also be capable of [4] relating these elements to one another and forming an orderly
desire regarding the disposition of the property.” RS3d Prop. § 8.1(b). This test for testamentary capacity is one of
capability, not knowledge.
1. Presumption of Testamentary Capacity: The legal presumption is always in favor of sanity, especially after
attestation by subscribing witnesses. A duty attaches to the witness to satisfy himself of the competency of the party
before he lends his name to attest the act. In re Wright’s Estate.
2. Low Threshold: Testamentary capacity cannot be destroyed by showing a few isolated acts, foibles, idiosyncrasies,
moral or mental irregularities or departures from the normal unless they directly bear upon and have influenced the
testamentary act. In re Wright’s Estate. {McL}
 Intelligence: Testator does not have to be of average or above average intelligence.
3. Testamentary Capacity vs. Contractual Capacity: In most states, capacity to make a will (and a revocable trust or
other will substitute) requires less mental ability than to make a contract or to complete an irrevocable lifetime gift.
 Rationale: Freedom of disposition; A dead person does not need protection from economic loss and is not at
risk of impoverishment.
 Appointment of Conservator: A person under a conservatorship may have testamentary capacity to make a
will—depending on the facts of the case. The lawyer should consult with the conservator before preparing a
will for such person.
 Institutionalized: If lucid moment—they may have testamentary capacity.
4. Testamentary Capacity v. Marriage Capacity: Because of the policy in favor of marriage, testamentary capacity is
higher than the capacity necessary to marry. As a result, a testator may have marriage capacity but not testamentary
capacity. If a will is set aside for lack of testamentary capacity, a marriage could still be valid. In that case, the surviving
spouse may take under intestate succession.
5. Will vs. Revocable Trusts & Will Substitutes: Capacity for revocable trust and other will substitutes is the same
has testamentary capacity.
6. Attorney’s Ethical Duty: The lawyer generally should not prepare a will, trust agreement or other dispositive
instrument for a client who the lawyer reasonably believes lacks the requisite capacity. The lawyer may assist clients
whose testamentary capacity appears to be borderline. In any such case the lawyer should take steps to preserve
evidence regarding the client’s testamentary capacity. ACTEC comments.
 Witness to Will Execution: Has duty to testify that testator was of sound mind.
7. Burden of Proof: Under the majority approach, once the will proponent offers prima facie proof that the testator had
testamentary capacity, a rebuttable presumption arises that the testator had testamentary capacity and the contestant
bears the burden of proving lack of capacity. Wilson: Wright; Breeden; UPC § 3-407. The minority approach holds that
once a contestant introduces evidence of lack of capacity, the will proponent bears the burden of proving testamentary
capacity.

In re Wright’s Estate (CA): Wright (testator) was 69 years old and eccentric at the time he died. He kept junk and old liquor bottles hidden
around his house, sprayed kids with hoses, held his breath to pretend he was dead, etc. His will devised a house to a friend and another house to

10 The people society would expect the testator to leave his property to—those that would inherit under the laws of intestate succession.
33/87
his daughter, who contested the will. The drawer of the will and three witnesses all testified that they thought that Wright was of unsound mind
based upon minor idiosyncrasies. On appeal, the court held that because there was no proof that Wright was insane or suffered from delusions,
there was no proof of testamentary incapacity. There was no evidence that he did not appreciate his relations and obligations to others, or that he
was not mindful of the property which he possessed.
Wilson v. Lane (GA): Greer (testator) signed the will in question three
years before her death. At trial, the attorney who drafted her will testified
that, in his opinion, Greer was mentally competent at the time that the will
was signed and that she emphatically selected all 17 beneficiaries (16
family members and 1 caretaker). The contestants challenged Greer’s
capacity by showing that she was eccentric, aged, and peculiar in the last
years of her life (irrational fear of flooding, reported nonexistent fire to
fire department, did not bath). The contestants also introduced evidence
that Greer was suffering from dementia; a letter from Greer’s physician
that she suffered from senile dementia; and a guardianship petition. On
appeal, the court found that the evidence showed that Greer had
testamentary capacity in that she was able to form a rational desire
regarding the disposition of her property. The court noted that no
testimony was offered to establish that, at the time that the will was
executed, Greer suffered from a form of dementia sufficient in form or
extent to render her unable to form a decided and rational desire regarding the disposition of the assets (the doctor had not examined Greer
personally; Greer’s physician drafted the letter to help Greer get a benefit; many old people have guardians). In summary, the factors leading to
the Majority’s holding: (1) standard for capacity is very low; (2) Majority rule: burden of proof on contestants to prove lack of capacity; (3) the
disposition was not unusual.
2. Insane Delusion A. Introduction: A person may satisfy the test for testamentary capacity but nonetheless be suffering from an insane
delusion that causes the entire will or a particular disposition to fail for lack of capacity.
 Insane Delusion: To prevail on an insane delusion case, the contestant must show both that (1) the testator
labored under an insane delusion and (2) that the will or some part thereof was a product 11 of the insane
delusion. {McL}
 Rationale: The rule was devised to cover a gap in existing law, which held that “idiots and persons of non-
sane memory” could not make wills, but accepted as valid the will of a testator “who knew the natural objects
of his or her bounty, the nature and extent of his or her property, and could make a ‘rational plan’ for
disposition, but who nonetheless was crazy as a March hare.” Dougherty. {McL}
1. Definition—Insane Delusion: An insane delusion is a false conception of reality to which the testator adheres
against all evidence and reason to the contrary. See Breeden. {McL}
 Delusion: A delusion is a false conception of reality. {McL}
 Mistake: Under traditional law, courts do not reform or invalidate wills because of mistake. A mistake is
susceptible to correction if the testator is told the truth. {CT}
2. Majority: In most states, if there is any evidence to support the testator’s delusion, the delusion is not insane.
3. Traumatic Event: Court is more likely to conclude that the testator had an insane delusion when something
traumatic happened to the testator than to find that an insane delusion just developed. {CT}
4. Traditional Approach—Social Context: What constitutes a socially acceptable belief is somewhat time and place
sensitive. See In re Strittmater’s Estate.
B. Causation:
 Majority/Modern: There must be a causal connection between the insane delusion and testamentary
capacity: Before a will can be invalidated because of lack of testamentary capacity due to an insane delusion,
the insane delusion “must materially affect or influence” the will or its provisions. Breeden. {McL}
 Older Cases: Causation was presumed if there was an insane delusion and an unnatural disposition. The
insane delusion was deemed to have caused the bequest if it “might have been caused or affected.” See
Strittmater.
In re Strittmater’s Estate (NJ): Strittmater (testatrix) had schizophrenia (split personality), according to her doctor. She had a normal
childhood and was devoted to her parents. However, four years after her parents’ death, she wrote terrible things about her parents. She also
wrote numerous memoranda and comments in margins of books saying that she hated men, she looked forward to the day when women would
bear their children without the aid of men, and all males would be put to death and birth. She smashed a clocked and killed a pet kitten. She also
joined the National Women’s Party (feminist group). However, in her dealings with her lawyer, she was entirely normal and reasonable (she had
general mental capacity). Strittmater left her estate to the party. The court set her will aside because (1) it felt her hatred of men demonstrated
she was insane and (2) it was that insane delusion about the male that that caused her to leave her estate to the National Women’s Party.
Breeden v, Stone (CO): Just prior to his suicide, Breeden (decedent) penned a holographic will purporting to leave everything to Sydney Stone;
in doing so, he disinherited his family. Stone offered the will for probate; Breeden’s family challenged it claiming that Breeden lacked general
mental capacity and because he suffered from insane delusion. On appeal, the court found Breeden had the requisite mental capacity because
Breeden (1) could index the major categories of the property comprising his estate; (2) knew his home and rental addresses; and (3) identified
the devisee by name and provided her current address. The court noted that the will was legible, logical in content, and reasonably set out
Breeden’s intent. The court found that Breeden was suffering from insane delusions at the time he executed his will (i.e., delusion regarding
listening devices in his home and car and assassination plots against himself and his dog, he also used drugs and alcohol) but held that the
insane delusions did not materially affect or influence the disposition made in the holographic will. The court weighed (1) the testimony of

11 Causation
34/87
numerous expert witnesses regarding the Breeden’s handwriting, his mental state near the time he executed the will, and the impact of his drug
and alcohol use on his mental faculties and (2) evidence that Breeden was not close to his family, had infrequent contact with them, indicated to
friends that he believed his father was irresponsible with money, disliked his sister’s husband, did not have a close relationship with his brother,
and had not made provisions for them in a previous will.
B. Undue Influence
 The second defect in testamentary capacity that may render a will invalid is undue influence.
1. What is Undue A. Two Ways to Show Undue Influence: (1) Contestant establishes every element; or (2) burden-shifting approach.
Influence? 1. Undue Influence—Contestant Must Establish the Following: To establish Undue Influence, the burden of proof is
on the person contesting the will to show (1) T was susceptible to undue influence; (2) the influencer had the
disposition to exercise undue influence; (3) the influencer had the opportunity to exercise undue influence; (4) the
disposition was the result of the undue influence; and (5) the wrongdoer exerted such influence over T that it overcame
T’s free will and caused T to make a transfer that T would not otherwise have made. {McL}
 Rationale: Protect against overreaching by a wrongdoer seeking to take unfair advantage of a donor who is
susceptible to such wrongdoing on account of the donor’s age, inexperience, dependence, physical or mental
weakness, or other factor; Protect testator’s freedom of disposition. {CT}
 Example: Lipper; Kidnapping, torture, and brainwash of testator who then wills over everything to the
kidnapper.
2. Burden-Shifting Approach: In most states, if a presumption of undue influence is triggered, the burden shifts to the
proponent to come forward with rebuttable evidence. In the absence of such evidence, the contestant is entitled to
judgment as a matter of law.
a. Presumption of Undue Influence: A presumption of undue influence arises if (1) there was a confidential
relationship between the defendant and the testator and (2) there are suspicious circumstances present. RS3d § 8.3. cmt.
f. After the presumption arises, the burden of proof shifts to the proponent of the will, and the proponent must prove
absence of undue influence by clear and convincing evidence. {McL}
 Rationale: Most people who exercise undue influence do not leave a trail of their wrongdoing and often
perjure themselves.
 Example: Lakatosh: Reid.
i. Confidential Relationship: The term “confidential relationship” includes (1) a fiduciary relationship, (2) a reliant
relationship, and (3) a subservient relationship. RS3d Prop. § 8.3 cmt. g. {McL}
 Fiduciary relationship: A fiduciary relationship is for example, the attorney/client relationship or the
principal/agent under power of attorney relationship. Lakatosh.
 Reliant relationship: A reliant relationship is one based on special trust and confidence and is a question of
fact (i.e., financial adviser/customer, doctor/patient)
 Dominant-Subservient Relationship: A dominant-subservient relationship is when the donor was
subservient to the alleged wrongdoer’s dominant influence and is a question of fact (i.e., hired caregiver/ill or
feeble donor; adult child/ill or feeble parent).
ii. Suspicious Circumstances: The following are indicators of suspicious circumstances: (1) whether the donor was in
a weakened condition, physically, mentally, or both; (2) whether the confidant receives the bulk of the estate; (3)
whether the alleged wrongdoer participated in the preparation or procurement of the will; (4) whether the donor
received independent advice from an attorney or other disinterested advisor in preparing the will; (5) whether the will
was prepared in secrecy or haste; (6) whether the donor’s attitude toward others had changed by reason of her
relationship with the alleged wrongdoer; (7) whether there is a decided discrepancy between a new and previous wills
of the donor; (8) whether there was a continuity of purpose running through former wills indicating a settled intent in
the disposition of her property; and (9) whether a reasonable person would consider the disposition unnatural, unjust, or
unfair. {McL}
B. Nontraditional Relationships: Court was suspicious of elderly woman/young man relationship. See Reid.
C. Gifts to Attorneys/Interested Drafter: Many courts hold that a presumption of undue influence arises when a
lawyer receives a bequest under a will that the lawyer drafted, unless the lawyer is closely related to the testator. The
presumption can be rebutted only by clear and convincing evidence provided by the lawyer. See Lipper.
D. Ethical Considerations: A lawyer shall not . . . prepare . . . an instrument giving the lawyer or a person related to
the lawyer any substantial gift unless the lawyer or other recipient of the gift is related to the client. MRPC 1.8(c).
{McL}
 Don’t draft wills that give you gifts unless you are related.
 Drafting wills for family members is acceptable, but if there is a conflict of interest (e.g., disinheriting an
heir), then go to an independent attorney.

35/87
Estate of Lakatosh (PA): Rose Lakatosh (testator) was a 70-
year-old woman when Roger Jacobs befriended her. She lived
alone, was easily distracted, and was forgetful. She quickly
became dependent on Roger. He visited her daily, sometimes
several times a day, and took her on errands. A few months after
first meeting Roger, Rose gave him power of attorney over her
affairs (“to protect her”) and executed a will giving all but $1,000
of her $268,000 estate to him. Roger was not present when the
will was drafted or executed, but the attorney was his cousin (to
whom Roger had referred Rose for a slander suit). Roger used the
power of attorney to use Rose’s assets ($128,000) for his own
benefit and that of his friends (whom Rose did not know). Rose’s
living standards deteriorated, and shortly before her death, she
revoked the power of attorney but not the will. The guardian of
Rose’s estate sought revocation of Lakatosh’s will on grounds of
undue influence. On appeal, the court held that the trial court
properly concluded that a confidential relationship existed between Rose and Roger because the grant of a power of attorney to Roger was in
itself sufficient for a finding that a confidential relationship existed. The court also found that Roger was to receive the bulk of Rose’s estate
since he was to receive all but $1,000 of the $268,000 estate. The court found that Rose suffered from a weakened intellect at the time the will
was executed because had trouble remembering things and had no understanding of her estate or assets; Rose’s attorney questioned her
competence in the slander law suit; an audio tape revealed that Rose was easily distracted and clearly had difficulty remaining focused on the
will at the time of execution; and Rose was living in shambles (house partially burned down, dead cats in freezer and bathtub). The court held
that Roger failed to carry his burden of proving the absence of undue influence. A constructive trust was to be imposed and Rose’s will was to
be revoked.
In re Estate of Reid (MS): While in law school, 24-year-old Cupit developed a (perhaps intimate) relationship with 78-year-old Mary Reid
(testator), had her adopt him (to cut off any potential heirs), had Reid convey her property to Cupit in a deed (she devised him 205 acres and an
antebellum home for $10, reserving a life estate for herself), and then helped her compose a holographic will devising all of her property to him.
Cupit then took Reid to another lawyer and made an attested will based on the holographic will. After law school, Reid gave Cupit power of
attorney. Reid died at age 96. Regarding the deed and will, the court held that a confidential relationship existed and Cupit did not disprove the
presumption of undue influence, because he did not act in good faith: Cupit initiated the preparation of the deed and gave $10 consideration for
the gift; and Cupit helped Reid prepare the holographic will and the attested will was almost an exact copy of it. Reid did not receive
independent counsel in making her will either: the drafting attorney acted as a mere scribe.
Lipper v. Weslow (TX): Sophie Block’s (decedent’s) will disinherited
her 3 grandchildren (Weslows) by a deceased son and left the estate to
her 2 surviving children (Frank and Irene Lipper). The Weslows sued
Lipper, alleging that the will was the result of undue influence by
Frank. Frank lived next door to Block, was the attorney who drafted the
will, bore malice toward the pre-deceased half-brother, and had a key to
Block’s house. Block was 81 when she executed the will; and she did
not read the will or discuss its terms when she signed the will. The will
contained a paragraph that stated that Block disinherited the
grandchildren because they and their mother had been most
“unfriendly” to her since the death of her son, though there was
conflicting evidence as to the accuracy of this assertion. Block was of a
sound mind and strong will. On appeal the court held that, while the
Weslows did establish a confidential relationship (Frank was Block’s attorney) and an opportunity and a motive for undue influence by Lipper
(see above), they failed to prove that the will resulted (causation) from his substituting his mind and will for Block’s.
3. Planning for &
Avoiding a Will
Contest
a. Warning Signs A. Anticipating & Deterring Challenges:
 “Unnatural” Disposition: Perhaps the most common warning sign that a contest is more likely than usual is
an unnatural disposition—i.e., an unexpected omission of a close family member; an unexplainable
distinction among family members of equal relation; or disinheritance, disfavor, or disparagement of a
child. }
b. Strategies to 1. Explanatory Statement: One professor has found that individualized, expressive language in the will stating the
Deter Will Contests reasons for what might appear to be an unnatural disposition is helpful in resisting a later claim of undue influence. See
Lipper. It is better to use the handwritten letter technique outlined below.
 Three Dangers: (1) any factual recitation for the disinheritance must be accurate; (2) will becomes part of
public record—if a hurtful statement is in the will, it may trigger a will contest; (3) testamentary libel—a
disinherited heir that is insulted in the will can sue the estate for testamentary libel.
2. No-Contest or in Terrorem Clause: A no-contest clause deprives an unsuccessful contestant of her bequest under
the challenged will. If enforceable and if accompanied by a substantial enough bequest, a no-contest clause may deter a
contest. See Lipper (“If any legatee contests the will, they forfeit all benefits under the will.”). However, a no contest
clause may not have any potency unless the client is willing to make a significant bequest to the potential contestant.
See Lipper. Best practice is to make an alternative disposition to a charity, which will fight to uphold a will.

36/87
 Example: “$100,000 to each grandchild, but if any grandchild contests the will, all Grandchildren forfeit all
benefits and the $100,000 bequests pass to CHARITY.”
 a. Public Policy Considerations: Enforcement of such clauses may deter unmeritorious litigation, family
quarrels, and damage to the testator’s reputation. But enforcement might also suppress a contest of a flawed
will.
 b. Majority: The majority rule is to enforce no-contest clause only if the unsuccessful contestant lacked
“probable cause” for bringing the contest. See UPC §§ 2-517 and 3-905.
 c. Minority: A minority of states do not enforce no-contest clauses at all or enforce them unless the contest
alleges forgery or subsequent revocation.
3. Extra Precautions at a Will Execution: The lawyer should take extra precautions at the will execution:
 a. Choosing Witnesses: Use witnesses who represent well in court, perhaps friends of the testator or
community leaders
 b. Conversation with Witnesses: Ask the client to tell the witnesses about her dispositive plan and the
reasoning behind it
 c. Affidavits: Ask witnesses to sign affidavits relating their impressions of the testator and what they saw,
heard, and talked about at the time of execution
4. Handwritten Letter: Have the client handwrite a letter to the lawyer setting forth in detail the disposition the client
wishes to make. The lawyer replies detailing the consequences and asks for a letter detailing the reasons for the
disposition. The testator responds that she understands and still wants to proceed.
5. Video Recording: Video recording of a discussion between the testator and the lawyer during which the testator
explains his dispositive plan. The discussion may include why the testator wants to disinherit an heir or otherwise
deviate from the norm. But remember, any facts stated by the testator as justifying disinheritance may be contradicted
by a contestant.
6. Dictation to a Stenographer: Dictation to a stenographer
7. Family Meeting: Hold a family meeting at which the testator explains his dispositive plan and rationale
8. Professional Examination: A professional examination of the client’s mental capacity immediately before executing
a will or trust can make it much more difficult for a contestant to prove lack of capacity.
9. Inter vivos trust: Although trusts can be contested, as a practical matter it is harder to upset a trust if the settlor had a
course of dealing with the trustee to evidence competence and the absence of undue influence. In most states, the
existence of an inter vivos trust may be kept secret to the donor’s heirs. In most states, a trust dispute is tried before a
judge rather than a jury.
10. Inter vivos gift: Unlike a transfer at death, the donor is able to testify in defense of her satisfy and absence of undue
influence.
5. Wills: Construction
 Construction of Wills: The process of determining the meaning that should be attributed to a will.
 Freedom of Disposition: The controlling consideration in determining the meaning of a donative document is the donor’s intention.
RS3d § 10.1.
 The Worst Evidence Rule: The best witness is dead by the time the issue is litigated.
 Overview: At the macro level, basically two types of construction problems can arise with respect to a will: (1) a drafting mistake and
(2) a change in circumstances. {CT}
A. Mistaken or Ambiguous Language in Wills
 There are three approaches that courts have taken in dealing with mistaken or ambiguous language in wills: (1) plain meaning and no
reformation; (2) ad hoc relief; and (3) open reformation.
1. Plain Meaning & No A. Plain Meaning & No Reformation (Majority): In construing wills, a majority of states follow two rules that,
Reformation operating in tandem, bar the admission of extrinsic evidence to vary the terms of a will: (1) the plain meaning or no-
(Traditional extrinsic evidence-rule and (2) the no reformation rule.
Approach)  Rationale: (1) Worst evidence problem—because a testator is unable to corroborate or refute extrinsic
evidence of intent that is at odds with the words of her will, she is protected from fraud and error by
categorically excluding such evidence; (2) Evidence of extrinsic evidence would violate the Wills Act
formalities; (3) security of wills.
1. Plain Meaning 1. Plain Meaning Rule (Majority): Extrinsic evidence may be admitted in order to resolve certain ambiguities, but the
Rule plain meaning of the words of a will cannot be disturbed by evidence that the testator intended another meaning.
{McL}
 Example: Mahoney.
 Some courts allow evidence of circumstances but not declarations of testator.
 Examples of No Ambiguity: Estate of Smith (finding no ambiguity in the phrase “heirs per stirpes” even
when the devise would descend to a single person); Estate of Scale (finding no ambiguity where T left
residuary estate to “The Audubon Society of New York State” when there existed the National Audubon
Society (founded and operated in New York) and the Audubon Society of New York State, Inc.)
 Testamentary Intent: Testimony of the drafting lawyer is admissible to show that a duly executed will,
which recites that it is the testator’s will, was not intended as such—thus contradicting the words of the
instrument.
2. No Reformation 2. No Reformation Rule (Majority): Courts may not reform a will to correct a mistake term to reflect what the testator
Rule intended the will to say. {McL}
 Example: Mahoney.
3. Personal Usage 3. Personal Usage Exception: If extrinsic evidence shows that a testator habitually used a term in an idiosyncratic
37/87
Exception manner, the evidence is admissible to show that the testator used that term in accordance with his personal usage rather
than its ordinary meaning.
4. Patent vs. Latent 1. Patent (Obvious) Ambiguity: An ambiguity that appears on the face of the will. {McL} (i.e., “two hundred
Ambiguity thousand dollars ($25,000)”, bequest of entire estate: 25% each to three named charities).
 Traditional: Extrinsic evidence is not admissible to clarify a patent ambiguity. The court is confined to the
four corners of the will even if as a result the ambiguous devise fails and the property passes by intestacy.
 Modern Trend: Courts are inclined to admit reliable extrinsic evidence to resolve a patent ambiguity. See
Cole.
o Reliable evidence: Surrounding circumstances; Testimony of drafting attorney
2. Latent (Hidden) Ambiguity: An ambiguity that does not appear on the face of the will but appears when the terms
of the will are applied to the testator’s property or designated beneficiaries. {McL} (i.e., a description for which two or
more persons or things exactly fit, a description for which no person or thing fits exactly but two or more person or
things fit partially)
 Traditional: At common law, the court admit extrinsic evidence to help construe a latent ambiguity.
 Testimony by drafting attorney more likely to admitted than testimony of family or friends.
a. Example—Equivocation: When two or more persons or things fit the description exactly (e.g., a devise “to my
niece Alicia,” when the testator has two nieces named Alicia).
b. Example—No Exact Fit: A description in a will does not exactly fit any person or thing.
c. Example--Personal Usage: T used term in idiosyncratic manner.
B. Collapsing the Distinction Between Patent & Latent Ambiguities: Modern courts are increasingly inclined to
admit extrinsic evidence to resolve both patent and latent ambiguities (see Cole). Some courts have abandoned the
distinction as irrelevant.
Mahoney v. Grainger (MA): When Helen Sullivan (testator) died her sole
heir at law was Hawkes Greene (her maternal aunt). Helen was a single
woman of about 64. She lived alone but maintained cordial relationships with
her aunt and several first cousins. When Helen was making her will, her
attorney asked her who she wanted to “leave the rest of her property to.” She
replied, “I’ve got about twenty-five first cousins. Let them share it equally.
The will’s residuary clause provided for the residue to be “divided equally”
among to the Helen’s “heirs at law living at the time of [her death].” Helen’s
heir under the intestate succession statute was her aunt because she was the
surviving relative of the closest degree. The cousins sought to admit Helen’s
statement to her attorney. The probate court ruled that Helen’s statements to
the attorney were admissible only to show the material circumstances at the
time of executing the will. Regardless of language that the heirs were to
divide the residue among them equally, it was clear that Helen had but one heir (the aunt). There was no latent ambiguity in the will permitting
Helen’s statements to be introduced to prove her testamentary intent. The court affirmed on appeal, finding no ambiguity in the will; the words
referred alone to the aunt, not the cousins. The use of the plural word “heirs” did not prevent the aunt from taking the entire gift.
In re Estate of Cole (MN): The will of Ruth Cole (decedent) stated a bequest to her friend, Veta Vining, in “the sum of two hundred thousand
dollars ($25,000).” Ruth’s personal representative introduced evidence (file notes and an affidavit) that Ruth’s attorney had copied and pasted a
paragraph, but had forgotten to change the words to say, “twenty-five thousand dollars.” On appeal, Veta disputed the trial court’s determination
to consider testimony of the attorney. The court held that the trial court properly determined what type of extrinsic evidence should be
considered when construing the ambiguous and contradictory provision of Ruth’s will. The trial court first considered the surrounding
circumstances before examining extrinsic evidence. The extrinsic evidence (the Attorney’s testimony) was used only to determine what Ruth
meant by the words used.
2. Ad Hoc Relief for A. Ad hoc examples of Reformation: Some argue that courts are already de facto reforming wills on an ad hoc basis
Mistaken Terms when they feel the evidence and equities justify it. {CT}
B. Courts Still Profess They Have No Power to Reform: “It matters not that an obvious mistake in the form of a
misdescription is proved. A court has no power to correct or reform a will or change any of the language therein by
substituting or adding words. The will of a decedent executed pursuant to statute is what it is and no court can add to
it.” Arnheiter; Gibbs’. {McL}
C. Correcting Mistakes—Trend: Court have sometime corrected a mistake under the guise of using extrinsic evidence
to construe a supposedly ambiguous term.
 1. Falsa Demonstratio Non Nocet (Mere Erroneous Description Does Not Vitiate): Where a description of
a thing or person consists of several particulars and all of them do not fit any one person or thing, less
essential particulars may be rejected provided the remainder of the description clearly fits. Arnheiter.
o *There is no functional difference between reforming the will and using the doctrine of falsa
demonstratio non nocet.
 2. Clear & Convincing Evidence: Wills must not be reformed even in the case of demonstrable mistake.
Gibbs’. Details of identification, particularly such matters as middle initials, street addresses, and the like,
which are highly susceptible to mistake, particularly in metropolitan areas, should not be accorded such
sanctity as to frustrate an otherwise clearly demonstrable intent. Where such details of identification are
involved, courts should receive evidence tending to show that a mistake has been made and should disregard
the details when the proof establishes to the highest degree of certainty (clear and convincing evidence) that a
mistake was, in fact, made. Gibbs’.

38/87
Arnheiter v. Arnheiter (NJ): Burnette Guterl (decedent) left a last will
and testament that specifically devised “No. 304 Harrison Avenue,
Harrison, New Jersey” to her two nieces. However, Burnette did not own
that property: She owned 317 Harrison Avenue, Harrison, New Jersey.
The executrix applied to the court to correct the mistake and to change the
street number in the will to the proper address. The court noted that it did
not have the power to correct or reform a will or change any of the
language in it by substituting or adding words. Instead, the court granted
relief to the executrix by construing the will provision under the doctrine falsa demonstratio non nocet. By applying this doctrine, the court
dropped the house number (304) and, because Burnette only owned one parcel of real property on Harrison Avenue, and did not otherwise
dispose of it in any other provision of the will, was able to conclude that the remaining description of the property (“Harrison Avenue, Harrison,
New Jersey”) was sufficient to identify the property (317).
In re Gibbs’ Estate (WI): Mr. and Mrs. Gibbs died approximately one month apart. Each had a provision in their will devising one percent of
their respective estates to “Robert J. Krause, now of 4708 North 46th Street, Milwaukee, Wisconsin.” Robert W. Krause, who was a 30-year
employee of Mr. Gibbs and friend of the family, claimed the gifts. Robert J. Krause, who lived at the address listed in the will, also claimed the
gift. The trial court found that the Gibbs intended to leave the bequest to the friend (RWK), not the stranger (RJK), and so ordered. On appeal,
the court first found that RJK could not offer any logical reason why the Gibbs would have left anything to him in the will (maybe a taxi
incident). However, the court observed that there was no ambiguity—RJK was the only person bearing the name of the legatee in the will.
Ordinarily, where there is no ambiguity, courts would not reform wills. However, the court found where there were details subject to mistake,
such as middle initials and addresses, courts overlooked such details in preference to the intent of the decedent. Here, the trial court properly
disregarded the middle initial and street address, and determined that RWK was the Robert Krause whom the Gibbs had in mind.
3. Openly Reforming A. Overview: A small but growing number of courts are willing to reform a mistaken term in a will and do so openly.
Wills for Mistake B. Minority/Restatement/UPC: Extrinsic evidence of mistake by a scrivener is admissible and, if proved by clear and
convincing evidence, the court may reform the will to reflect the testator’s actual intent. See UPC § 2-805.
Reformation to Correct Mistakes. (“The court may reform the terms of a governing instrument, even if
unambiguous, to conform the terms to the transferor’s intention if it is proved by clear and convincing evidence what
the transferor’s intention was and that the terms of the governing instrument were affected by a mistake of fact or law,
whether in expression or inducement.”); RS3d Prop. § 12.1 (An unambiguous document may be reformed, but to do so
takes a showing of clear and convincing evidence that the donor’s intention differed from the terms of the contract. In
a construction suit to resolve ambiguities, the donor’s intention need only be established by a preponderance of the
evidence.)
 Rationale: (1) If attorney intentionally or fraudulently makes change to will, extrinsic evidence is allowed;
this is no different; (2) Courts allow extrinsic evidence in testamentary capacity; undue influence; and insane
delusion cases; this is no different.
 Example: Herceg.
 The Contrary View: Reformation would essentially invite disgruntled individuals excluded from a will to
demonstrate extrinsic evidence of the decedent’s “intent” to include them. Fannery (MA).
C. Omissions: Where material is omitted from a will, the court may look at extrinsic evidence to correct the omission.
Herceg.
D. Inter Vivos Trusts & the Uniform Trust Code: Uniform Trust Code—adopted by about half the states—authorizes
reformation for mistake in a trust (both testamentary and inter vivos).
In re Estate of Herceg (NY): Eugenia Herceg (testator)
executed a will in 1999 with a residuary clause (10% of estate)
that was missing the name of the intended beneficiary. Herceg’s
attorney, the draftsman of the will, filed an affidavit stating that
the 1997 will had been redrafted in 1999, using computer
software that accidentally deleted some of the lines of the
residuary clause. On appeal, the court held there was clear and
convincing evidence that Colomba was the intended beneficiary
of the residuary, and that her name should be inserted into the
will because (1) previous wills provided for the residuary to pass
to Herceg’s nephew, or, if he was dead, then to his wife,
Colomba; (2) Columba was named the alternate executrix in the
will admitted to probate in the event of the death of her husband,
demonstrating that she had not fallen out of favor with Herceg
and had not been deliberately removed from the residuary; (3)
the attorney’s affidavit; and (4) a niece who would take in intestacy acknowledged the mistake.
B. Death of Beneficiary Before Death of Testator
 Introduction: Now we consider the rules that apply if a named beneficiary predeceases the testator or if beneficiary is deemed to have
predeceased
 If the testator’s actual intent is in not evidence, the court will apply rules of construction that are meant to implement the probable intent
of the typical testator.

39/87
1. Lapsed & Void A. Lapse: If a devisee does not survive the testator, the devise fails and is said to have lapsed. See supra survival
Devises requirement.
 Deemed to Have Predeceased: The lapse doctrine covers scenarios where the beneficiary is treated as
predeceasing the defendant as a result of (1) disclaimer; (2) failure to survive for survivorship period; (3)
slayer statute; and (4) revocation by divorce statute.
 Antilapse Statutes: Nearly all states have enacted antilapse statutes that, under certain circumstances,
substitute another beneficiary for the predeceased devisee.
 Common Law: A gift made by will is subject to a condition that the devisee survive the testator, unless the
testator specifies otherwise.
B. Failed/Void Gift—Common Law Default Takers: A gift may fail for a variety of reasons: it may be void, it may
lapse, it may be to an ineligible taker (e.g., a pet—see Russell), or the gift may violate the RAP. Whatever the reason, if
a gift fails and it is not “saved,” it falls in a cascading scheme. {CT}
 A dog cannot be a beneficiary under a will and an attempted gift to a dog is void. Russell.
a. Specific or General Devise: If a specific (i.e., my gold watch) or general devise (i.e., $10,000) lapses, the devise
falls into the residue, if there is one. Otherwise, it goes to intestacy.
b. Residuary Devise: If the residuary devise fails completely (e.g., lapses), the heirs of the testator take by intestacy.
c. Part of Residuary Devise: If there are multiple takers in the residuary clause, and the gift fails as to one or more of
them, but does not as to all of them, the jurisdictions are split as to what happens to the part of the residuary clause that
fails. {CT}
 i. Traditional Rule—No-Residue-of-a-Residue Rule: If only a share of the residue lapses, such as when one
of two residuary devisees predeceases the testator, at common law the lapsed share passes by intestacy to the
testator’s heirs rather than the remaining residuary devises. See Russell. This rule has been overturned by
statute or judicial decision in most states.
o Ambiguity & Extrinsic Evidence: Extrinsic evidence should be admitted so that the judge may be
placed in the position of the testator whose language he is interpreting. Russell.
 ii. Surviving Residuary Takers (Modern Rule): If residuary clause fails, the other part catches it. The part
that fails is distributed among the other beneficiaries in the residuary clause. UPC § 2-604(b) (the assumption
is that the testator would probably prefer for the remaining residuary devisees to receive the residue).
d. Class Gift: If a devise is to a class of persons (i.e., the children of Aaron, my nieces), and one member of the class
predeceases the testator, the surviving members of the class divide the gift.
2. Antilapse Statutes A. Antilapse Statutes: Virtually all states have adopted the anti-lapse doctrine statutorily, but the details of the statutes
vary greatly from state to state.
 Rationale—Presumed Intent: The theory behind the antilapse statutes is one of presumed intent: The
testator would prefer a substitute gift to the devisee’s descendants rather than for the gift to pass in accordance
with the common law of lapse. {McL}
B. Mechanics: Antilapse statutes do not prevent a lapse; rather, they substitute other beneficiaries, usually the dead
beneficiary’s descendants, if certain requirements are met.
C. Typical Antilapse Statute: A typical antilapse statute provides that (1) where there is a lapse, and (2) the
predeceased beneficiary meets the statutory degree of relationship to the testator; and (3) the predeceased beneficiary
has issue who survive the testator, then the lapsed gift goes to the issue of the predeceased beneficiary, unless (4) the
will expresses contrary intent. See UPC § 2-605(1969) (Duke, at 357).
a. Lapse  Beneficiary must actually predecease decedent or be treated as if predeceased
Requirement
b. Requisite Degree  An antilapse statute applies to a lapsed devise only if the devisee bears the particular relationship to the
of Relationship testator specified in the statute (i.e., descendant of T, descendant of T’s parents or grandparents).
 UPC: Must be grandparent or lineal decedent of grandparent. UPC § 2-206 (1969)
c. Survived by issue  The predeceased must have issue who survive not only the predeceased beneficiary, but also the testator.
{CT}

40/87
d. Contrary Intent A. Contrary Intent: Because the antilapse statutes are designed to implement presumed intent, they are default rules
that yield to an expression of the testator’s actual intent that is contrary to the statute.
 a. Majority Rule: Express requirement of survivorship (e.g., “if he survives me”) evidences intent that the
antilapse statute does not apply. {McL}
o i. Low Threshold: If a will is not clear, courts sometimes struggle with the question of whether it
imposes a condition of survival that precludes application of the antilapse statute. See Allen
(holding that the terms “living” and “share and share alike” in the phrase “to my living brothers and
sisters, A, B, and C, to share and share alike” precluded application of the antilapse statute);
Raymond (holding that “to my brothers and sisters that survive me” imposed a condition of survival
and precluded the antilapse statute). But see Belardo (holding the term “share and share alike,” by
itself, did not preclude application of the antilapse statute).
 b. UPC § 2-603(b)(3) (1990, rev. 2008): [W]ords of survivorship” (i.e., “if he survives me,” “to my
surviving children”) are not, in the absence of additional evidence, a sufficient indication of an intent” to
preclude application of the antilapse statute.
o Rationale: Words of survivorship likely came from the testator’s attorney’s boilerplate language in
will form—not the testator’s reasoning.
o Criticism: The UPC’s presumptions as to a testator’s intent cover the express words in the
testator’s own will.

 Good Drafting: “I give Blackacre to A, if A survives me, but if A does not survive me, to B if B survives me,
and if both A and B do not survive me, to be added to the residue of my estate.” {McL}
In re Estate of Russell (CA): Thelma Russell’s (decedent’s) holographic will stated, “I leave everything I own, Real & Personal to Chester H.
Quinn and Roxy Russell.” Roxy was her dog that predeceased her and Chester was her friend and neighbor. Thelma’s niece (the only heir)
challenged the gift. The trial court admitted Chester’s extrinsic evidence (address book and quitclaim deed) that tended to show that Thelma did
not want to die intestate. The trial court ruled that Thelma intended to leave
her entire estate to Chester with the hope that he would care for Roxy the dog.
On appeal, the court ruled it was proper to admit extrinsic evidence to show
that “Roxy Russell” was a dog. It then held that the will’s language
unambiguously demonstrated that Thelma intended to make a disposition of
the of the residue of the estate to Quinn and the dog in equal shares. The court
ruled that the intended gift to the dog was void because a dog could not be a
beneficiary under a will. The void gift was to pass to Thelma’s niece by
intestate succession.

6. Trusts: Characteristics & Creation


 Trusts have eclipsed wills as the preferred vehicle for implementing a donor’s freedom of disposition
 A trust is a legal arrangement created by a settlor in which a trustee holds property as a fiduciary for one or more beneficiaries. The
trustee takes legal title to the trust property, which allows the trustee to deal with third parties as owner of the property. The beneficiaries
have equitable title to the trust property, which allows them to hold the trustee accountable for breach of the trustee’s fiduciary duties.
 Trusts may be testamentary, created by will and arising in probate. Or they may be inter vivos, created during the settlor’s lifetime by
declaration of trust or by deed of trust, often as a will substitute to avoid probate.
A. The Trust in American Law
1. Origins of the Trust  The ancestor of the modern trust is the medieval use.
o O, owner of Blackacre, would enfeoff A and his heirs to hold Blackacre to the use of the friars.
 In 1535, Parliament enacted the Statute of Uses, which took legal title from the feoffee to uses and gave it to
the cestui que use.
o Use, feoffment to uses: trust
o Feoffor to uses: settlor (also grantor, trustor)
o Feoffee to uses: trustee
o Cestui que use: beneficiary

41/87
 Courts eventually held that the statute did not operate if the feoffee to uses was given active duties to perform
beyond simply holding title to the property. This was known as a trust.
2. Sources of Law  Common law; Restatement of Trusts; State Statutes based on uniform laws.
 Top-down reforms: Designed to update the law in view of the transformation of the irrevocable trust into a
management device for financial assets, the revocable trust as a will substitute, and the rise of the statutory
business trust.
 Bottom-up reforms: Tend to promote dead hand control, reflecting donor preferences.
 Toward the Uniform Trust Code: The Uniform Trust Code was the first systematic, national codification of
American trust law. It has been enacted in 25 jurisdictions (listed in Duke, at 389).
o Most of the UTC’s provisions state default rules that may be overridden by the terms of the trust.
The only exceptions are the mandatory rules scheduled in UTC § 105(b), such as a trustee’s duty to
follow the terms of the trust and to act in good faith. Any provision of the UTC that is not scheduled
in § 105(b) is a default that may be varied by the terms of the trust.
3. Vocabulary,  Settlor, Grantor, Trustor: A person who creates a trust.
Typology, and  Trustee(s): The party to whom the settlor transfers the trust property; the trustee holds legal title to and
Illustrative Uses exercises managerial responsibility over trust corpus or res. {McL}
 Beneficiary: Hold equitable title to and have “beneficial” ownership of trust corpus or res. {McL}
 Inter vivos Trust: A trust created during the settlor’s life. {McL} An inter vivos trust may be revocable or
irrevocable, depending on the intent of the settlor. An inter vivos trust can be used as a will substitute to avoid
probate. At the death of the settlor, the trust property is distributed or held in further trust in accordance with
the terms of the trust. An inter vivos trust may be created either. (1) by declaration of trust, whereby the
settlor declares himself to be trustee of certain property, or (2) by deed of trust, whereby the settlor transfers
to the trustee the property to be held in trust.
o Modern Presumption that Trust is Revocable: A revocable inter vivos trust permits the settlor to
revoke the trust and demand return of the trust property. The modern presumption is that the trust is
revocable.
 Testamentary Trust: A trust created by will (at Settlor’s death). {McL} Once established a testamentary trust
is irrevocable. Because a testamentary trust is created by will and arises in probate, it involves a probate
transfer.
 Illustrative Uses:
o Trust for incompetent person
o Discretionary trust
o Testamentary marital trusts
o Trust for minor
 Trust Distinguished from Equitable Charge: If a testator devises property to a person, subject to the
payment of a certain sum of money to a third person, the testator creates not a trust but an equitable charge.
Inter vivos Trust Testamentary Trust
Creation Declaration of Trust or Deed of Trust Will
Type of Transfer Nonprobate (Will Substitute) Probate
Revocability Revocable or Irrevocable Irrevocable
4. Bifurcation of A. Bifurcation: The hallmark characteristic of a common law trust is bifurcation: The trustee hold legal title to the trust
Ownership property, but the beneficiaries have equitable or beneficial ownership. Two categories of issues arise from this splitting
of legal and equitable ownership: (a) the effect on the rights of third parties with respect to the trust property and the
property of the trustee personally (asset partitioning), and (b) the powers and duties of the trustee and the corresponding
rights of the beneficiaries with respect to the trust property and against the trustee (fiduciary administration)
a. Asset Partitioning and the Rights of Third Parties: Asset partitioning rules separate the personal property and
obligations of an organization’s insiders from the property and obligations of the organization.
 Juridical Entity Status: For an organization that enjoys juridical entity status, such as a corporation, asset
partitioning is easy.
 Trust: A trust is not a juridical entity but rather is a fiduciary relationship. A trustee cannot sue, be sued, hold
property, or transact in its own name. Instead the trustee sues, is sued, holds property, and transacts with
respect to trust property in the trustee’s fiduciary capacity as such.
o Traditional Law: Trustee was personally liable for the debts and obligations arising from
ownership of the trust. The trustee had a right to indemnification out of the trust fund.
o Modern Law: A creditor of the trustee in the trustee’s fiduciary capacity recovers directly out of
the trust fund without recourse against the property of the trustee personally. A personal creditor of
the trustee has no recourse against the trust property. Modern law effectively splits the trustee into
“two distinct legal persons: a natural person contracting on behalf of himself, and an artificial
person acting on behalf of the beneficiaries. The contemporary American trust is in function
(though not in juridical form) an entity.
b. Fiduciary Administration & the Rights of the Beneficiaries: The trustee has the responsibility for managing the
trust property, but the beneficiaries bear the consequences of the trustee’s actions. To protect the beneficiaries from
mismanagement or misappropriation by a trustee, “in deciding whether and how to exercise the powers of the
trusteeship,” the trustee “is subject to and must act in accordance with the trustee’s fiduciary duties.”
42/87
 Fiduciary Duty: The law requires the trustee, as a fiduciary, to subordinate her interests to those of the
beneficiaries.
o Duty of Loyalty
o Duty of Prudence
 Subsidiary Rules: Reinforce and elaborate the loyalty and care norms and include the duty of impartiality
between beneficiaries; the duty not to commingle the trust property with the trustee’s own property; and the
duty to inform and account to beneficiaries.
 Damages: A trustee who breaches her fiduciary duties can be liable for compensatory damages to restore the
trust estate and trust distributions to what they would have been without the breach (make-whole
compensatory damages), and to disgorgement by the trustee of any profit to the trustee owing to the breach
(unjust enrichment).
 Compensation Denied/Trustee Removed: A trustee may also be denied compensation and removed as
trustee.
 The Four Functions of Trusteeship: Trusteeship involves four overlapping functions:
o (1) custodial—taking custody of the trust property and properly safeguarding it.
o (2) administrative—accounting and recordkeeping and making tax and other required filings
o (3) investment—reviewing the trust assets and making and implementing an investment program
for those assets as part of an overall strategy reasonably suited to the purpose of the trust and the
needs of the beneficiaries
o (4) distribution—making disbursements of income or principle to the beneficiaries in accordance
with the terms of the trust

5. A Trust Compared  The creation of a trust usually involves the creation of one or more equitable future interests as well as a
with a Legal Life present interest in the income.
Estate o Present Interest: Equitable Life Estate
o Future Interest: Equitable remainder for life; Equitable contingent remainder in fee simple
o Settlor’s Interest: Equitable Reversionary Interest
a. Legal Life Estate  A legal life tenant has no power to sell a fee simple unless such a power is granted in the instrument creating
the life estate.
 The life tenant has a duty to pay taxes and keep the property in repair
 If the life tenant gets into debt, the creditor can seize the life estate and sell it
b. Equitable Life  A trustee usually has broad enough powers to act promptly and to allocate the costs and benefits fairly
Estate—A Trust between life and remainder beneficiaries
 Third parties need deal only with the trustee
 An equitable trust can be put out of the reach of creditors, protecting an incautious beneficiary from himself.
6. Business Trusts  Large-scale business enterprises sometimes organize in trust form—the common law business trust. The trust
is still the preferred form of organization for mutual funds. Trusts are commonly used in asset securitization
and for employee pension funds.
 A little more than half the states have enacted business trust statutes—the statutory business trust.
7. Foreign Trust Law  The use of the trust for donative transfer is a distinctive characteristic of the Anglo-American legal tradition.
Foreign Trust law focuses on the commercial uses of the trust.
8. Basic Trust Rules 1. Same Party Can Wear All Three Hats—Need Duties to Someone Other Than Self: Although there are three

43/87
(Repeated & distinct parties to a trust—settlor, trustee, and beneficiary—the same person can wear all three hats at the same time as
Expanded Upon long as there is another trustee or another beneficiary. {CT} To have a valid trust, the trustee must owe fiduciary duties
Below) to someone other than herself.
 Merger: If O is the sole trustee and also the sole beneficiary, the equitable and legal titles merge, leaving O
with absolute legal title, as under UTC § 402(a)(5).
2. A Trust is not Created Until it is Funded: A trust cannot exist without the trust property, often called the res.
3. A Trust Will Not Fail for Want of a Trustee: A trust may have one trustee or several. A trustee may be an
individual or a corporation. A trustee may be the settlor, a beneficiary, or a third party. In the absence of a trustee, the
court one (usually the executor in the case of a testamentary trust)
4. Active Duties of Trustee: The trustee must have some active duties to perform. If the trustee has no duties at all, the
trust is said to be “passive” or “dry,” and it fails.
 Accounting: A trustee’s duty to maintain and render accurate accounts is a strict one and the trustee has the
burden to prove that the expenditures were made for trust purposes. Jimenez.
 Accepting Trusteeship: The law does not impose trusteeship upon a person unless the person accepts. A
trustee has onerous duties and is exposed to potential liability.
 Withdrawing as Trustee: Under traditional law, once a person accepts appointment as trustee, she can be
released from office only with the consent of the beneficiaries or by court order. UTC § 705 allows for
resignation by a trustee with 30 days’ notice to all interested parties.
9. Remedial Trusts A. Resulting Trusts: A resulting trust arises anytime a trust fails in whole or in part. The courts use it to require the
party holding the property (typically the trustee) to return the property to the settlor (or the settlor’s estate if the settlor
is dead).
B. Constructive Trust: Constructive trusts are used to prevent unjust enrichment. *?
B. Creation of a Trust
A. Requirements to Create a Trust: The creation of a trust requires (1) intent by the settlor to create a trust; (2) ascertainable beneficiaries who
can enforce a trust; and (3) specific property, the res, to be held in trust. In addition, if the trust is testamentary or is to hold land, (4) a writing
may be required to satisfy the Wills Act or the Statute of Frauds.
1. Intent to Create a A. Intent to Create a Trust: The first requirement or a valid trust is the intent to create a trust. The intent to create a
Trust trust exists when one party transfers property to a second party (trustee) with the intent of having the second party
retain and manage the property for the benefit of a third party. Lux.
o Typical Example: A transfer of property to X “for the use and benefit” of A.
 Evidence of the Party’s Intent: The best evidence of the party’s intent is the words he or she uses with
respect to the arrangement. No particular form of words is necessary to manifest an intent to create a trust.
Not even the word trust or trustee is required. The settlor need only manifest an intent to create the fiduciary
relationship known by the law as a trust. The focus is on function rather than form.
 A Trust Will Not Fail for Want of a Trustee: A trust may have one trustee or several. A trustee may be an
individual or a corporation. A trustee may be the settlor, a beneficiary, or a third party. In the absence of a
trustee, the court appoint one (usually the executor in the case of a testamentary trust)
 Precatory Language: If the language indicates merely a moral obligation (hope or wish) rather than intent to
create a legal obligation, it is called precatory, and does not create an enforceable trust.
a. Testamentary A. Testamentary Trust: A testamentary trust is created by will. Discerning whether a testator intended to create a trust
Trust involves first looking to the words in the will and then construing the will. If the testator’s intent is not stated clearly, it
must be inferred from the language and structure of the will in light of all the circumstances.
 In a well-drafted will, the testator’s intent to create a trust is stated clearly.
o Example: All the residue of my estate I give to _______ as trustee, to be held in trust and disposed
of as follows: __________.
 Example: Lux v. Lux
b. Inter vivos Trust A. Inter vivos Trust: If the trust in question is an inter vivos trust, the words in question will either be in the
declaration of trust (if the settlor is the trustee) or the deed of trust (if someone other than the settlor is the trustee). An
inter vivos trust requires that both the expression of the intent to create a trust and the funding occur while the settlor is
alive. {CT}

44/87
1. Deed of Trust: If the trustee is a third party, the expression of the intent to create a trust is called a deed of trust
(whether oral or written). {CT} A deed of trust does not fund a trust. No particular formalities are required to create an
inter vivos trust of personal property (unlike a testamentary trust—Wills Act—and an inter vivos trust of land—Statute
of Frauds). If the court determines that a transfer of property by O to T was intended to be in trust for the benefit of A, T
is subject to fiduciary duties to A.
 Creating a Trust: Where donors do not expressly direct a third party to hold a gift “in trust,” this is not
essential to create a trust relationship. It is enough if the transfer of the property is made with the intent to
vest the beneficial ownership in a third person. Jimenez.
B. Declaration of Trust: If the trustee is the settlor, the expression of the intent to create a trust is called a declaration
of trust. Under a declaration of trust, the settlor simply declares himself to be a trustee of certain property. The settlor
may also be a beneficiary of the trust.
 Same Party Can Wear All Three Hats—Need Duties to Someone Other Than Self: Although there are
three distinct parties to a trust—settlor, trustee, and beneficiary—the same person can wear all three hats at
the same time as long as there is another trustee or another beneficiary. {CT} To have a valid trust, the trustee
must owe fiduciary duties to someone other than herself.
o Merger: If O is the sole trustee and also the sole beneficiary, the equitable and legal titles merge,
leaving O with absolute legal title, as under UTC § 402(a)(5).
 Requirements of Declaration of Trust: A declaration of trust of personal property requires no particular
formalities. It does not require delivery of an instrument of transfer. The settlor need only manifest an
intention to hold certain of the settlor’s property in trust for an ascertainable beneficiary.
C. Failed Gift: Where a donor has the intent to make a gratuitous inter vivos gift, but the gift fails for want of delivery
(typically the donor dies before making the delivery), the donee may try to save the failed gift by recharacterizing the
donor’s intent as the intent to create a trust. The donee will argue that when the donor expressed the intent to create a
trust, the donor also appointed herself trustee, thereby, “delivering” the property from herself as settlor to herself as
trustee. Because a trust will not fail for want of a trustee, the donee will then ask the court to appoint a successor trustee
and instruct and instruct him or her to transfer the property to the donee, thereby satisfying the delivery requirement for
the gift, but only be recharacterizing the failed inter vivos gift as an intent to create a trust. {CT}
 Inter vivos gift: A valid inter vivos gift requires (1) present donative intent; (2) delivery; and (3) acceptance.
{McL} Acceptance is generally presumed but can be rebutted on a showing that the gift would pose onerous
burdens on the acceptee.
o Delivery: In contrary to a declaration of trust, an outright gift requires the donor to deliver the
property to the donee. Delivery can be constructive (i.e., a key) or symbolic (i.e., certificate of title),
rather than actual, but delivery of some kind is required. Intention alone is not enough to perfect the
gift. Consequently, if a donor manifests an intention to make a gift but fails to complete delivery,
the manifestation might be recharacterized as a trust.
 Constructive Deliver: A constructive delivery gives the donee the means of obtaining
the property, such as a key.
 Symbolic Delivery: A symbolic delivery something symbolic of the object, for example a
written instrument handed over when manual delivery is impracticable.
 Inter vivos gift vs. Declaration of Trust: A gift which is imperfect for lack of a delivery will not be turned
into a declaration of trust for no better reason than that it is imperfect for lack of a delivery. Courts do not
supply conveyances where there are none. This is true, even though the intended donee is a charity. Hebrew
University I.
o *The difference between an inter vivos gift and a declaration of trust is closer than Hebrew
University held. {McL} thinks that a declaration of trust could have been found in that case.
 One can orally constitute himself a trustee of personal property for the benefit of another and thereby create a
trust enforceable in equity, even though without consideration and without delivery. However, he must in
effect constitute himself a trustee. There must be an express trust, even though oral. It is not sufficient that he
declare himself a donor. While he need not use the term “trustee,” nor even manifest an understanding of its
technical meaning or the technical meaning of the term “trust,” he must manifest an intention to impose upon

45/87
himself enforceable duties of a trust nature. Hebrew University I.
 Restatement: A gift of personal property can be perfected on the basis of donative intent alone if the donor’s
intent to make a gift is established by clear and convincing evidence. RS3d Prop. § 6.2. A failure to satisfy the
formality of delivery can be excused as harmless error if there is clear and convincing evidence of donative
intent.
Lux v. Lux (RI): Paragraph 2 of Lux’s will left the residue of her estate to her
grandchildren. Paragraph 3 provided that her real estate “shall be maintained” for the
benefit of the grandchildren and “shall not be sold” until the youngest reached age 21.
The court ruled that the use of the phrases “shall be maintained” and “shall not be sold,”
when coupled with the surrounding circumstances surrounding the testator and her
grandchildren, indicated the intent to create a trust. Although the will failed to name a
trustee, the court appointed the executor of the estate trustee.
Jimenez v. Lee (OR): Betsy Lee’s paternal grandmother and Mrs. Diercks, a client of
Betsy’s father, Jason Lee, both made separate gifts for Betsy’s educational benefit.
Jason invested the proceeds of both gifts in bank stock and took title thereto as
“Custodian under the Laws of Oregon” his children. On appeal, the court held that
the intent of the donors was enough to vest beneficial ownership in Betsy, thereby
creating a trust. The court reasoned that Jason, an attorney, had referred to the
arrangement as a trust in a letter to his mother and had testified that the gifts were
for Betsy’s education’s needs. The court found that Jason breached his duty to the
Betsy to administer the trust solely in the interest of her as beneficiary where he
never provided any accounting and used some of the money for purposes other than
Betsy’s educational needs (i.e., a trip to Canada and “expensive” clothes).
 Lee wanted to be a custodian, not a trustee, because he would be subject
to less restriction—he could use the money for general support (not just
education) and he wouldn’t need to account beyond year two of the minor
reaching the age of majority because of the statute of limitations.
Hebrew University I (CT): Ethel Yahud (decedent) purchased a large library from
her husband’s estate, a professor. During her lifetime, she told Hebrew University
(plaintiff) that she was making a gift of the library to it and even went so far as to attend a luncheon in her honor and make a formal
announcement of her gift. However, she never actually tendered possession of the library to the university, although she did prepare it for
shipping. Upon her death, Ethel bequeathed the bulk of her estate to a different charity (defendant). The university was not mentioned in her
will. On appeal, the court held that the trial court’s judgment declaring that the university was the owner of the library was without support in
the findings and could not stand. The court held that, at most, Ethel the requisite donative intent such that, had she subsequently made a delivery
of the property while that intent persisted, there would have been a valid, legal gift inter vivos. The court also held that there were no facts to
indicate that Ethel imposed upon herself any duties of a trust nature. The court remanded for a new trial on other theories.
Hebrew University II (CT): Ethel Yahud (decedent) purchased a large library from her husband’s estate, a professor. During her lifetime, she
told Hebrew University (plaintiff) that she was making a gift of the library to it and even went so far as to attend a luncheon in her honor and
make a formal announcement of her gift. At the luncheon, she gave the university a memorandum containing a list of most of the contents of
the library and of all of the important books, documents, etc. However, she never
actually tendered possession of the library to the university, although she did prepare
it for shipping. Upon her death, Ethel bequeathed the bulk of her estate to a different
charity (defendant). The university was not mentioned in her will. The court held that
the delivery of a memorandum, coupled with the Ethel’s acts and declarations
clearly showing an intention to give and to divest herself of any ownership of the
library, were sufficient to complete the gift.

46/87
2. Trust Property (or A. Trust Property Required for a Valid Trust: The second requirement for a valid trust is that the trust must be
corpus or res) funded—some property must be transferred12 to the trustee. (1) A trust is not created until some property is transferred
to the trust (to the trustee), and only property transferred to the trust is part of the trust. (2) The property transferred can
be any interest that is transferable (i.e., real property, personal property, money (even as little as a dollar or a penny),
leasehold interests, possessory estates, future interests (even contingent remainders), life insurance policies, choses in
action, royalties, animals). {CT}
 Future Profits and Other Expectancies: An expectation or hope of receiving property in the future, or an
interest that has not come into existence or has ceased to exist, cannot be held in trust.
1. Exception for Revocable Trust with a Pour-Over Will: In the case of revocable trust connected with a pour-over
will, many state statutes do not require property to be in the trust at the time of the execution of the trust.
B. Contrast Failed Gift: Where a donor promises to make a gift of a particular asset but then dies before delivering it,
the frustrated donee may try to save the gift my recharacterizing the promise as a declaration of trust that also
constituted funding and then ask the court to appoint a successor trustee and order delivery to complete the gift.
 Example: Unthank (“The declarant must express an intent to declare a trust over specific property”)
Unthank v. Rippstein (TX): Shortly before his death, Craft wrote a letter to Rippstein
donee, in which he acknowledged that he would send her monthly payments of $200.00 for
a period of five years, even in the event of his death. Specifically, he wrote, “I . . . hereby
and herewith bind my estate to make the $200.00 monthly payments.” On appeal, the court
determined that the writing was insufficient to establish a voluntary trust, because a
voluntary trust would have divested Craft of the exercise of further dominion and because
Craft did not expressly declare that all of his property, or any specific portion of the assets
which he owned, would constitute the res of a trust for the benefit of Rippstein. The court
held that the writing was merely a promise to make a series of gifts in the future and was
therefore unenforceable.

3. Ascertainable A. Ascertainable Beneficiaries—The Beneficiary Principle: A private13 trust must have one or more ascertainable
Beneficiaries beneficiaries to whom the trustee owes fiduciary duties and who can call the trustee to account. See UTC § 402(a)(3)
(2000). The beneficiaries need not be ascertained when the trust is created—only ascertainable (i.e., the unborn children
of O are ascertainable—the court will appoint a guardian ad litem). If at the time the trust becomes effective the
beneficiaries are too indefinite to be ascertainable, the attempted trust will fail for want of an ascertainable beneficiary.

 Not Ascertainable: “Friends” (see Clark).

 Ascertainable: Relatives, heirs, next-of-kin, siblings, children, issue, nephews, nieces, relations, unborn
children)
B. Exception—Public Trusts & Charitable Trusts: The beneficiary principle applies to private but not to public trusts
and charities. Clark. Charities cannot have ascertainable beneficiaries. {CT}
C. Exception—Honorary Trusts: See Below
D. Power of Appointment: See Clark
 Traditional Rule: If there is a transfer in trust for members of an indefinite class of persons, no enforceable
trust is created, but the transferee has a discretionary power to convey the property to such members of the
class as he may select. The power of appointment is discretionary; it is a nonfiduciary power. It cannot be
given to a trustee. See Clark.
 Modern Rule: A power in a trustee to select a beneficiary from an indefinite class is valid. If the power is not
exercised within a reasonable time, the power fails and the property subject to the power passes to the persons
who would have taken the property had the power not been conferred. UPC § 402(c).

12 Physical transfer not necessary


13 If charitable trust, no need for ascertainable beneficiaries because the AG enforces the trust
47/87
a. Pet & Other A. Modern Trend: The trend in the cases, codified by the UTC, has been toward allowing enforceable trust for pet
Noncharitable animals (§ 408) and certain other noncharitable purposes (§ 409).
Purpose Trusts 1. Accommodating Trusts for Pets and Other Noncharitable Purposes.
 A pet animal, being property, cannot be the beneficiary of a trust because it cannot bring suit to enforce the
trustee’s fiduciary duties.
 A trust for the care of a particular animal is not a charitable trust because it is not a valid charitable
purpose.
 People want noncharitable trusts for (1) the care of pets; (2) the maintenance of graves; and (3) the saying of
masses
 To accommodate the desire for trusts for noncharitable purposes and for pet animals in particular, two
adaptations have evolved: (a) common law honorary trusts, and (b) statutory pet and other noncharitable
purpose trusts.
a. Honorary Trusts: In an honorary trust, the transferee is not under a legal obligation to carry out the settlor’s stated
purpose (hence the qualifier honorary), but if the transferee declines or neglects to do so, she holds the property upon a
resulting trust and the property reverts to the settlor or the settlor’s successors {McL} See In re Searight’s Estate.
 Rule Against Perpetuities: In drafting an honorary trust, care must be taken not to violate the Rule Against
Perpetuities. Under the common law Rule, an honorary trust for a noncharitable purpose is void if it can last
beyond all relevant lives in being at the creation of the trust plus 21 years. If the trust is for a pet, the pet is
not a validating life.
o Searight is inconsistent with orthodox understanding of the common law Rule Against Perpetuities.
b. Uniform Acts & Other Statutes: Almost every state has enacted legislation that permits a trust for a pet animal for
the life of the animal and often other noncharitable purposes such as perpetual maintenance of a grave.
 UTC & UPC Approach: A trust for a pet animal or certain other noncharitable purposes is valid, but the
court is authorized to reduce the amount of the trust property if it is excessive. See UTC §§ 408–409 (2000)
and UPC § 2-907 (1990, rev. 1993). The UPC and UTC authorize a person named by the settlor or the court
to enforce the trustee’s fiduciary duties.
 Utah: A trust for the care of a designated domestic or pet animal is valid. Intended use of Principal and
Interest can be enforced by an individual in the trust instrument or appointed by a court. A court may reduce
amount of property transferred if it determines the amount substantially exceeds the amount required for the
intended use. If no trustee [caretaker] is designated or no designated trustee is will or able to serve, a court
shall name a trustee.
B. Some Options for Pets After Owners Die
 Precatory Request: Generous gift in Will or Trust Agreement outright to a trusted beneficiary with a clearly
precatory request that the beneficiary use as much of the funds as are needed to care for the pet for its
remaining lifetime (“I wish but do not legally require . . .”). {McL}
 Honorary Trust; Statutory Trust; Pet Life Care Center {McL}
Clark v. Campbell (NH): The testator died and under the terms of her will bequeathed his tangible personal property (TPP) and collectibles to
her trustees for the benefit of her “friends.” Her “friends” as purported heirs, commenced suit against the state and sought to recover under the
will. The court held that: (1) the bequest was invalid because it was a bequest to an indefinite person; (2) the term “friends” was too indefinite
because it had no accepted statutory or controlling limitation and was not precise at all; and (3) the bequest was not a power of appointment
because the power was given to the trustees whose powers over the trust property necessarily were held in a fiduciary capacity.
In re Searight’s Estate (OH): Searight (decedent) bequeathed his dog and the sum of $ 1,000 for the care of his dog, which was to be paid to
the dog’s caretaker at the rate of 75 cents per day. The court held that—whether called an “honorary trust” or a gift with a power—the bequest
for the care of the dog was valid because (1) the person to whom Searight made the bequest was willing to carry out Searight’s wishes and (2)
the bequest did not violate the rule against perpetuities—at 75 cents per day the money would run out without 5 years.
4. Written Instrument A. Generally—No Writing Requirement: The law of trusts, standing alone, does not require a writing to create a
(Maybe) valid trust. An oral inter vivos trust of personal property, whether by declaration or by transfer to another as trustee, is
enforceable.
 Exception for Testamentary Trusts & Trust of Land: However, a testamentary trust must be in writing to
satisfy the Wills Act, and an inter vivos trust of land must be in writing to satisfy the Statute of Frauds.

48/87
a. Oral Inter Vivos A. Oral Inter vivos Trust & Personal Property: As a general rule, inter vivos trusts do not have to be in writing,
Trusts of Personal unless the trust holds real property. {CT}
Property 1. Versus Testamentary Trust: A trust where the trustee is to deliver personal property at the settlor’s death, if funded
inter vivos, is an inter vivos trust and does not have to be in writing. {CT}
 Example: In re Estate of Fournier.
2. UTC § 407 (2000). Evidence of Oral Trust. Except as required by a statute other than this [Code], a trust need not
be evidenced by a trust instrument, but the creation of an oral trust and its terms may be established only by clear and
convincing evidence.
 Oral Testimony as Proof: An oral trust that provides for the disposition of personal property at the death of
the settlor may be proved by oral testimony. Fournier.
o Testimony probably should be by a disinterested party like an attorney for it to constitute clear and
convincing evidence. {McL}
In re Estate of Fournier (ME): George Fournier (settlor), the late brother
of Faustina (prospective beneficiary), gave money to a couple (Madores)
and told them that the money was to be held by them and given to Faustina
upon his death. After he died, the couple gave the money to her. Fournier’s
other sister, Juanita, claimed that Fournier meant that Faustina was to take
the money as the estate’s personal representative. On appeal, the court
found that there was clear and convincing evidence that Fournier had
created an oral trust for Faustina’s benefit, and thus, the money passed to
her as a beneficiary. The evidence consisted of the oral testimony of the
couple.

7. Nonprobate Transfers
A. Introduction: Will substitutes are modes of transfer that operate outside of probate and include (1) revocable inter vivos trusts, (2) life
insurance; (3) various types of pay-on-death (POD) bank accounts; (4) transfer-on-death (TOD) security accounts; and (5) pension plans and
retirement accounts.
 Many lawyers recommend creating a revocable trust and making it the beneficiary of the client’s will and various will substitutes. The
revocable trust has thus emerged as the successor to the will as the centerpiece instrument in contemporary estate planning.
A. The Rise of Nonprobate Succession
1. Legal Questions Raised by Nonprobate Succession: The rise of nonprobate succession raises two questions: (1) Whether the Wills Act
formalities should be required of will substitutes and (2) Whether the subsidiary law of wills14 should apply also to will substitutes. {McL}
 Wills Act Formalities—Modern View: A will substitute is valid even if not executed with Wills Act formalities—it need only to be in
writing and signed by testator.
 Subsidiary Law—Modern Trend: Courts and legislatures have increasingly subjected will substitutes to the subsidiary law of wills,
with one exception: Most pensions and life insurance policies obtained as a benefit of employment are governed by federal law, which
preempts state law.
2. Probate Procedure, Estate Planning Practice, and Nonprobate Succession: (1) Successful experience with nonprobate transfers unsupervised
by a court has prompted reforms to probate process. (2) The proliferation of asset-specific will substitutes and the dominance of nonprobate
succession complicate the creation of a unified and coherent estate plan—the combined use of a revocable inter vivos trust and pour-over will has
become the method of choice for solving this problem. {McL}
B. Revocable Trusts
 Most flexible of will substitutes—the settlor can draft the provisions precisely to her liking. {McL}
 Most “will-like” of will-substitutes—not inherently asset-specific; settlor remains free to amend or revoke the trust at any time and for
any reason. {McL}
 Attestation not required
1. The Wills Act A. Revocable Deed of Trust: A revocable trust may be created by a deed of trust whereby the settlor transfers to the
trustee the property to be held in trust. On the settlor’s death, the trust property is then distributed or held in further trust
in accordance with the terms of the trust.

14 Subsidiary law of wills: Policy-based substantive limits on testation by will, such as creditors’ rights and the forced share for a surviving
spouse, and rules of construction, such as antilapse, simultaneous death, and revocation on divorce.
49/87
 All states allow a revocable trust created by deed of trust without Wills Act formalities to effect a nonprobate
transfer on death.
B. Revocable Declaration of Trust: A revocable trust may be created by a declaration of trust: the settlor simply
declares herself to be the trustee of certain property for his own benefit during his life, with the remainder to pass at his
death in accordance with the terms of his declaration. The settlor retains the power to revoke the trust and, as trustee,
controls the management of the trust property.
 Attestation is not required. {McL}
C. Classic Irrevocable Deed of Trust: Assuming the settlor, the trustee, and beneficiaries are all different parties, and
assuming the trust is irrevocable, the settlor must transfer the property to the trust inter vivos. Once the settlor transfers
the property to the trust, the settlor no longer has any interest in the property. The trust, not the settlor, holds legal title
to the property. When the settlor dies, legal title does not have to be transferred because the trust continues to hold the
legal title. {CT}
D. Irrevocable Declaration of Trust & Settlor as Life Beneficiary: If the settlor is trustee and life beneficiary, the
trust is irrevocable, and the remainder interest has been conveyed to another beneficiary, the future interest (a vested
remainder, typically) passes inter vivos to the beneficiary holding the future interest. Because the trust holds legal title,
there is no need to transfer title upon the settlor/life beneficiary’s death. {CT}
2. Beneficiaries Have A. Present Interest—UTC Approach/Modern Trend: “While a trust is revocable [and the settlor has capacity to
No Present Interest revoke the trust15], rights of the beneficiaries are subject to the control of, and the duties of the trustee are owed
exclusively to, the settlor. UTC § 603(a).” {McL}
o Rationale: Treat revocable trust the same as will; Disallow angry kids from suing settlors on
ground of incapacity
 Capacity: When the settlor becomes incapacitated, the beneficiaries gain a present interest and can sue.
Many states do not include the capacity language because (1) it might discourage people from creating
revocable trusts out of fear that a prospective beneficiary might argue a settlor does not have capacity and (2)
allowing beneficiaries to sue the settlor for breach of fiduciary duties is contrary to rules surrounding wills.
o Utah has adopted the bracket language
B. No Present Interest—Case law:16 Beneficiaries under a revocable trust have no legally enforceable interest in the
trust as long as it is revocable. Accordingly, a trustee owes the beneficiary no fiduciary duties, and a beneficiary has no
standing to contest any actions undertaken by the settlor (such as amendment to the trust or transactions involving the
trust property) prior to the settlor’s death. Furthermore, the beneficiary has no right to see the trust or to receive an
accounting for the trust. {CT} Moon.
 No Notice Requirement: When the trustee is also the settlor of the revocable trust, the settlor is not required
to serve written notice on himself of revocation. Moon.
C. Post-Death Rights: After the death of the settlor, the beneficiaries of a revocable trust have standing to challenge
pre-death withdrawals from the trust by a third party which are outside of the purposes authorized by the
trust and which were not approved or ratified by the settlor personally or through a method contemplated through the
trust instrument (made without the settlor’s knowledge or consent). See Siegel.
Moon v. Lesikar (TX): Carolyn and Woody’s father,
Woodrow established a revocable family trust, naming
himself and Woody co-trustees. Woodrow placed 10,000
shares of stock in an airport corporation in the trust. Later,
Woodrow decided to transfer the stock to a trust (S&S Trust)
established for Woody’s children. Then, Woodrow signed a
new trust agreement (Amended Family Trust) that modified,
amended and superseded the family trust agreement and any
modifications and amendments previous to the date of the
signing of the new agreement. The Amended Family Trust did not mention the distribution of the airport stock to Woody as the original trust
had. It was to become irrevocable upon Woodrow’s death. Carolyn was a beneficiary of the residuary. Woodrow sold the airport stock to Woody
for $ 2,000 for the benefit of the S&S Trust. After Woodrow’s death (the AFT became irrevocable), Carolyn complained that the sale was for an
inadequate price. The court concluded that Woodrow showed his intent to revoke the family trust with respect to the airport stock by his
conveyance of that stock to the S&S trust. Because Woodrow was the settlor of the family trust with the power to revoke, the sole beneficiary
while alive, and co-trustee, Carolyn had no standing to complain of Woodrow’s disposition of family trust assets.
3. Revoking or A. Presumption of Irrevocability—Traditional Law: An inter vivos trust created by a written instrument was
Amending a Revocable presumed to be irrevocable unless there was an express or implied reservation by the settlor of a power to revoke the

15 The bracketed language is optional


16 Same as UTC approach
50/87
Trust trust. {McL}
 Revoking/Amending a Trust: To amend or revoke a trust, the settlor had to strictly comply with the method
specified for amendment or revocation specified in the trust instrument. {McL}
B. Modern Majority Rule: “Unless the terms of a trust expressly provide that the trust is irrevocable, the settlor may
revoke or amend the trust.” UTC § 602(a). 17 In other words, an inter vivos trust is revocable unless declared to be
irrevocable. “The settlor may revoke or amend a revocable trust: (1) by substantial compliance with a method provided
in the terms of the trust; or (2) if the terms of the trust do not provide a method or the method provided in the terms is
not expressly made exclusive, by: (A) a later will or codicil that expressly refers to the trust or specifically devises
property that would otherwise have passed according to the terms of the trust; (B) any other method manifesting clear
and convincing evidence of the settlor’s intent.” UTC § 602(c). {McL}
1. Revocation—Particular Method Expressed: The UTC provides that where the trust sets forth a particular method
of revocation, it should not be construed as the exclusive method unless the trust provision expressly makes it
exclusive. UTC § 602(c)(2). Substantial compliance with the particular method is sufficient. UTC § 602(c)(1). {CT}
 Example: Patterson v. Patterson (“Like a will, over which the testator has absolute control until the testator’s
death, [UTC § 602] provides the settlor of a revocable trust with complete control over the trust until the
settlor’s death.”)
2. Revocation by Will/Codicil: The UTC provides that a will or codicil executed after the trust, by a later will or
codicil, which (a) expressly refers to the trust or (b) specifically devises property that would otherwise have passed
according to the terms of the trust, IF the trust terms do not specify an exclusive method of revocation and if the will is
not thereafter revoked. UTC § 602(c)(1). {CT}
3. Revocation—No Particular Method Expressed: If the trust is revocable but silent as to the method of revocation or
does no set forth an exclusive method, revocation can be accomplished by any method manifesting clear and
convincing18 evidence of the settlor’s intent. UTC § 602(c)(2)(b).
 Revocation of Revocable Trust by Physical Act/Written Document/Oral Statement: While revocation of
a trust will ordinarily continue to be accomplished by signing and delivering a written document 19 to the
trustee, other methods, such as a physical act or an oral statement coupled with a withdrawal of the property,
might also demonstrate the necessary intent. These less formal methods, because they provide less reliable
indicia of intent, will often be insufficient, however. The method specified in the terms of the trust is a
reliable safe harbor and should be followed whenever possible. UTC § 602 cmt.
C. Revocation Without Trustee’s Knowledge: “A trustee who does not know that a trust has been revoked or
amended is not liable to the settlor or settlor’s successors in interest for distributions made and other actions taken on
the assumption that the trust had not been amended or revoked.” UTC § 602(g). {McL}
 Solution for Trustee: Require delivery of revocation to trustee as exclusive method of revocation using UTC
§ 602(c)(2). {McL}
Patterson v. Patterson (UT): Darlene Patterson created a revocable trust. The document stated that “revocation or amendment . . . may be in
whole or in party by written instrument . . . delivered in writing to the then acting Trustee.” The trust also provided that the “interests of the
beneficiaries are presently vested interests subject to divestment which shall continue until this Trust is revoked or terminated other than by
death.” Shortly before she passed away, Darlene executed an amendment to the trust to remove Ron (one of her sons) as beneficiary of the trust
because “I have already properly provided for this son.” Ron sought a declaration that the amendment was void because it violated the terms of
the trust. The court held that the amendment was valid under UTC § 602(c)(2)(B) (even though she did not deliver the amendment to the
trustee) because the terms of the trust did not provide an exclusive method of amendment or revocation and the Amendment qualified as a clear
expression of Darlene’s intent to terminate Ron’s interest as a beneficiary. Darlene modified that part of the trust that named Ron as a
beneficiary and did so in a manner that provided clear and convincing evidence of her intent to eliminate the Ron’s interest (signed writing,
gave a reason for the Amendment in the amendment) in the trust. Thus, the
amendment was valid. The court overruled the cases that required a settlor to
revoke the entire trust in order to revoke the trust.

17 Duke, at 452.
18 The trust can be revoked by writing (even if the writing does not qualify as a will), by act (destructive act coupled with intent), by
presumption (arguably), and even orally (unless real property is involved). {CT}
19 Even if the writing does not qualify as a will
51/87
4. The Subsidiary Law A. The Subsidiary Law of Wills—Modern Trend: “A will substitute is . . . to the extent appropriate, subject to
of Wills substantive restrictions on testation [e.g., surviving spouse and creditor protections] and to rules of construction and
other rules applicable to testamentary dispositions [e.g., simultaneous death, slayer, revocation on divorce, anti-lapse
statutes]. See R3d Prop.: Wills, § 7.2. cmt. a. {McL}. In other words, the policy limits on testation by will (i.e., rules
that protect creditor and spouses) and the rules of construction for will (i.e., simultaneous death, revocation on divorce)
generally apply also to a revocable trust.
1. Creditors & Revocable Trusts
 Traditional Law: A creditor of the settlor had no recourse against property in the settlor’s revocable trust
unless the settlor was also a beneficiary of the trust. The settlor’s power to revoke and take back the property
was not considered equivalent to ownership of the property. See RS2d Trusts § 330 cmt. o (1959).
 Modern Rule: The settlor’s power to revoke the trust and take back the trust property is regarded as
equivalent to ownership and, hence, the trust property is subject to the claims of the settlor’s creditors during
life and at death. See UTC § 505(a)(3).
o A settlor’s creditors can reach assets of a revocable trust during the settlor’s life. Reiser. After
the settlor’s death, creditors can reach those assets of a revocable trust over which the settlor had
such control at the time of his death as would have enabled the settlor to use the trust assets for his
own benefit. Reiser.
o Rationale: A revocable trust is the same as a will in this regard because the settlor retains total
control during lifetime; bad public policy to allow people to escape debts; assets are still taxable;
settlor does not depart with dominion and control
a. Creditors & Other Nonprobate Transfers
o Joint Tenant: The creditors of a joint tenant cannot reach the jointly held property after the joint
tenant’s death.
o Life Insurance: Life insurance proceeds are usually exempt from the insured’s creditors if payable
to a spouse or child.
o Retirement Benefits: Retirement benefits are usually exempt from creditors.
o Savings Bonds/POD: U.S. savings bonds with a POD beneficiary may be exempt from creditors.
 UPC § 6-102 (1998): The UPC permits the decedent’s creditors to reach nonprobate transfers (except joint
tenancies in real estate), such as revocable inter vivos trusts and joint bank accounts, if the probate estate is
insufficient to pay the debts.
2. Spousal Rights & Revocable Trusts:
 Applying Wills Divorce Statutes to Testamentary Trusts: When a settlor creates a revocable pour-over
trust to be funded entirely at the time of the settlor’s death according to a pour-over will and names a spouse
as a beneficiary, the spouse’s beneficiary interest is revoked if the couple gets divorced before the settlor’s
death. Clymer.
o Rephrased: All jurisdictions have statutes providing that upon divorce, all provisions in each
spouse’s will in favor of the ex-spouse are automatically revoked by operation of law. Most statutes
apply only to the ex-spouse’s will and not to any will substitutes. Because a testamentary trust is
considered part of the will, the doctrine of revocation applies to testamentary trusts. Because inter
vivos trusts are will substitutes the statutes do not apply to the inter vivos trust. {CT}
 UPC Approach: Under the UPC, divorce revokes a disposition in favor of a former spouse in a governing
instrument, which is defined to mean a deed, will, trust, insurance or annuity policy, account with a pay-on-
death designation, pension plan, or other such nonprobate transfer. See UPC § 2-804 (1990, rev. 1997).
o Retroactive Application: Most courts have upheld retroactive application of revocation-on-divorce
statutes, but a few have not.
o Pension Accounts/Life Insurance: Federal law preempts the application of state revocation-on-
divorce statutes to most pension accounts and life insurance obtained as a benefit of employment.
Egelhoff.
C. Capacity for Revocable Trusts—Modern Rule: The capacity required to create, amend, revoke, or add property to
a revocable trust, or to direct the actions of the trustee of a revocable trust, is the same as that required to make a will.
See UPC § 601 (2000).
D. Limitations: The UPC and UTC provide a three-year limitations period in which to bring a challenge against a will
or a revocable trust. See UPC § 2-108(a); UTC § 604(a)(1).

52/87
State Street Bank & Trust v. Reiser (MA): Wilford Dunnebier (settlor) transferred capital stock to an inter vivos trust prior to obtaining an
unsecured personal loan from State Street Bank. Wilford also executed a will under which he left his residuary clause to the trust he had
established. The probate court found that such was done without fraudulent intent. Wilford died before repaying the loan, and his probate assets
were insufficient to repay the balance owed. In creating the trust, Wilford retained the power to amend or revoke and the right to direct the
disposition of principal and income. The court held that because Wilford had retained such authority during life that the bank could reach the
trust assets to pay the loan after death to the extent that Wilford’s estate was
insufficient to satisfy the loan. The bank’s access to trust assets was limited to those
assets over which the settlor had retained control during the settlor’s lifetime.
Clymer v. Mayo (MA): At the time she executed her will and a revocable pour-over
trust, Mayo (decedent) and her ex-husband were still married. After the divorce,
Mayo left her ex-husband as beneficiary of the trust. The trust was not funded until
Mayo’s death. The court held that the ex-husband’s beneficial interest in the trust
was revoked by operation of a Massachusetts statute providing that divorce revokes
an ex-spouse’s interests under a will because the trust was coordinated with the will to fashion a unified estate plan that would not take effect
until Mayo’s death (even though the statute technically as written only applied
to wills).

5. Revocable Trusts in A. Introduction: The inter vivos trust and pour-over will combination is the most common estate-planning scheme
Contemporary today. The revocable trust has the principal advantage of avoiding probate but only as to those assets placed in the trust
Practice inter vivos. Unlike a will, which automatically reaches out and applies to all of the decedent’s probate assets at death,
an inter vivos trust applies only to those assets transferred to the trust inter vivos during the settlor’s lifetime. Because
people acquire and dispose of assets on a daily basis, it is virtually impossible to put all of one’s assets in an inter vivos
trust, even if one is so inclined. Therefore, most settlors put their larger assets, and those assets that they do not use on a
regular basis, in the trust. They use a pour-over will for their other assets and for their newly acquired assets that they
did not have time to put in the trust before they died. {CT}

Advantages of Revocable Trust:


 Avoids probate (including ancillary admin)
 Management of assets in event of settlor incapacity
 Continuity in property management at death
 Uniform/consolidated disposition of assets.

53/87
 Easier to amend than will
 Less court supervision over trusts
 Privacy
 More difficult to contest than a will
 Choose state law to govern w/r/t personal property
a. The Pour-Over A. Pour-Over Will: A pour-over will is a will that contains an express clause giving some or all of the decedent’s
Will probate property to the trustee of the decedent’s inter vivos trust, to hold and distribute pursuant to the terms of the
trust. Typically, the pour-over clause is the residuary clause, but it need not be. The clause can transfer to the trust a
specific or general gift, but the norm is either the residuary clause of a general gift of money. {CT}
 Example: O sets up a revocable trust with himself or a third party as trustee. O then executes a will devising
his probate estate to the trustee of that trust.
o “I give the rest, residue, and remainder of my estate to the trustee of my inter vivos trust, to hold
and distribute pursuant to its terms.”
B. Common-Law: Validity of Pour-Over Will: The effect of a pour-over clause is that a document that was not
executed with Wills Act formalities (the trust instrument) controls who takes some or all of the decedent’s probate
property. Despite that, there are two possible doctrines that are used to try to validate a pour-over clause: (1)
incorporation by reference and (2) acts of independent significance. {CT} Neither permits a pour over into an unfunded
revocable trust drafted or amended after execution of the will. To do this required statutory authorization.
1. Incorporation: Incorporation requires that the trust instrument be in existence at the time the will is executed.
 Trust Instrument Amendments: The doctrine of incorporation gives effect to the trust instrument as it
existed at the time the will was executed. Any subsequent amendment to the inter vivos trust instrument
cannot be incorporated and given effect unless (1) the will is re-executed or (2) a codicil is executed that
redates the will under republication by codicil. {CT}
 Funding: Incorporation by reference incorporates the trust instrument, not the trust, so there does not have to
be any funding in the trust.
2. Acts of Independent Significance: Independent significance requires that the trust have some property transferred to
it during the settlor’s life.
 Timing: It does not matter when the inter vivos trust is created (before or after execution of the will) as long
as the trust was created inter vivos (that some property is placed in the trust inter vivos) and has property in it
at the time of the decedent’s death.
 Trust Amendments: Amendments to the trust are valid and can be given effect, regardless of when they
happen.
b. Statutory  The current form of the UPC and UTATA permits the trust instrument to be executed or amended after the
Validation for Pour- will. See UPC § 2-511(b) (Duke, at 464).
Over Wills &  Benefits: As long as the transferor meets the UTATA requirements (1) the transferor does not have to put a
Unfunded penny into his or her trust inter vivos; (2) the pour-over clause is valid; (3) all amendments to the trust are
Revocable Trusts valid even if executed after the date of the will; and (4) after the probate property is poured over to the trust
pursuant to the pour-over clause, the trust is treated as an inter vivos trust that is not subject to probate court.
c. Advantages of A. Lifetime Consequences: The revocable trust has lifetime consequences that figure into contemporary estate
Revocable Trust planning.
 Property Management by a Fiduciary: A settlor may appoint a professional trustee. This option is
particularly attractive if (1) the settlor anticipates impending incapacity (creating an inter vivos trust is
cheaper and easier than initiating guardianship or conservatorship proceedings), or (2) the settlor wants to
transfer property to minors who lack capacity to hold title. Putting property in trust, however, increases costs
(trustee’s fees) and administrative expenses in dealing with the assets (increased paperwork). {McL}
 Segregating Assets: A revocable trust may be useful in keeping title clear, such as when a newly married
person wants to avoid commingling separate property with property acquired during the marriage.
 Easier to Amend Than a Will: To change a trust, the settlor does not have to follow the Wills Act
Formalities. {McL}
 Taxes: There are no federal income, gift, or estate tax benefits to the creation of a revocable trust.
B. After Death
 Probate Avoidance: The principal benefit of using a revocable, inter vivos trust is that the property
transferred to the trust inter vivos avoids probate.
o Costs: Avoiding probate saves probate court costs, executor’s fees, and attorney fees, but offsetting
these future savings are the present fees inherent in having the trust drafted, in transferring assets to
the trust, and possibly in paying trustee’s fees to administer the trust. {CT}
o Tying Property Up: Property placed in an inter vivos trust avoids probate, thereby avoiding the
tying up of the property in probate. Putting property in trust, however, does increase the
administrative difficulties of dealing with the property. {CT}
 Creditor’s Claims: If one anticipates a number of creditors’ claims against a decedent’s property, it may be
better to have the decedent’s property pass through probate. The probate process (nonclaim statutes) requires

54/87
creditors to bring their claims within a fairly limited time frame or be barred forever. In contrast, if a
decedent’s property is in a revocable trust, there is no shortened time period within which creditors have to
bring their claims; creditors get the full benefit of the standard statute of limitations to bring their claims.
{CT}
 Privacy: When a decedent dies, an inventory of the decedent’s probate asset’s and creditor’s claims is public
information. Moreover, when a will is offered for probate, it becomes a public document revealing the
testator’s wishes as to who got what that anyone can access. Inter vivos trusts, on the other hand, are private
legal documents not subject to public inspection. {CT} {McL}
 Ancillary Probate: By placing real estate located in a jurisdiction other than the jurisdiction where probate is
opened in a revocable, inter vivos trust, ancillary probate can be avoided. {CT}
 Family Protection Doctrines: In some jurisdictions, family protection doctrines such as elective share apply
only to the decedent’s probate property. In such jurisdictions, an individual can shield his or her assets from
such claims by putting the property in a revocable trust.
 Continuity in Property Management: A revocable trust ensures continuity in property management upon
death. {McL}
 Challenges to Testamentary Scheme: A revocable trust can be contested for lack of general capacity; insane
delusion; or undue influence. However, as a practical matter, it is much more difficult to challenge a trust.
{McL}
o Unlike a will, heirs who are not beneficiaries of the trust are not usually entitled to see the trust
instrument
o If a trust continues as an ongoing operation for years before the settlor dies, generating monthly or
yearly statements and involving various property transfers by a third-party trustee, a court is likely
reluctant to set the trust aside.
 Less Court Supervision: A revocable trust is not subject to ongoing court supervision after the settlor’s death
unless an interested party brings suit to settle some trust matter. {McL}
o Testamentary Trusts: Testamentary trusts are subject to court supervision in the probate process.
Such probate court supervision adds costs and administrative burdens that are avoided by using an
inter vivos trust. {CT}
 Uniform/Consolidated Disposition of Assets: {McL}
 Choose the State Law: The settlor of a revocable trust that is funded with personal property—but not land,
which is governed by the law of its situs—may be able to choose the state law that is to govern the trust.
{McL}
o Deny Forced Share: In a few states, a married person may be able to use an out-of-state revocable
trust as a will substitute to deny a forced share to a surviving spouse
o Subsidiary Law of Wills: A potential downside to the use of a revocable trust as a will substitute is
the lack of certainty about the applicability of the constructional aids in the subsidiary law of wills.
 Estate Taxes: There are no federal estate tax benefits to using an inter vivos revocable trust. {CT}
 Dead Hand Control: A will typically devises property to the beneficiary in fee simple absolute. The
beneficiary then is free to use the property as he or she sees fit. A trust, on the other hand, facilitates
conditional gifts to beneficiaries. Beneficiaries may be entitled to receive property from the trust only as long
as they behave in a certain manner. Trusts can also be used to ensure that a surviving spouse complies with a
mutually agreed-on estate plan. Both spouses can put their assets in an inter vivos trust with an agreed-on
testamentary disposition scheme that becomes irrevocable upon the death of the first spouse. {CT}
C. The Other Will Substitutes
1. Life Insurance A. Life Insurance: Life insurance shifts the economic risk of premature death to an insurance company. Life insurance
contracts can validly transfer property on death without Wills Act formalities—they usually must be in writing and
signed by the insured.
 Term Life Insurance: A term policy is a contract that obligates the insurance company to pay the named
beneficiary if the insured dies within the policy’s term—commonly 1, 5, 10, 20, or even 30 years. Because
there is no saving feature or cash surrender value, term policies are less expensive than permanent life
insurance.
 Whole Life Insurance: Whole life insurance combines life insurance with a savings plan. The policy
eventually becomes paid up or endowed, after which no premiums are owed. It is the most expensive form of
life insurance.
B. Strict Compliance with Policy Requirements (Majority): An attempt to change the beneficiary of a life insurance
policy by will and in disregard of the methods prescribed under the contract is ineffective. Cook. Three exceptions to
this general rule are recognized:
 Waiver: If the insurer has waived a strict compliance with its own rules, and in pursuance of a request of the
insured to change the beneficiary, has issued a new certificate to him, the original beneficiary will not be
heard to complain that the course indicated by the regulations was not pursued.

55/87
 Beyond the Power: If it be beyond the power of the insured to comply literally with the regulations, a court
of equity will treat the change as having been legally made.
 Insured Has Done All in his Power: If the insured has pursued the course pointed out by the laws of the
insurer, and has done all in his power to change the beneficiary; but before the new certificate is actually
issued, he dies, a court of equity will decree that to be done which ought to be done, and act as though the
certificate had been issued.
1. Rationale: Protect rights of all the parties concerned (insurer, insured, and beneficiary)
 Designated Beneficiary: Right to be entitled to prompt payment of benefits by insurance companies
 Life Insurance Company: Ability to pay over benefits to person properly entitled to them without subjection
to claims by others of whose rights they had no notice or knowledge
 Insured: Confidence that designated beneficiary will receive death benefit.
 Society: Conservation of judicial energy and expense
C. Revocation by Divorce: In many states, the statute that revokes a will provision for an ex-spouse does not apply to
life insurance contracts. See Cook.
 UPC Approach: Under UPC § 2-940 (Duke, at 239), the revocation by operation of law doctrine applies to
all will substitutes, including life insurance policies.
 Sveen v. Melin: A revocation by divorce statute that applies to life insurance may not be able to apply
retroactively. This case currently decided by SCOTUS. {McL}

Does the application of a revocation-upon-divorce statute to a contract signed before the law was enacted violate the
Contract Clause of the Constitution?

 It found that first, the law in this case was intended to reflect the policyholder’s intent that he or she likely
would not want their life insurance proceeds to pass to a former spouse, thus supporting rather than
undermining the contractual scheme. Second, the law was not likely to thwart the policyholder’s
expectations, as the policyholder could not reasonably expect a beneficiary designation to stay in place after
a divorce. Third, the law in this case served as a mere default rule, which the policyholder could undo at any
time by sending in a new beneficiary designation form. The Court stated that such a minimal paperwork
burden does not violate the contracts clause under its well-established precedent.

D. Superwill: If a will is permitted to change the terms of a will substitute (i.e., insurance policy), such a will is known
as a “superwill.” Most jurisdictions have rejected the idea of a superwill. See Cook. {CT}
 UPC: The UPC adopts the superwill doctrine only if the contract permits the beneficiary of the policy to be
changed by a subsequently executed will. UPC § 6-101. {CT}
 Restatement: The Restatement (Duke, at 475) endorsed the superwill concept.
Cook v. Equitable Life Assurance Society (IN): Douglas (insured) named his first wife, Doris as the designated beneficiary of his life
insurance policy prior to their divorce. After his divorce, he married Margaret and had a son, Daniel, with her. Although he never changed the
beneficiary of the life insurance policy, he made a holographic will leaving the proceeds from the life insurance policy to Margaret and Daniel.
On appeal, the court held that in order for Margaret and Daniel to have defeated the motion for summary judgment in this case, they must have
made some showing that Douglass had done all within his powers or all that reasonably could have been expected of him to comply with the
policy provisions respecting a change of beneficiary, but that through no fault
of his own he was unable to achieve his goal. Here there was no such
indication or implication.

3. Pay-on-Death & A. POD & TOD Contracts


Transfer-on-Death  Old Rule: At common law, a written agreement that purported to transfer a property interest at time of death
Contracts was valid only if it qualified as a valid will or as a valid nonprobate transfer (i.e., joint tenancy). {CT}
 Majority/UPC: The UPC has authorized POD designations in all contracts (despite the lack of Wills Act
formalities), and almost every state has followed suit. {McL}
o UPC § 6-101. NONPROBATE TRANSFERS ON DEATH. A provision for a nonprobate transfer
on death in an insurance policy, contract of employment, bond, mortgage, promissory note,
certificated or uncertificated security, account agreement, custodial agreement, deposit agreement,

56/87
compensation plan, pension plan, individual retirement plan, employee benefit plan, trust,
conveyance, deed of gift, marital property agreement, or other written instrument of a similar nature
is nontestamentary. This subsection includes a written provision that:
 (1) money or other benefits due to, controlled by, or owned by a decedent before death
must be paid after the decedent’s death to a person whom the decedent designates either
in the instrument or in a separate writing, including a will, executed either before or at the
same time as the instrument, or later;
 (2) money due or to become due under the instrument ceases to be payable in the event of
death of the promisee or the promisor before payment or demand; or
 (3) any property controlled by or owned by the decedent before death which is the subject
of the instrument passes to a person the decedent designates either in the instrument or in
a separate writing, including a will, executed either before or at the same time as the
instrument, or later.
B. TOD Security Accounts: The Uniform Transfer on Death Security Registration Act permits securities to be
registered in TOD form. The UPC and virtually all states have adopted the act. UPC §§ 6-301 to -311. Most banks,
brokerage houses, mutual funds, and other such financial intermediaries allow POD and TOD designations on their
customer’s account. (Duke, at 490) {McL}
C. The Savings Account (Totten) Trust: Totten trust accounts are very similar to POD accounts. The depositor sets up
a Totten account by depositing money in the account in the name of the depositor “for the benefit of” the beneficiary.
The trust is deemed revocable so that depositor withdrawals are permitted. The beneficiary is entitled to what remains
on the depositor’s death.
 UPC: Under the UPC, the Totten trust is abolished as a formal category and is instead treated as a POD
account.
D. Multiple-Party Bank Accounts: If a bank or brokerage account is joint and survivor, owned by “A and B, as joint
tenants with right of survivorship,” both A and B have the power to draw on the account, but the survivor alone owns
the balance, which does not pass through probate. Sometimes, however, something other than a true joint tenancy is
intended.
 Latent Issue: Because banks and brokerage houses often give their customers a joint tenancy form without
regard to the customer’s particular intention, courts are sometimes left with the problem of discerning which
type of account was intended.
o Example: A, depositor, opens “joint account” with B.
1. Joint & Survivor Account: A depositor may create a multiple-party account because he or she intended to create a
joint tenancy account. Upon death, a right of survivorship exists such that the proper will pass nonprobate.
 Either A or B can draw on account—survivor owns balance {McL}
2. POD Account: The depositor may intent a POD account to avoid probate. The depositor does not intend for the other
party on the account to take any interest inter vivos, but at a time of death the depositor intends a right of survivorship.
 B can’t draw on account during A’s life, but is entitled to balance on A’s death {McL}
3. Agency Account: An agency account is where a depositor adds a second party to an account for the benefit of the
depositor (e.g., to facilitate paying bills). Inter vivos, the second party can access the account, but only to use it for the
benefit of the depositor, and upon death, there is no right of survivorship.
 B can draw on account during A’s life for A’s benefit but is not entitled to balance on A’s death. {McL}

4. Nonprobate A. Nonprobate Transfer of Real Property: Real property, for which ownership is indicated in a recorded deed, is less
Transfer of Real suited to nonprobate transfer, as the deed must be changed by probate administration or otherwise.
Property 1. Revocable Trust: Real property transferred during life to a revocable trust is owned by the trustee, obviating the
need for probate to change title to the property at the death of the settlor.
2. Joint Tenancy/Tenancy by the Entirety: Upon the death of one tenant, the surviving tenant owns the property
absolutely. No probate is necessary because nothing passes to the survivor.
 Devisability: Because a joint tenant’s interest in the property is extinguished at her death, she cannot devise
her interest in the property by will. If a joint tenant wants someone other than the other joint tenant to take her

57/87
share at death, she must sever the joint tenancy during life, converting it into a tenancy in common.
 Creditor’s Claims: A creditor of a joint tenancy ordinarily must seize the joint tenant’s interest, if at all,
during the joint tenant’s life because a joint tenant’s interest in the property is extinguished upon his or her
death.
3. Transfer-on-Death Deeds: A substantial minority of states (17 states) have enacted statutes, such as the Uniform
Real Property Transfer on Death Act (2009), absorbed into the UPC as §§ 6-401 to -407, which allow the transfer of
real property by TOD deed. {McL} T can identify the beneficiary who will succeed to Blackacre on T’s death by
recording a TOD deed (i.e., a deed conveying “to T, transfer on death to A”). During T’s life, the TOD beneficiary has
no interest in Blackacre, and T retains the power to transfer it to others or to revoke the TOD designation. {McL}
(Duke, at 495)
 TOD Deed for Real Property States: Duke, at 494.
8. Limits on Freedom of Disposition: Protection of the Surviving Spouse & Children
8. Limits on Freedom of Disposition: Some spousal protection and family protection doctrines limit one’s power to transfer one’s property at
death (freedom of disposition). They include: (1) the elective share; (2) community property; (3) omitted spouse; and (4) omitted child.
A. Protection of the Surviving Spouse
A. Separate Property vs. Community Property
 Separate Property: Under the separate property approach, spouses own all earnings and acquisitions from earnings during the marriage,
unless they agree to a joint form of ownership.
o Rationale: Individual autonomy over earning
 Community Property: Under community property, spouses retain separate ownership of property brought to the marriage, but they own
all earnings and acquisitions from earnings during the marriage in equal, undivided shares. The death of one spouse dissolves the
community. The deceased spouse owns and has testamentary power over his or her half of the community. The surviving spouse owns the
other half.
o Rationale: Marriage is an economic partnership
 Separate vs. Community Property States: Duke, at 513.

Separate Property Community Property


“Common Law Jurisdiction” “Community Property Jurisdiction”
 No automatic sharing of earnings; whatever spouse earns or  Earnings and property acquired with earnings during
acquires is his/hers unless he/she agrees to joint ownership. marriage are community property in which each spouse owns
{McL} an equal, undivided share. {McL}
 Protection against disinheritance of Surviving Spouse is  No elective share because SS already owns half of
provided through elective (aka forced) share {McL} community property; deceased spouse owns and has
testamentary power over his/her half. {McL}
1. Elective Share of a 1. Elective Share of a Separate Property Surviving Spouse: In all but one of the separate property states, a surviving
Separate Property spouse is entitled to an elective or forced share, typically one-third, of the decedent spouse’s estate. Under the statutes, a
Surviving Spouse surviving spouse can elect to take under the decedent’s will or to renounce the will and take a fractional share of the
decedent’s estate.
 Waiver: Spouses may waiver their elective share rights by premarital or marital agreement.
 Community Property State: In a community property state, there is no elective share, because each spouse
owns one-half of the earnings of the other spouse during the marriage.

58/87
A. Elective Share vs. Intestate Share vs. Share Under Will
 Intestate Share of SS: Percentage of probate estate (could be 100%) {McL}
 Share of SS under Decedent Spouse’s Will: Percentage of probate estate (could be 100%) {McL}
 Elective Share: May be smaller percentage, but of a potentially larger pie; probate and certain nonprobate
assets (“augmented estate”). {McL}
 Example: W transfers $2.9 million to Revocable Trust, daughter is beneficiary of trust at W’s death. W leaves
entire probate estate ($100,000) to H under her Will. Live in Common Law Jurisdiction with Elective Share
of 1/3 of decedent’s probate and nonprobate assets. Augmented estate is $2.9 million plus $100,000, which is
$3 million. One third of that is $1 million. The elective share is $ 1 million. {McL}
 YOU DON’T NEED TO BE DISINHERITED TO CLAIM AN ELECTIVE SHARE
a. Policies a. Policies Underlying Elective (Forced) Share:
Underlying Elective  Primary Policy Justification—Partnership Theory: Reward SS’s contributions to economic success of
(Forced) Share marriage. {McL}
 Second (Narrower) Policy Justification—Support Theory: Provide SS with adequate support after
decedent’s death. {McL}
1. Elective Share—Criticisms {McL}
 Not all common law jurisdictions have fully embraced partnership theory of marriage.
 The Elective Share can be inequitable if it gives the surviving spouse a fixed fractional share regardless of
length of marriage (i.e., 1 hour or 50 years)
 If estate is small, elective share (e.g., ½ or 1/3) may not be sufficient to provide surviving spouse with
adequate support
 Deceased spouse can sometimes circumvent elective share (i.e., disinherit SS) using nonprobate transfers.
b. Cohabitating &  Under traditional law a surviving cohabitating partner is not entitled to an elective share in the decedent
Same-Sex Marriage partner’s estate unless they were married.
 At year-end 2012, same-sex marriage with spousal elective share rights was recognized in multiple states.
Spousal-like elective shares were also available in a civil union or in a registered domestic partnership in a
few states.
c. Variation Across 1. Must the Surviving Spouse Accept a Life Estate?
States  Partnership Theory (Majority/1969 UPC): If a decedent were to leave the surviving spouse a life estate in
a certain amount of property, the surviving spouse does not have to accept the life estate or its value in partial
satisfaction of the elective share. If the surviving spouse renounces the life estate and elects to take her share
in fee simple, she is not credited (charged) with the life estate.
 1990 UPC: A surviving spouse who rejects the life estate is charged an amount equal to one-half the total
value of the property subject to the life estate. The practical effect is to force the surviving spouse to take the
life estate given to her by the decedent’s will.
 1993 UPC: A life estate renounced by a surviving spouse is not charged against the spouse’s elective share.
UPC § 2-209.
 Support Theory: The surviving spouse is not required to accept the life estate or its value in partial
satisfaction of the elective share.
2. Subsequently Deceased Surviving Spouse:
 Support Theory (Majority/UPC): The right of election may only be exercised by the surviving spouse or a
representative of the surviving spouse during the surviving spouse’s life
 Partnership Theory: The right of election may be exercised by a personal representative of the surviving
spouse after the surviving spouse’s death.
3. Incompetent Surviving Spouse
 Majority: To determine whether a guardian or other representative of an incompetent surviving spouse can
elect against the decedent’s will, a majority of states hold that all the surrounding facts and circumstances
should be taken into consideration. The majority view allows the representative to take into account the
preservation of the decedent’s estate plan and whether the surviving spouse would have wanted to abide by
her dead spouse’s will.
 Minority: To determine whether a guardian or other representative of an incompetent surviving spouse can
elect against the decedent’s will, A minority of states hold that a guardian can and should elect to take against
the will if it is to the surviving spouse’s economic benefit, calculated mathematically.
 1969 UPC: A court, acting for an incompetent surviving spouse, could order election against the decedent
spouse’s will only “after finding that exercise is necessary to provide adequate support for the protected
person during his probable life expectancy.” UPC § 2-203.
 1990 UPC: If a representative claims the elective share for an incompetent surviving spouse, the portion of
the elective share that exceeds what the decedent spouse provided for the survivor must be placed in a
custodial trust for the benefit of the surviving spouse. The trustee is given the power to expend income and
principal for the surviving spouse’s support, but upon the spouse’s death the trustee must transfer the trust
property to the residuary devisee’s under the will of the predeceased spouse or to the predeceased spouse’s
heirs. UPC § 2-212.
4. Abandonment
 Minority: In a minority of states, the elective share is denied to a surviving spouse who abandoned or refused

59/87
to support the deceased spouse.
d. Nonprobate d. Nonprobate Property: The original elective share statutes gave the surviving spouse a fractional share of the
Property decedent’s probate estate. One issue is whether the elective share should apply also to nonprobate transfers.
 Revocable Trusts: Either by statute or by judicial decision, in the majority of states a revocable trust created
by the decedent spouse is included in determining the surviving spouse’s elective share. But in some states, a
revocable trust is not subject to the elective share.
1. Judicial Responses
 Inter Vivos Trust with General Power of Appointment (MA—Modern Rule): The rule the court now
favors will treat as part of the estate of the deceased for the purposes of Mass. Gen. Laws ch. 191, § 15, assets
of an inter vivos trust created during the marriage 20 by the deceased spouse over which he or she alone had a
general power of appointment,21 exercisable by deed or by will. The objective test will involve no
consideration of the motive or intention of the spouse in creating the trust. Sullivan.
o Rationale: A divorced spouse greater rights in the assets of an inter vivos trust created and
controlled by the other spouse than are extended to a spouse who remains married until the death of
his or her spouse
o Old Rule: A surviving spouse’s elective share does not apply to property in a revocable trust
created by the decedent during that marriage. Sullivan.
 Inter Vivos Trust Created by a Third Party: Trust property over which a decedent spouse had a general
power of appointment (i.e., right to terminate trust/take assets during life) is not included in the estate of the
deceased that is subject to the elective share if the trust is created by a third party. Bongaards (MA).
o Rationale: The court will not frustrate the intent of a third party who is a stranger to the marriage.
a. Other Judicial Approaches
 Illusory Transfer Test: The property in a revocable trust established by the deceased spouse during the
marriage counts as part of the decedent’s assets subject to the elective share if the trust is an “illusory”
revocable trust.
 Intent-to-Defraud Test: If the decedent had an intent to defraud the surviving spouse of the elective share,
the property is subject to the elective share. Some courts look for subjective intent, others look for objective
intent, and still others consider whether the decedent had a present donative intent to transfer an interest in the
property.
b. Choice of Law: If a married couple lives in a state that does not apply its own law to nondomiciliary sited property,
it may be possible for one spouse to defeat the right of election of the other spouse by settling a revocable a trust in a
state where the revocable trust is not subject to the elective share.
 UPC: The law of the decedent’s domicile governs the right to take an elective share of property located in
another state. UPC § 2-202(d) (1990, rev. 2008).
 MA: The law of the trustee’s domicile governs the right to take an elective share
 NY: The law of the state where the couple was domiciled governs the right to take an elective share
2. Statutory Reform Regarding Elective Share & Nonprobate Assets: Many states have enacted statutes providing
objective criteria for determining which nonprobate transfers are subject to the elective share. For the most part, these
statutes include a list of nonprobate transfers that are added to the probate estate to constitute an augmented estate
against which the surviving spouse’s elective share is applied. One issue is whether a court should include in the
elective share a type of nonprobate transfer not specifically mentioned in the statute. See Myers.
 Iowa Approach: The elective share is limited to the four categories listed in the elective share statute (real
property, personal property, revocable trust). In re Estate of Myers.
 Delaware Approach: Delaware defines property subject to the elective share as all property includible in the
decedent’s gross estate under the federal estate tax, whether or not an estate tax return is filed. If a nonprobate
transfer is taxable at death (as are revocable trusts, POD contracts, and joint tenancies), it is subject to the
surviving spouse’s elective share.
a. Creditors & Medicaid: In most states, an insolvent surviving spouse cannot be compelled by an ordinary creditor to
take his elective share. However, the value of his elective share will be counted toward the surviving spouse’s available
resources in determining eligibility for Medicaid.

20 Includes revocable trust created before the marriage if the trust is amended during the marriage
21 The settlor of a revocable trust has a general power of appointment—a power to revoke the trust and take back the trust property is a
power to appoint oneself [the holder of the power].
60/87
Sullivan v. Burkin (MA): Mary Sullivan, the widow or Ernest Sullivan,
exercised her right, under a Massachusetts statute (§ 15), to take a share
of Ernest’s estate. Mary sought a determination that assets held in an
inter vivos trust created by Ernest during the marriage should be
considered as part of the estate in determining that share. On appeal, the
court held the assets of the inter vivos trust were not to be considered in
determining the portion of Ernest’s estate in which Mary had rights.
Mary was denied any claim against the assets of a valid inter vivos trust
created by Ernest, even where Ernest alone retained substantial rights
and powers under the trust instrument. For the future, however, as to any
inter vivos trust created or amended after the date of the opinion, the
estate of a decedent, for the purposes of § 15, was to include the value of assets held in an inter vivos trust created by the deceased spouse as to
which the deceased spouse alone retained the power during his or her life to direct the disposition of those trust assets for his or her benefit.
Bongaards v. Millen (MA):

In re Estate of Myers (IA): The probate court concluded that a checking account, a certificate of deposit, and an annuity that all had pay-on-
death (POD) clauses making them payable to the Karen Myer’s daughters were part Karen’s estate for purposes of the elective share. On appeal,
the court held that based on the language of Iowa Code § 633.238, POD assets were not included in the surviving spouse’s elective share. The
court noted that the legislature had amended § 633.238 to specifically provide that the assets of a revocable trust were included in the elective
share. By listing the specific assets that were included, and not mentioning POD assets or other non-probate assets, the legislature intended that
the POD assets not be included in the spouse’s elective share.
e. The Uniform 1. The 1969 Uniform Probate Code: Duke, at 533.
Probate Code  ES = 1/3 of “Augmented Estate”
o Augmented estate = probate estate + other listed transfers
2. The 1990 Uniform Probate Code: Duke, at 534.
 Make equal to community property state
 ES = % of Augmented Estate
o Augmented Estate = All property of both spouses
o % = sliding amount (Duke, at 535)

61/87
f. Waiver by f. Waiver by Premarital/Prenuptial or Marital/Postnuptial Agreement: A surviving spouse need not make an
Premarital or election, and a spouse can waive her right to an elective share (and/or intestate share) in advance. Typically, such a
Postnuptial waiver occurs in a premarital agreement (aka prenuptial or antenuptial agreement). All separate property states enforce
Agreement a waiver of the right of election by premarital agreement, and most also enforce a waiver agreed to during the marriage
—a postnuptial agreement.
 Uniform Premarital & Martial Agreement Act (UPMAA) (2012): Supersedes the UPAA. The UPMAA
expressly validates postnuptial marital agreements and applies the same substantive standards to the
enforceability of both the premarital and marital agreements. The UPMAA requires (1) voluntary consent, (2)
independent legal representation, (3) a notice of waiver of rights, and (4) adequate financial disclosure. Duke,
at 538.
 Adequate Financial Disclosure: Regarding prenuptial/antenuptial agreements, the term “knowledgeably” is
interpreted as meaning that the proponent of the agreement had to prove that a full and fair disclosure of the
nature, extent and value of the party’s holdings was provided or that such disclosure was unnecessary because
the spouse had independent knowledge of the same. The extent of what constitutes “full and fair” disclosure
varies from case to case depending upon a number of factors, including the relative sophistication of the
parties, the apparent fairness or unfairness of the substantive terms of the agreement, and any other
circumstance unique to the litigants and their specific situation. While disclosure need not reveal precisely
every asset owned by an individual spouse, at a minimum, full and fair disclosure requires that each
contracting party be given a clear idea of the nature, extent, and value of the other party’s property and
resources. Though not required, a fairly simple and effective method of proving disclosure is to attach a net
worth schedule of assets, liabilities, and income to the agreement itself. Though not required, representation
by independent counsel is a factor to take into account. Reece.
o Constructive Knowledge: A spouse may be charged with constructive knowledge of the other
party’s property, as under UMPAA § 9(d)(3). See Lawrence (constructive knowledge based on
awareness, rooted in more than two years of cohabitation, of spouse’s business dealings and
wealth); Martin (constructive knowledge based on access to financial records).
1. Good Practices for Prenuptial Agreements
 To avoid subsequent challenge to a premarital or marital agreement on the grounds of inadequate disclosure,
it is advisable to prepare a schedule of each party’s assets that includes a good faith valuation of each asset,
and to attach the schedule to the agreement. {McL}
 Have each party represented by independent counsel, even if not required. {McL}
a. What Provisions Can Be Included in Prenup?
 Courts generally will enforce provisions relating to property distribution at divorce/death. {McL}
 Courts will not enforce provisions that are contrary to public policy (i.e., waive parent’s obligation to pay
child support, deny child custody) {McL}
 Courts also generally refuse to enforce “lifestyle” provisions (i.e., one party must walk the dog, husband has
option of getting divorce if wife gained 15 pounds, health sex three times a week, probably infidelity clauses)
{McL}
2. Income Tax Basis Rules
 Income Tax: Income tax is a tax on accession of wealth
 Basis: Basis is a running record of after-tax dollars in property so we are not taxed again on that property
after sale. (Buy stocks with after-tax $10. Sell for $100. Basis is $90).

62/87
a. Lifetime Gifts: Carry-Over Basis, IRC § 1015
 Basis for determining loss limited to Fair-Market Value if FMV < AB at time of gift
 i.e., can’t shift losses by gift
b. Transfers on Death: Basis step-up or -down to Fair-Market-Value, IRC § 1014
 Erases any built-in gain
 Can’t shift losses at death either
 Inability to transfer losses not onerous b/c can sell before gift/death

c. Basis Step-Up at Death: Community Property vs. Separate Property (Duke, at 547)

Reece v. Elliott (TN): The widow of Eugene Reece sought rescission of an antenuptial agreement based upon her claim that Eugene failed to
make a full disclosure regarding his assets and financial condition. The agreement stated that Eugene and the widow had an interest in real and
personal property identified in the attached lists, and that the property listed would be considered the separate property of each. The document
also stated that the parties affirmed that a full disclosure of assets had been made. One item listed on Eugene’s document was “Stock in JH
Routh Packaging Company consisting of 1,687 shares.” On appeal, the widow’s argument was that she did not enter into the agreement with full
knowledge of the value of the Eugene’s assets because there was no value disclosed regarding the stock. The court held that the widow had full
knowledge that Eugene was a man of wealth, as shown by the list of assets that was provided. The fact that there was no value listed for one
particular asset, even though it was significant, did not invalidate the agreement that she entered freely. She consulted with independent counsel
and admitted to clearly understanding what the agreement meant and that she would have no claim to any of those assets. She had the
opportunity to ask questions about the assets and did not have her counsel investigate, stating that it “did not matter” because she knew the
assets would not be hers. She admitted that her husband was never dishonest with her and was very straightforward and open with her about his
financial dealings and never misled her. The widow did not avail herself of the opportunity to ask her husband the value of the stock nor make
any independent investigation; therefore, the agreement was binding and enforceable on the widow.
2. Community 2. Community Property: In community property states, each spouse owns all earnings during the marriage in equal,
Property undivided shares. There is no elective share because the surviving spouse already owns half of the couple’s community
property.
a. The Spread of the a. The Spread of the Community Property System: A community property system has long existed in eight states
Community (AZ, CA, ID, LA, NV, MM, TX, WA), and WI adopted the system in 1984. AL and TN have statutes that permit
Property System couples to opt into community property by way of a community property trust.
 Community Property: The fundamental principle of community property is that during marriage all
earnings of the spouses, and property acquired from those earnings, are community property unless both

63/87
spouses agree to separate ownership. Each spouse is the owner of an undivided one-half interest in the
community property. The death of one spouse dissolves the community. The deceased spouse owns and has
testamentary power over his or her half of the community; the surviving spouse already owns the other half.
 Separate Property: Property that is not community property is the separate property of one spouse or the
other or, in the case of a tenancy in common or joint tenancy, of both. Separate property includes property
acquired before marriage and property acquired during marriage by gift or inheritance.
 Income from Separate Property: In ID, LA, TX, and WI, income from separate property is community
property. In the other community property states, income from separate property retains its separate character.
 Presumption: In circumstances in which the characterization of property is doubtful, there is a strong
presumption in favor of community property.
 Tracing: If property has been commingled, or acquired from both separate and community funds, tracing
determines whether the portion of the property that is community.
o Basis-Step-Up at Death—Community Property: Upon the death of one spouse, under IRC §
1014, the entire value of community property receives a stepped-up basis for determining capital
gains when the property is eventually sold. Any appreciation in value between acquisition and the
date of the first spouse’s death is never taxed as capital gain.
o Basis-Step-Up at Death—Separate Property: If the property is not community property, only the
decedent’s one-half interest in the property receives a stepped-up basis.
 The Alaska & Tennessee Community Property Trusts: Alaska and Tennessee permit married couples to
transfer personal property into a community property trust and provide in the trust agreement that the
property is community property. The settlors need not reside in Alaska or Tennessee if they appoint as trustee
a bank in one of those states.
 Community Property with Right of Survivorship: Under the community property with a right of
survivorship system (AZ, CA, ID, NV, NM, TX), the decedent spouse cannot dispose of his share by will;
instead it passes under a right of survivorship to the surviving spouse.
b. Management & b. Management & Disposition of Community Property: Either spouse can dispose of his or her half of the couple’s
Disposition of community property at death. The surviving spouse owns the other half, which is not, which is not subject to
Community testamentary disposition by the deceased spouse. The one-half of the community property belonging to the deceased
Property spouse may be devised to whomever the decedent pleases, the same as separate property. There is no elective share over
the decedent spouse’s half of the community property because the surviving spouse already owns the other half.
 Management: In most community property states, each spouse generally has the power to manage
community property, but there are some exceptions for which both spouses are necessary, the most important
being for a transaction involving real property. TX and WI give each spouse exclusive management over his
or her earnings and property acquired with those earnings as long as they are held in the name of the earning
spouse.
 Creditors: In managerial states, the creditor has recourse against all community property subject to the
debtor spouse’s control. Under the community debt system, the creditor’s claim is characterized as separate or
community, and the creditor’s recourse follows accordingly.
 Disposition: To the extent that a spouse may act alone, that spouse may sell community property to a
purchaser for valuable consideration. But neither spouse can make a gift of community property without the
consent of the other spouse. Various remedies are available to a spouse who did not consent to the gift.
o Almost all community property states follow the theory that the spouses own equal shares in each
item of community property at death.
o If a nonprobate transfer to a person other than the spouse is treated as a death transfer, the surviving
spouse is entitled to one-half of it. If treated as an inter vivos gift, the surviving spouse may set
aside the transfer to the extent state law allows for lifetime gifts.
3. Migrating Couples  3. Migrating Couples & Multistate Property Holdings: The traditional conflict of law rules used to
& Multistate Property determine which state law governs marital property are as follows: (1) the law of the situs controls problems
Holdings related to land; (2) the law of the marital domicile at the time that personal property is acquired controls the
characterization of the property as separate or community; and (3) the law of the marital domicile at the death
of one spouse controls the survivor’s rights.
a. Moving from a. Moving from Separate to Community Property: If a married couple acquires property in a separate property state
Separate to and moves to a community property state . . .
Community  Traditional Law: The law of the state where the couple is domiciled when movable property is acquired
Property determines the ownership of that property.
 Quasi-Community Property: Several community property states give a remedy to the surviving spouse in
this situation by way of quasi-property. During the continuance of the marriage, quasi-community property is
treated for most purposes as the separate property of the acquiring spouse. However, in a state that recognizes
quasi-community property for probate purposes, one-half of the quasi-community property belongs to the
surviving spouse at death, with the other half subject to testamentary disposition by the decedent. The
surviving spouse may have the right to reach one-half of any nonprobate inter vivos transfer of quasi-
community property if the decedent retained possession or enjoyment, or the right to income, or the power to
revoke or consumer, or a right of survivorship.
b. Moving from b. Moving from Community Property to Separate Property: Generally, a change in domicile from a community

64/87
Community property state to a separate property state does not change the preexisting property rights of the spouses. However,
Property to couple moving community property into a separate property state should be careful to preserve its community nature, if
Separate Property doing so is desirable (done by revocable trust).
4. Misc. Additional
Rights
a. Social Security a. Social Security: Under the Social Security system, retirement benefits are paid to a work and the worker’s spouse. A
spouse can generally draw her owned earned benefits, if any, or one-half of the other spouse’s benefits, whichever is
greater. At the death of the other spouse, the surviving spouse is entitled to her owned earned benefits, if any, or the full
amount of the decedent spouse’s benefits. Workers have no power to transfer their right to benefits to any other person
or to divest the surviving spouse of his spousal right to benefits.
b. Pension & b. Pension & Retirement Accounts: Most private pension plans provided as a benefit of employment are governed by
Retirement the federal Employee Retirement Income Security Act of 1974 (ERISA), which per Egelhoff preempts state law. ERISA
Accounts requires that the spouse of an employee have survivorship rights if the employee predeceases the spouse.
 ERISA preempts state community property law to the extent that it allows the first wife to make a
testamentary transfer of her interest in her husband’s pension, making it unavailable to a second wife. Boggs.
 A spouse may waiver her rights under the employee’s pensions plan. A prenuptial agreement cannot waive
ERISA-covered pension rights.
c. Homestead c. Homestead: Most states have a homestead law that is designed to secure the family home to the surviving spouse
and minor children, free of the claims of the decedent’s creditors.
d. Personal d. Personal Property Set-Aside: The surviving spouse has the right to receive tangible personal property of the
Property Set-Aside decedent up to a certain value.
e. Family Allowance e. Family Allowance: Every state has a statute authorizing the probate court to award allowance for maintenance and
support of the surviving spouse (and often of dependent children). As with the homestead and personal property set-
aside, any family allowance is in addition to whatever other interests pass to the surviving spouse.
f. Dower & Curtesy f. Dower & Curtesy
 Dower: At common law, a widow had a dower in all the land of which her deceased husband had been seised
during marriage and that was inheritable by the descendants of husband and wife. Dower entitles the widow
to a life estate in one-third of her husband’s qualifying land. The right of dower attaches the moment the
husband acquires title to land or upon marriage, whichever is later. No purchaser, bona fide or not, can cut off
the wife’s dower without her consent.
 Curtesy: At common law, a husband had a support interest in his wife’s lands, called curtesy. It was
comparable to dower except (1) the husband did not acquire curtesy unless children were born of the
marriage, and (2) the husband was given a life estate in the entire parcel, not merely in one-third.
B. Intentional Omission of a Child
B. Intentional Omission of a Child: In the American Legal Tradition, a surviving child has no rights to a mandatory share. A property owner may
disinherit her blood relations, including her children, if that is her desire.
1. American Law 1. American Law: In all states, except Louisiana, a child or descendant has no statutory protection against intentional
disinheritance by a parent. There is no requirement that a testator leave any property to a child.
 Prof. Langbein: “[T]he American rule, allowing liberal disinheritance of children, creates the type of
plaintiff . . . most prone to bring these actions.” {McL}
o He nonetheless prefers “the American position of liberal disinheritance of liberal testamentary
freedom to disinherit children who turn out to be . . . disappointing and unsavory.” {McL}
 Louisiana: The Louisiana forced share for children is called a legitime. It protects against the disinheritance
of children under 23, the mentally infirm, and the disabled. Disinheritance of a covered child is allowed for
“just cause.”
C. Protection Against Unintentional Omission
C. Protection Against Unintentional Omission
 Spouse: Both separate property and community property states protect a pretermitted surviving spouse from accidental disinheritance by
way of a premarital will that the decedent spouse neglected to update after the marriage.
 Child: American law protects a child who is accidentally omitted from a will—such as a child born after the execution of a will that does
not contemplate subsequent children—but this is a default rule that may be overcome by express language in the will.
1. Spouse Omitted 1. Spouse Omitted from Premarital Will
from Premarital Will a. Common Law: At common law, the entire premarital will was revoked by the testator’s marriage or marriage
followed by the birth of issue.
 Rationale: Testator forgot to update will or procrastinated updating will
b. Modern Statutes: In most states, the deceased spouse’s will is revoked only with respect to the surviving spouse’s
intestate share, otherwise leaving the premarital will intact. The statutes are default rules only and do not apply if there
is evidence that the testator deliberately omitted the surviving spouse or did not mistakenly fail to update will. {McL}
o Rationale: Intended to correct for Testator’s assumed mistake in failing to update premarital will
(but not intentional failure to update). {McL}
 Example: An amendment to an inter vivos trust cannot serve to rebut the presumption that a will is revoked
as to an unintentionally omitted spouse. In re Estate of Prestie.
o W.R.’s form should have cited to Mayo and argued substance over form. W.R. saw this revocable
trust essentially as a will. {McL}

65/87
o Nevada has subsequently revised the statute to include revisions to revocable trusts.
 Example—UPC § 2-301. Entitlement of Spouse; Premarital Will. (a) If a testator’s surviving spouse
married the testator after the testator executed his [or her] will, the surviving spouse is entitled to receive, as
an intestate share, no less than the value of the share of the estate he [or she] would have received if the
testator had died intestate as to that portion of the testator’s estate, if any, that neither is devised to a child of
the testator who was born before the testator married the surviving spouse and who is not a child of the
surviving spouse nor is devised to a descendant of such a child or passes under Sections 2-603 or 2-604 to
such a child or to a descendant of such a child, unless:
o (1) it appears from the will or other evidence that the will was made in contemplation of the
testator’s marriage to the surviving spouse;
o (2) the will expresses the intention that it is to be effective notwithstanding any subsequent
marriage; or
o (3) the testator provided for the spouse by transfer outside the will and the intent that the transfer be
in lieu of a testamentary provision is shown by the testator’s statements or is reasonably inferred
from the amount of the transfer or other evidence.
 (b) In satisfying the share provided by this section, devises made by the will to the testator’s surviving spouse,
if any, are applied first, and other devises, other than a devise to a child of the testator who was born before
the testator married the surviving spouse and who is not a child of the surviving spouse or a devise or
substitute gift under Section 2-603 or 2-604 to a descendant of such a child, abate as provided in Section 3-
902.
Review—Protection of 1. SS Disinherited Under POSTmarital Will: Disinheritance Intentional
Surviving Spouse  Common Law: Elective Share (sliding percentage of augmented estate)
{McL}  Community Property: SS owns ½ community property
2. SS Disinherited Under PREmarital Will: Disinheritance may not be intentional
 Common Law:
o 1. Omitted Spouse Share OR
o 2. Elective Share
 Community Property:
o 1. SS owns ½ Community Property AND (in some jurisdictions)
o 2. Omitted Spouse Share
In re Estate of Prestie (NV): W.R. Prestie and Maria were married in 1987. They were divorced two years later. In 1994, W.R. simultaneously
executed in California a pour-over will and an inter vivos trust. The pour-over will devised W.R.’s entire estate to the trust. W.R.’s son was
named as both the trustee and a beneficiary of the trust. Neither the will nor the trust provided for Maria. In 2001, W.R. amended the trust to
grant Maria a life estate in his condominium upon his death. A few weeks later, W.R. and Maria remarried. W.R. passed away approximately
nine months later. On appeal, the court held that the amendment to the trust could not serve to rebut the presumption that the will was revoked
as to Maria because NRS § 133.110 permitted only three exceptions to rebut the presumption of revocation—a provision in a marriage
contract, the spouse is provided for in the will, or the spouse is mentioned in the will as to show an intention not to provide for him/her
—and an amendment to an inter vivos trust was not one of the exceptions. Therefore, the will was revoked as to Maria and she received a ½
intestate share along with what was provided for her in the trust agreement.

66/87
2. Unintentional 2. Unintentional Disinheritance of a Child: Pretermitted heir statutes are designed to prevent the unintentional
Disinheritance of a disinheritance of a child. Many pretermitted heir statutes apply only to children born after the execution of the will (i.e.,
Child UPC § 2-302), but some protect children alive when the will was executed as well as afterborn children.
 UPC § 2-302. Omitted Children. (Duke, at 566).
 (a) Except as provided in subsection (b), if a testator fails to provide in his [or her] will for any of his [or her]
children born or adopted after the execution of the will, the omitted afterborn or after-adopted child receives a
share in the estate as follows:
o (1) If the testator had no child living when he [or she] executed the will, an omitted after-born or
after-adopted child receives a share in the estate equal in value to that which the child would have
received had the testator died intestate, unless the will devised all or substantially all of the estate to
the other parent of the omitted child and that other parent survives the testator and is entitled to take
under the will.
o (2) If the testator had one or more children living when he [or she] executed the will, and the will
devised property or an interest in property to one or more of the then-living children, an omitted
after-born or after-adopted child is entitled to share in the testator’s estate as follows:
 (A) The portion of the testator’s estate in which the omitted after-born or after-adopted
child is entitled to share is limited to devises made to the testator’s then-living children
under the will.
 (B) The omitted after-born or after-adopted child is entitled to receive the share of the
testator’s estate, as limited in subparagraph (A), that the child would have received had
the testator included all omitted after-born and after-adopted children with the children to
whom devises were made under the will and had given an equal share of the estate to
each child.
 (C) To the extent feasible, the interest granted an omitted after-born or after-adopted child
under this section must be of the same character, whether equitable or legal, present or
future, as that devised to the testator’s then-living children under the will.
 (D) In satisfying a share provided by this paragraph, devises to the testator’s children who
were living when the will was executed abate ratably.22 In abating the devises of the then-
living children, the court shall preserve to the maximum extent possible the character of
the testamentary plan adopted by the testator.
 (b) Neither subsection (a)(1) nor subsection (a)(2) applies if: (1) it appears from the will that the omission
was intentional; or (2) the testator provided for the omitted after-born or after-adopted child by transfer
outside the will and the intent that the transfer be in lieu of a testamentary provision is shown by the testator’s
statements or is reasonably inferred from the amount of the transfer or other evidence.
 (c) If at the time of execution of the will the testator fails to provide in his [or her] will for a living child
solely because he [or she] believes the child to be dead, the child is entitled to share in the estate as if the
child were an omitted after-born or after-adopted child. (d) In satisfying a share provided by subsection (a)(1),
devises made by the will abate under Section 3-902.
 Applies only to children born after the execution of the will.
 UCC—Omitted Children: Omitted child receives nothing if the testator’s will devises all or substantially all
of the testator’s estate to the other parent and that parent survives and is entitled to take. {McL}
o Omitted children participates on pro rata basis in property devised to the testator’s other children.
{McL}
 Example: When G executed her will, she had two living children, A and B. Her will devised $7,500 to each
child. After G executed her will, she had another child, C.
o C is entitled to $5,000. $2,500 (1/3 of $7,500) of C’s entitlement comes from A’s $7,500 devise
(reducing it to $5,000); and $2,500 (1/3 of $7,500) comes from B’s $7,500 devise (reducing it to
$5,000).
o Variation: If G’s will had devised $10,000 to A and $5,000 to B, C would be entitled to $5,000.
$3,333 (1/3 of $10,000) of C’s entitlement comes from A’s $10,000 devise (reducing it to $6,667);
and $1,667 (1/3 of $5,000) comes from B’s $5,000 devise (reducing it to $3,333).
9. Trusts: Fiduciary Administration
9. Trusts: Fiduciary Administration: One advantage of a trustee holding legal title to the trust property is managerial intermediation between the
beneficiary and the trust property. The intermediary role of the trustee involves custody, administration, investment, and distribution of the trust
property in accordance with the terms of the trust. The purpose of the law of fiduciary administration is to induce the trustee to adhere to the terms
of the trust and to act prudently and in the best interests of the beneficiaries. Trustees are subject to overarching fiduciary duties of loyalty and
prudence, and to a host of subsidiary duties, such as keeping adequate records and disclosing information about the trust to the beneficiaries. A
trustee who is found to be in breach may be removed from office, and the beneficiaries will be entitled to an election among remedies that include
disgorgement of any profit by the trustee and compensatory damages.
A. From Limited Powers to Fiduciary Administration
A. From Limited Powers to Fiduciary Administration: Originally, trust law protected beneficiaries by giving the trustee only limited powers.
Now, the trustee is given broad powers of administration, but the exercise of these powers is subject to the trustee’s beneficiary duties. See RS3d
Trust § 70 cmt. a.
1. From Conveyance 1. From Conveyance to Management: Originally, the trust was a conveyancing device that defeated both the rigidity

22 proportionally
67/87
to Management of intestacy and the burdensome taxes that pertained when land passed on intestacy. Today’s trust has become instead a
management device for holding a portfolio of financial assets.
 From Amateur to Professional Trusteeship: Trustees of older were unpaid amateurs. Private trustees still
abound, but the prototypical modern trustee is the fee-paid professional whose business is to enter into and
carry out trust agreements.
 Overcoming Trustee Disempowerment: Trustee disempowerment was the original system of beneficiary
safeguard in the law of trusts. However, disempowerment had to be abandoned because a modern trustee
conducts a program of investing and managing financial assets that requires extensive discretion to respond to
changing market forces.
 The Rise of Fiduciary Law: Trustees with transactional powers necessarily have the power to abuse as well
as to advance the interests of beneficiaries. To prevent abuse, trustees were subject to duties, protective in
nature, which were elaborated into trust fiduciary law. All trust fiduciary law rests on (1) the care norm (the
duty of prudent administration) and (2) the loyalty norm (the duty to administer the trust for the benefit of the
beneficiary).
a. Three Kinds of Trusts in Practice: Contemporary American trust practice has fractured into three branches: (a)
business trusts for commercial deals; (b) revocable trusts for nonprobate transfers; and (c) irrevocable trusts for ongoing
fiduciary administration.
 Irrevocable Trusts for Ongoing Fiduciary Administration: An irrevocable trust involves ongoing fiduciary
administration of property by a trustee in accordance with the settlor’s intent.
2. Trustee’s Powers 2. Trustee’s Powers:
 Traditional Law: A trustee’s powers were limited to those granted expressly in the trust instrument.
 Modern Approach: Statutes broaden trustees’ powers by (1) permitting the settlor to incorporate by
reference in the trust instrument all or some enumerated statutory powers, or (2) changing the default law to
give trustees a statutory list of powers, such as under UTPA § 3(c).
o A well-drafted trust agreement still drafts a list of powers because people feel more comfortable if
the powers are in the trust document.
 UTC: In addition to the “powers conferred by the terms of the trust,” a trustee has “all powers over the trust
property which an unmarried competent owner has over individually owned property,” plus “any other
powers appropriate to achieve the proper investment, management, and distribution of the trust property. UTC
§ 815. In other words, under the UTC, a trustee has the same powers in dealing with trust property as would
an outright owner.
a. Duties of a Third Party Dealing with a Trustee: At common law, a third party dealing with a trustee was required
to inquire whether the trustee had the power to undertake the proposed transaction. If the trustee did not have the power
to do so, the third party could be held liable by the beneficiary for the trustee’s breach. Most states have abrogated this
common law rule by statute. See UTPA § 7; UTC § 1012(a)-(b).
 Certification of Trust: A trustee may provide a third party with a “certification of trust” to verify that the
trust is valid and that the trustee has the power to engage in the transaction. UTC § 1013. The third party is
entitled to “assume without inquiry the existence of the facts contained in the certification.”
3. Fiduciary 3. Fiduciary Governance: In trust practice today, fiduciary governance has replaced limited powers as the primary
Governance means of protecting the beneficiary from abuse by the trustee.
 Trust law provides the trustee expansive default powers of administration, the trustee’s exercise of which is
subject to the trustee’s fiduciary duties. The core fiduciary duties of the trustee are the duties of (1) loyalty to
beneficiaries and (2) prudence—which entails a host of subsidiary duties, including the duty of impartiality
and the duty to inform and account. {McL}
 Rationale: (1) Lowering transaction costs. (2) The functional core of fiduciary law is deterrence. The
fiduciary is induced to act in the best interests of the beneficiary by the threat of after-the-fact liability for
breach of the fiduciary standards of conduct. {McL}
a. The Difference Between Power and Duty: Litigation over trust administration can involve one or both of two
allegations of breach of trust: (a) the trustee lacked the power to act as she did, or (b) the trustee’s act, even if within her
powers, was a breach of duty.
b. Who is the Principal? In many respects American trust law regards the settlor as the primary principal. A
beneficiary cannot easily remove a trustee, modify or terminate the trust without the settlor’s consent, or alienate her
beneficial interest.
B. The Duty of Loyalty (to beneficiaries)
B. The Duty of Loyalty: The trustee is under a duty to the beneficiary to administer the trust solely in the interest of the beneficiary. RS2d Trusts §
170.
 Rule Against Self-Interest (self-dealing):
o A trustee cannot purchase from himself at his own sale, and the trustee’s spouse cannot purchase from the trustee, unless leave
to do so has been previously obtained under an order of the court. Harman.
o A trustee cannot deal in his individual capacity with the trust property, including leasing the property to himself. Gleeson’s Will.
a. The No-Further-Inquiry Rule: If a trustee undertakes a transaction that involves self-dealing or a conflict between the trustee’s fiduciary
capacity and personal interests, good faith and fairness are not enough to save the trustee from liability. In such a case, no further inquiry is made;
the trustee’s good faith and the reasonableness of the transaction are irrelevant. See Harman; Gleeson. {McL}
 Rationale: Because appropriating trust property is profitable for the trustee but often difficult for the beneficiary to detect, the law infers
disloyalty from the fact of self-dealing. The purpose is to deter self-dealing and conflicted transactions, or to induce the trustee to seek
advance judicial approval or to disclose material facts to the beneficiaries to obtain their consent. {McL}
68/87
i. Categorical Exceptions
 Corporate Trustee: Statutes in most states allow a corporate trustee to deposit trust assets with its own banking department and to invest
the trust assets in a mutual fund operated by the trustee or an affiliate. UTC § 802(f). {McL}
 Compensation: The trustee may take reasonable compensation even though, strictly speaking, compensating herself with trust funds is
self-dealing. UTC § 802(h). {McL}
 Structural Conflict: The no-further-inquiry rule is inapplicable to a structural conflict created by the settlor (i.e., a trustee who is a
remainder beneficiary—see Howard). {McL}
 RATIONALE: These transactions actually benefit the beneficiaries.
b. Defenses: The only defenses that a trustee may raise are that: (1) the settlor authorized the particular self-dealing or conflicted action in the trust
instrument; (2) the beneficiary consented after full disclosure; or (3) the trustee obtained judicial approval in advance. Even if the trustee has such a
defense, the beneficiary remains entitled to judicial review of whether the trustee acted in good faith and of the fairness of the transaction. See
RS3d Trusts § 78 cmts. b, c. {McL}
c. Remedies for Breach: In the event of a breach of duty by a trustee, a beneficiary is entitled to an election among remedies that include
compensatory damages to restore the trust estate and trust distributions to what they would have been but for the breach. In addition, the
beneficiary is entitled to disgorge the trustee of any profit made on the transaction.
 If the trustee has sold his own property to the trust, the beneficiary may compel the trustee to repay the purchase price and take back the
property.
 If the trustee has bought trust property, the beneficiary may compel the trustee to restore the property to the trust.
 If in wrongfully disposing of trust property the trustee acquires other property, the beneficiary is entitled to enforce a constructive trust or
equitable lien on the property so acquired, treating it as part of the trust.
 The beneficiaries may enforce a constructive trust or equitable lien against a third party who acquires trust property, unless the third party
is a bona fide purchaser for value and without notice of the breach of trust.
Harman v. Hartle (NJ): Geick died testate, leaving five children. She named her two sons-in-law executors and they qualified. Among other
matters the will expressly directed her executors to sell her real estate and to divide the proceeds equally among her children. The executors sold
part of the real estate at public auction, for $ 3,900, to one of Geick’s sons, Lewis Geick, who actually bought the property for his sister,
Josephine Dieker, who is the wife of one of the executors. Josephine sold the property to a third party for $ 5,500. The executors settled their
final accounts, and that time the complainant (one of Geick’s children) expressed her dissatisfaction with the price realized from the sale of the
farm. The court held that Josephine and the executors were accountable for complainant’s one-fifth share of the profits made on the resale of the
property because, under the rule against self-dealing and the no-further-inquiry rule, a trustee cannot sell trust property to himself or to his
spouse.
In re Gleeson’s Will (IL): Colbrook was the executor of Gleeson’s will and was the trustee over Gleeson’s farmland. The beneficiaries were the
Gleeson’s children. Gleeson lease farmland to Colbrook for two years. After Gleeson’s death, Colbrook—with the expiration of lease imminent
—leased the farmland to himself and another person as a partnership for another year with increase in rent plus a share of the crops. On appeal,
the court held that Colbrook violated the rule that a trustee could not deal with trust property in an individual capacity and receive a profit by
leasing a portion of the real estate to himself. The court rejected the argument that Colbrook fell within an exception because of his good faith
attempt to farm the land after farm tenants could not be found because(1) there was no showing that Colbrook tried to find a satisfactory tenant;
(2) Colbrook’s good faith and honesty even where the trust sustained no loss did not take him outside of the rule; and (3) Colbrook failed to
confer with the children concerning the tenancy. Instead, he chose to proceed as a trustee and violated the rule against self-interest.
Consequently, the court disgorged Colbrook of any profits from the lease.
C. The Duty of Prudence
C. The Duty of Prudence: The duty of prudence imposes an objective standard of care in all functions of the trusteeship: custodial, administrative,
investment, and distribution. {McL}
 UTC § 804. Prudent Administration. A trustee shall administer the trust as a prudent person would, by considering the purposes, terms,
distributional requirements, and other circumstances of the trust. In satisfying this standard, the trustee shall exercise reasonable care,
skill, and caution. {McL}
1. The Distribution 1. The Distribution Function: The distribution function involves making disbursements of income or principal to the
Function beneficiaries in accordance with the terms of the trust, which may be mandatory or discretionary. A fiduciary must
exercise this discretion prudently, in good faith, and in accordance with the terms of the trust and in light of the needs
and circumstances of the beneficiaries. {McL}
 Mandatory Trust: In a mandatory trust, the trustee must make specified distributions to an identified
beneficiary (i.e., distribute all income of trust quarterly to A)
 Discretionary Trust: In a discretionary trust, the trustee has discretion over when, to whom, or in what
amounts to make a distribution.
o Spray Trust: The trustee must distribute all income currently but has so discretion over to whom
and in what amounts.
o Sprinkle Trust: The trustee may accumulate income and add it to the principal
o Support Trust: The trustee’s discretion over distributions is subject to a standard, such as “to
provide for the beneficiary’s comfortable support and maintenance.”
a. Discretionary a. Discretionary Distributions: Duty to make disbursements of income or principal to beneficiaries in accordance with
Distributions the terms of the trust. A fiduciary must exercise this discretion prudently, good faith, and in accordance with the terms
of the trust and in light of the needs and circumstances of the beneficiaries. {McL}
 “Comfortable Support & Maintenance”: The language, “as they shall deem advisable for his comfortable
support and maintenance” sets an ascertainable standard, namely to maintain the life beneficiary “in
accordance with the standard of living which was normal for him before he became a beneficiary of the
trust.” Marsman.
o Trustees Generally Conservative: Trustees are usually conservative in the “comfortable support
69/87
and maintenance” standard because they can be personally liable to remainder beneficiaries if they
overdistribute to the lifetime beneficiary, but if they under-distribute, the only thing that happens is
the lifetime gets a court order for the trustee to distribute more from the trust. Trustees also usually
get a percentage of the fee. {McL}
 Duty of Inquiry: That there is a duty of inquiry into the needs of the beneficiary follows from the
requirement that the trustee’s power must be exercised with that soundness of judgment which follows from a
due appreciation of trust responsibility. Marsman.
o Feed the file, the file is your friend: Periodically (monthly, quarterly, biannually, etc.) ask for
documentation of beneficiary’s income. {McL}
o Do not shame beneficiaries into not asking for money. Marsman. {McL}
 The Beneficiary’s Other Resources: Many cases hold that a trustee, in exercising a discretionary power to
use income or principal for a beneficiary’s support, should not consider the other resources available to the
beneficiary. However, many cases and RS3d of Trusts hold the opposite.
b. Sole, Absolute, or b. Sole, Absolute, or Uncontrolled Discretion: A court will not interfere with a discretionary judgment of a trustee so
Uncontrolled long as the trustee acts reasonably and in good faith. Even when a settlor purports to free the trustee from fiduciary
Discretion responsibility by use of adjectives such as sole, absolute, or uncontrolled in describing the trustee’s discretion, there are
limits on the trustee’s sole, absolute, or uncontrolled discretion. See Marsan (Even where the only direction to the
trustee is that he shall “in his discretion” pay such portion of the principal as he shall “deem advisable,” the discretion is
not absolute. “Prudence and reasonableness, not caprice or careless good nature, much less a desire on the part of the
trustee to be relieved from trouble . . . furnish the standard of conduct.”).
 Rationale: Unfettered discretion granted to a trustee would result in more of an ownership relationship
(precatory trust) than a trust relationship. {McL}
 Good Faith Standard
 UTC Approach: Notwithstanding the breadth of discretion granted to a trustee in the terms of the trust,
including the use of such terms as “absolute”, “sole”, or “uncontrolled”, the trustee shall exercise a
discretionary power in good faith and in accordance with the terms and purposes of the trust and the interests
of the beneficiaries. UTC § 814(a).
 Restatement Approach: A trustee must act honestly and in a state of mind contemplated by the settlor. Thus,
the court will not permit the trustee to act in bad faith or for some purpose or motive other than to accomplish
the purposes of the discretionary power.
c. Exculpation c. Exculpation Clauses: Although exculpatory clauses are not looked upon with favor and are strictly construed, such
Clauses provisions inserted in the trust instrument without any overreaching or abuse by the trustee of any fiduciary or
confidential relationship to the settlor are generally held effective except as to breaches of trust committed in bad faith
or intentionally or with reckless indifference to the interest of the beneficiary. Marsman.
o Example: “No Trustee hereunder shall be liable except for his own willful neglect or default”
 RS3d Trusts: Even if a fully informed settlor knowingly includes an exculpation clause, the clause cannot
exculpate bad faith, reckless indifference, or intentional or willful neglect by the trustee. See RS3d Trusts §
96 cmt. c.
 UTC Approach: (a) A term of a trust relieving a trustee of liability for breach of trust is unenforceable to the
extent that it: (1) relieves the trustee of liability for breach of trust committed in bad faith or with reckless
indifference to the purposes of the trust or the interests of the beneficiaries. UTC § 1008(a).
i. Trustee-Drafter
 UTC Approach: An exculpatory term drafted or caused to be drafted by the trustee is invalid as an abuse of a
fiduciary or confidential relationship unless the trustee proves that the exculpatory term is fair under the
circumstances and that its existence and contents were adequately communicated to the settlor. UTC §
1008. {McL}
 Rationale: Easy for drafter to slip in an exculpatory clause provision.
o Independent Counsel: If the settlor was represented by independent counsel, the settlor’s attorney
is considered the drafter of the instrument even if the attorney used the trustee’s form. UTC § 1008
cmt.
 MA: A court may refuse to enforce an exculpatory clause inserted by the trustee-drafter. Rutanen (rejecting
Marsman)

70/87
Marsman v. Nasca (MA): Sara Marsman died, leaving a
will that created a testamentary trust. The trust instrument
provided that the income of the trust was to go to Fred
quarterly for life. Regarding the principal, the trustees
were to “consider the various available sources of
support for him . . . [and] in their sole and uncontrolled
discretion, pay over to him . . . amounts of the
principal . . . as they deem advisable for his comfortable
support and maintenance.” After Sara’s death Fred asked
for a disbursement and the trustee asked Fred to explain
in writing the need for some support. Fred never asked
for a disbursement again. Aside from that one payment,
the trustee never made any payments to Fred and, as a
result, Fred had to convey his house. On appeal, the court
held that the trustee should have made distributions to
Fred from the principal so that he could have kept the
house that Fred and Sara had owned in tenants by the
entirety. The court found that the trustee failed in his duty
of inquiry into the needs of Fred. The court determined, however, that the proper remedy was not to set aside the conveyance of the house
(because the conveyee was a BFP) but to determine the amounts which should have been paid to Fred to enable him to keep the house, and to
pay that amount from the trust to the Fred’s estate. The court also imposed a constructive trust. The court also found that the exculpatory clause
which had been drafted by the trustee was effective and thus the trustee would not be personally charged.
2. The Investment 2. The Investment Function: The investment function involves reviewing the trust assets and then making and
Function implementing an ongoing program of investment in light of the purpose of the trust and the circumstances of the
beneficiaries. {McL}
 Trustee often delegates the investment function
a. From Legal Lists a. From Legal Lists to Prudent Man to Prudent Investor
to Prudent Man to  Legal Lists: After the South Sea Bubble burst, the court developed a “legal lists” of presumptively prudent
Prudent Investor investments for trustees (e.g., government bonds, well-secured first mortgages, NOT stocks).
o Rationale: Preserve principal
 Prudent Man Rule: Directed trustees “to make such investments and only such investments as a prudent
man would make of his own property having in view the preservation of the estate and the amount and
regularity of income to be derived.” RS2d Trusts § 227 (based on Amory). {McL}
o Criticism: Too much emphasis on how risky investments are; Risk is correlated with return;
Prudence of one individual investment cannot be determined without attention to the portfolio as a
whole; Unsystematic risk can be removed through diversification; Low risk investments expose the
trust to inflation risk; courts employed hindsight bias
 Prudent Investor Rule: Eventually, the states repealed the prudent man rule in favor of the new prudent
investor rule (see section b).
b. The Prudent b. The Prudent Investor Rule & Its Rationale:
Investor Rule & Its 1. Text
Rationale UNIFORM PRUDENT INVESTOR ACT (1994) (Duke, at 621).
SECTION 1. PRUDENT INVESTOR RULE.
 (a) Except as otherwise provided in subsection (b), a trustee who invests and manages trust assets owes a duty
to the beneficiaries of the trust to comply with the prudent investor rule set forth in this [Act].
 (b) The prudent investor rule, a default rule, may be expanded, restricted, eliminated, or otherwise altered by
the provisions of a trust. A trustee is not liable to a beneficiary to the extent that the trustee acted in
reasonable reliance on the provisions of the trust.
SECTION 2. STANDARD OF CARE; PORTFOLIO STRATEGY; RISK AND RETURN OBJECTIVES.
 (a) A trustee shall invest and manage trust assets as a prudent investor would, by considering the purposes,
terms, distribution requirements, and other circumstances of the trust. In satisfying this standard, the trustee
shall exercise reasonable care, skill, and caution.
 (b) A trustee’s investment and management decisions respecting individual assets must be evaluated not in
isolation but in the context of the trust portfolio as a whole and as a part of an overall investment strategy
having risk and return objectives reasonably suited to the trust.
 (c) Among circumstances that a trustee shall consider in investing and managing trust assets are such of the
following as are relevant to the trust or its beneficiaries:
o (1) general economic conditions;
o (2) the possible effect of inflation or deflation;
o (3) the expected tax consequences of investment decisions or strategies;
o (4) the role that each investment or course of action plays within the overall trust portfolio, which
may include financial assets, interests in closely held enterprises, tangible and intangible personal
property, and real property;
o (5) the expected total return from income and the appreciation of capital;
o (6) other resources of the beneficiaries;

71/87
o (7) needs for liquidity, regularity of income, and preservation or appreciation of capital; and
o (8) an asset’s special relationship or special value, if any, to the purposes of the trust or to one or
more of the beneficiaries.
 (d) A trustee shall make a reasonable effort to verify facts relevant to the investment and management of trust
assets.
 (e) A trustee may invest in any kind of property or type of investment consistent with the standards of this
[Act].
 (f) A trustee who has special skills or expertise, or is named trustee in reliance upon the trustee’s
representation that the trustee has special skills or expertise, has a duty to use those special skills or expertise.
SECTION 3. DIVERSIFICATION.
 A trustee shall diversify the investments of the trust unless the trustee reasonably determines that, because of
special circumstances, the purposes of the trust are better served without diversifying.
SECTION 4. DUTIES AT INCEPTION OF TRUSTEESHIP.
 Within a reasonable time after accepting a trusteeship or receiving trust assets, a trustee shall review the trust
assets and make and implement decisions concerning the retention and disposition of assets, in order to bring
the trust portfolio into compliance with the purposes, terms, distribution requirements, and other
circumstances of the trust, and with the requirements of this [Act]
1. Uniform Prudent Investor Act:
a. Duty to Diversify: The “trustee shall diversify the investments of the trust unless the trustee reasonably determines
that, because of special circumstances, the purposes of the trust are better served without diversifying.” UPIA § 3.
{McL}
 Market Risk: Market risk is common to all securities; it reflects general economic and political conditions,
interest rates, and so forth. It cannot be reduced by diversification. {McL}
 Industry Risk: Industry risk is specific to the firms in a particular industry or an industry grouping (i.e.,
international oil industry). It can be reduced by diversification. {McL}
 Firm Risk: Firm risk refers to factors that touch the fortunes only of the individual firm. It can be reduced by
diversification. {McL}
b. Portfolio: The standard of prudence is applied to any investment as part of the total portfolio, rather than to
individual investments. UPIA § 2. {McL}
 “A trustee’s investment and management decisions respecting individual assets must be evaluated not in
isolation but in the context of the trust portfolio as a whole and as a part of an overall investment strategy
having risk and return objectives reasonably suited to the trust.” UPIA § 2(b). “An investment that might be
imprudent standing alone can become prudent if undertaken in sensible relation to other trust assets, or to
other nontrust assets.” UPIA cmt.
c. Sensitivity to the Risk & Return: The UPIA eliminates the need to identify categories of investment that are per se
prudent or imprudent. Instead, a “trustee may invest in any kind of property or type of investment consistent with the
standards of this [Act].” UPIA § 2(e). The “trustee’s investment and management decisions” are required “to hav[e] risk
and return objectives reasonably suited to the trust. UPIA § 2(b).
 The fiduciary’s central consideration is the tradeoff in all investing between risk and return. UPIA § 2. (McL}
 All categoric restrictions on types of investments have been abrogated; the trustee can invest in anything that
plays an appropriate role in achieving the risk/return objectives of the trust and that meets the other
requirements of prudent investing. UPIA § 2. {McL}
 “[T]olerance for risk varies greatly with the financial and other circumstances of the investor, or in the case of
a trust, with the purposes of the trust and the relevant circumstances of the beneficiaries.” UPIA cmt. (low
tolerance for risk for trust benefitting elderly widow, higher tolerance for toddler child of billionaire who has
50 trusts for the child). {McL}
o Factors: Financial/other circumstances of beneficiaries, purpose of trust, size of trust. {McL}
2. Rationale: The prudent investor rule implements teaching of “Modern Portfolio Theory.” {McL} Modern Portfolio
Theory isolates (1) market risk, (2) industry risk, and (3) firm risk. Market risk cannot be eliminated through
diversification, but industry risk and firm risk can be. Owning many securities enhances the chances of offsetting losers
with winners. *****Develop MORE—ADD FROM CT
c. Recurring c. Recurring Problems Applying the Prudent Investor Rule: Most of the litigation under the prudent investment rule
Problems in concerns diversification problems, often involving an allegation that a trustee failed within a reasonable time to divest
Applying the an inception concentration.
Prudent Investor 1. The Duty to Diversify & Inception Assets: “A trustee shall diversify the investments of the trust unless the trustee
Rule reasonably determines that, because of special circumstances, the purposes of the trust are better served without
diversifying.” UPIA § 3. {McL}
 Exceptions to the Duty to Diversify: A trustee need not diversify if, “because of special circumstances, the
purposes of the trust are better served without diversifying.” UPIA § 3. These special circumstances include
(1) where tax or other costs of reorganizing the portfolio would outweigh the benefits of diversification, (2)
where there is a family-run business, residence, vacation home, or similar asset that the family does not want
sold; (3) where the trust is part of a larger program of wealth management that, overall, is diversified. See
UPIA § 2(c)(6). {McL}
 Inception Assets & Reasonable Time: Under UPIA § 4, a trustee must divest inception assets within a
“reasonable time” if necessary to bring the trust portfolio into compliance with the prudent investor rule.
“[T]he reasonableness of . . . delay in making a disposition depends on such factors as: the nature of the

72/87
property involved; the reason the trustee is required to sell it; whether appraisals are necessary; whether there
is a ready market for the property; and the relative degree of price efficiency in that market.” RS3d § 92 cmt.
b.
 Adequate Diversification: A common rule of thumb is that a concentration in a single security of more than
5 percent requires explanation. (Principal split between 20 stocks)
 Professional and Corporate Trustees: “A trustee who has special skills or expertise, or is named trustee in
reliance upon the trustee’s representation that the trustee has special skills or expertise, has a duty to use those
special skills or expertise.” A trustee may breach this duty by failing to develop a written investment policy
statement for each new trust account to follow internal protocols that require enhanced review of any
portfolio with a concentration in excess of 10 or 20 percent and documentation of the reasons for the
concentration.
2. The Terms of the Trust
 Permissive Authorization to Retain Undiversified Portfolio: Trust provisions that mandate inception asset
retention should be distinguished from those that authorize inception asset retention. The prevailing view is
that a permissive authorization to retain an undiversified portfolio does not excuse from liability if not
diversifying was imprudent.
 Mandatory Retention of Undiversified Portfolio: Courts are reluctant to find an effective negation of the
duty to diversify. If a settlor clearly directs the trustee to retain certain assets, however, the trustee must do so,
“and the trustee may be subject to liability for disposing of them if they subsequently increase in value. A
trustee’s duty to conform to the terms of the trust is qualified by a duty “to petition the court for appropriate
modification of or deviation from the terms of the trust” if conforming will “cause substantial harm to the
trust or its beneficiaries.”
o Protect yourself as trustee by getting permission to retain undiversified portfolio or permission to
diversify {McL}
 Scholarly Debate & Legislative Change
o Langbein (benefit-the-beneficiary principle): A trust cannot have the dominant purpose of
retaining certain assets. For the trust to be valid, this purpose must be subordinate to the purpose of
benefitting the beneficiaries.
o Cooper: The benefit-the-beneficiary principle would frustrate the interpersonal aspects of wealth
transmission and carefully-designed provisions intended to further a living settlor’s unique estate
planning goals.
3. The Custodial & 3. The Custodial & Administrative Functions
Administrative  Custodial Function: Taking custody of the trust property and properly safeguarding it.
Functions  Administrative Function: Includes record keeping, bringing and defending claims held in trust, accounting
and giving information to the beneficiaries, and making tax and other required filings.
a. Duty to Collect & a. Duty to Collect & Protect Trust Property: “A trustee shall take reasonable steps to take control of and protect the
Protect Trust trust property” (without unreasonable delay). UTC § 809. What is unreasonable delay and what steps are necessary
Property depend on the circumstances.
 Testamentary Trusts: The trustee should collect the assets from the executor as promptly as circumstances
permit, as well as examine the property tendered to verify it is what the trustee ought to receive. The trustee
must pursue the executor to redress any breach of duty that diminished the assets intended for the trust. The
same rules apply to successor trustee in regard to the trust property tendered by a prior trustee. UTC § 812.
b. Duty to Earmark b. Duty to Earmark Trust Property: A trustee has a duty to earmark trust property, that is, to designate it as trust
Trust Property property. “[A] trustee shall cause the trust property to be designated so that the interest of the trust, to the extent
feasible, appears in records maintained by a party other than a trustee or beneficiary.” UTC § 810(c). “[D]eposits of
trust money in a bank should be made in a separate account in the name of the trustee as trustee,” and “title to land
acquired by a trustee should be taken and recorded in the name of the trustee as trustee.” RS3d Trusts.
 Exception—Assets Not Subject to Registration: Assets not subject to registration, such as bearer bonds or
tangible personal property, fall within an exception to the earmarking requirement. In such cases, the trustee
must keep records indicating that the property belongs to the trust and the property must still be kept separate
from the trustee’s own property.
 Exception—Pooled Investments: Although normally a trustee should not mingle the property of two or
more trusts, a trustee may make a joint investment from separate trusts (but not the trustee’s own property) if
the trustee maintains clear records of the respective interests. UTC § 810(d).
o Rationale: Achieve economies of scale, make available additional investment opportunities, allow
for greater diversification.
 Common Law—Strict Liability: Under older authority, if a trustee commits a breach of trust by failing to
earmark a trust investment, the trustee is strictly liable for any loss, even if the loss was not caused by the
failure to earmark.
 Modern Trend—Causation: The trustee is liable only if the loss results from the failure to earmark and is
not liable if the loss results from general economic conditions.
c. Duty Not to c. Duty Not to Mingle Trust funs with the Trustee’s Own: A trustee breaches the earmarking rules by commingling
Mingle Trust Funds the trust assets with the trustee’s own assets—a trustee must “keep trust property separate from the trustee’s own
with the Trustee’s property.” UTC § 810(b).
Own  Standard of Liability: The older view is that a trustee is strictly liable, even if the loss would have occurred
had there been no commingling. More recent authority holds the trustee liable only to the extent the
73/87
commingling caused the loss.
d. Duty to Keep d. Duty to Keep Adequate Records of Administration: A trustee must maintain adequate records of the trust property
Adequate Records and the administration of the trust, as under UTC § 810(a), including documentation of important decisions and actions
of Administration and the trustee’s reasoning for those decisions and actions.
 Rationale: Promote prudent and loyal administration by imposing discipline on the trustee, enable the
beneficiaries to undertake a meaningful review of the trustee’s administration of the trust, protect the trustee
against hindsight review
 Professional Trustee: A professional trustee is held to an even higher standard in this regard because of her
assumed expertise, experience, and training.
e. Duty to Bring & e. Duty to Bring & Defend Claims: A trustee is under a duty to “take reasonable steps to enforce claims of the trust
Defend Claims and to defend claims against the trust.” UTC § 811. A prudent trustee will consider the “likelihood of recovery and the
cost of suit and enforcement,” litigating cost-effective claims and compromising or dropping claims that are not cost
effective.
 Suit by Beneficiary: Where a trustee wrongfully fails to bring a claim, a trust beneficiary may bring the
claim against the third party (and join the trustee for failure to perform her duty).
o Rationale: Judicial economy—settle the controversy in a single suit.
D. The Duty of Impartiality
D. Duty of Impartiality: A trustee is under a duty to administer the trust solely in the interest of the beneficiaries. If a trust has two or more
beneficiaries, the trustee shall act impartially in investing, managing, and distributing the trust property, giving due regard to the beneficiaries’
respective interests. UTC § 803. {McL}
 Concurrent beneficiaries, lifetime beneficiaries vs. remainder beneficiaries
1. Due Regard & the 1. Due Regard & Terms of the Trust: “If a trust has two or more beneficiaries, the trustee shall act impartially in
Terms of the Trust investing, managing, and distributing the trust property, giving due regard to the beneficiaries’ respective interests.”
UTC § 803. {McL}
 Although a trustee owes a duty to any remainder beneficiaries as well as to a life income beneficiary, the
trustee must carry out those duties in light of any preference expressed in the trust instrument. An
appellate court construes trust instruments in accordance with the trustor’s intent by looking at the entire
instrument and, if possible, giving effect to all of the instrument’s provisions. Howard. *ADD
 Beneficiary’s Resources: The question of whether a trustee may or should consider a beneficiary’s other
resources in administering a trust is a matter of construction.
o Example: Language in the trust providing that upon the settlor’s death, the trustee has the power to
invade the trust principle for the support in reasonable comfort and maintenance of the spouse, has
been interpreted as reflecting the intention that the surviving spouse’s living expenses are to be
borne exclusively by the trust income and principle.
o Example: Where a trustee’s discretion to pay is qualified by such words as “when in need” or “if
necessary,” the courts find that the surviving spouse’s living expenses are to be borne by the trust
income and principle only under those circumstances.
Howard v. Howard (OR): Marcene (income beneficiary) was married to Leo Howard (settlor). They had trust agreements prepared to replace
their earlier wills. Leo’s trust provided that the income was to go to Marcene for life: “[a]fter my death, in the event my spouse survives me, my
spouse’s support, comfort, companionship, enjoyment and desires shall be preferred over the rights of the remaindermen.” After Leo’s death, a
dispute arose between Marcene the remainder beneficiaries. The appellate court determined that the trial court was correct in holding that Leo
did not intend for the trustee to consider Marcene’s other financial resources when administering the assets (investing) of the estate because the
provision relating to the payment of net trust income did not reference such a requirement, even though such was explicitly included in other
provisions (negative inference) and because Leo expressed a desire to his
attorney to abrogate the duty of impartiality. Leo’s decision not to provide for
his stepchildren circumscribed neither his direction of trust income to Marcene
nor his preference for her interest over those of the remainder beneficiaries.

2. The Principal & 2. The Principal & Income Problem: Typically, the life beneficiaries of a trust are entitled to the income, and the
Income Problem remainder beneficiaries are entitled to the principal. The bifurcated interests mean that decisions concerning what
constitutes income and what constitutes principal are critical to the interests of different categories of beneficiaries.
a. 1962 Uniform Principal & Income Act: The 1962 Principal & Income Act set forth the traditional approach to
allocating income and principal.
i. Income: Rents, cash dividends on common stock, interest on bonds, net profits from a business.
ii. Principal: increases in asset value such as stock or land appreciation.
b. Revised Uniform Principal & Income Act:
 Rationale: Addresses tensions between (i) investing trust portfolio to maximize total return and (ii) duty to
produce appropriate income for beneficiaries. {McL}
74/87
i. Equitable Power of Adjustment: The Act continues the traditional rule of allocation to income or principal on the
basis of form. However, to free the trustee’s hand in crafting a portfolio for total return (modern portfolio theory)
without the constraint of needing to produce income for a current beneficiary, § 104 gives the trustee a power to adjust
between income and principal: “A trustee may adjust between principal and income to the extent the trustee considers
necessary if the trustee invests and manages trust assets as a prudent investor, the terms of the trust describe the amount
that may or must be distributed to a beneficiary by referring to the trust’s income, and the trustee determines, after
applying the rules [of formal allocation], that the trustee is unable to comply with [the duty of impartiality].”
ii. Unitrust: The settlor may set a percentage of the value of the trust corpus that must be paid to the income
beneficiary each year. The corpus is then revalued each year.
 Trustee-Beneficiary: The statutory adjustment power is usually denied to a trustee who is also a beneficiary.
{McL} *ADD
o Potential for conflict of interest (structural conflict)
a. Unitrust Election: Many states have statutes authorizing a trustee to convert a traditional trust to a unitrust. In such
states, the percentage is usually set by statute, typically between 3 and 5 percent, and it is applied to a rolling average
value over the prior three years.
 Trustee-Beneficiary: A trustee’s status as a remainder beneficiary does not in itself invalidate a unitrust
election made by that trustee. Nevertheless, a unitrust election from which a trustee benefits will be
scrutinized by the courts with special care. In determining whether application of the optional unitrust
provision is appropriate, it remains for a trial court to review the process and assure the fairness of the
trustees’ election by applying relevant factors including (i) the nature, purpose, and expected duration of the
trust; (ii) the intent of the creator of the trust; (iii) the identity and circumstances of the beneficiaries; (iv) the
needs for liquidity, regularity of payment, and preservation and appreciation of capital; (v) the assets held in
the trust; the extent to which they consist of financial assets, interests in closely held enterprises, tangible and
intangible personal property, or real property; the extent to which an asset is used by a beneficiary; and
whether an asset was purchased by the trustee or received from the creator of the trust. In re Heller. *ADD
In re Heller (NY): Jacob created a testamentary trust for the benefit of his wife, Bertha, and his children from a previous marriage. Bertha was
to receive the income from the trustee each year, and after her death, the principal was to be distributed to Jacob’s children. Jacob’s two
daughters were to receive a 30 percent share each, and his two sons, who were also trustees, 20 percent each. Following Jacob’s death, the
income to Bertha averaged $190,000 a year for a number of years. In 2001, New York enacted legislation that created an optional unitrust
formula and based on the net fair market value of the trust assets. In 2003, the sons elected to use this option and apply it retroactively, reducing
Bertha’s annual income to $70,000. The court ruled that when an interested trustee elects a unitrust, it is not per se inconsistent with his or her
fiduciary duties, and therefore, though the sons were beneficiaries and trustees, they could elect unitrust treatment because they had had
fiduciary obligations not only to Bertha, but also to other
remainder beneficiaries. The court ruled, however, that the
trial court should scrutinize the unitrust election with
special care in such cases.

E. The Duty to Inform & Account


E. The Duty to Inform & Account: A trustee is under an ongoing duty, codified by UTC § 813, to keep the beneficiaries informed about the
administration of the trust, in particular information needed by the beneficiaries to protect their interests in the trust.
 Common Law: A beneficiary is entitled to compel the trustee to render accountings of the trust, and nonroutine caselaw **look in {CT}
a. UTC § 813. Duty to Inform & Report. {McL}
 (a) A trustee shall keep the qualified beneficiaries of the trust reasonably informed about the administration of the trust and of the
material facts necessary for them to protect their interests. Unless unreasonable under the circumstances, a trustee shall promptly respond
to a beneficiary’s request for information related to the administration of the trust.
 (b) A trustee:
o (1) upon request of a beneficiary, shall promptly furnish to the beneficiary a copy of the trust instrument;
o (2) within 60 days after accepting a trusteeship, shall notify the qualified beneficiaries of the acceptance and of the trustee’s
name, address, and telephone number;
o (3) within 60 days after the date the trustee acquires knowledge of the creation of an irrevocable trust, or the date the trustee
acquires knowledge that a formerly revocable trust has become irrevocable, whether by the death of the settlor or otherwise,
shall notify the qualified beneficiaries of the trust’s existence, of the identity of the settlor or settlors, of the right to request a
copy of the trust instrument, and of the right to a trustee’s report as provided in subsection (c); and
o (4) shall notify the qualified beneficiaries in advance of any change in the method or rate of the trustee’s compensation.

75/87
 (c) A trustee shall send to the distributees or permissible distributees of trust income or principal, and to other qualified or nonqualified
beneficiaries who request it, at least annually and at the termination of the trust, a report of the trust property, liabilities, receipts, and
disbursements, including the source and amount of the trustee’s compensation, a listing of the trust assets and, if feasible, their respective
market values. Upon a vacancy in a trusteeship, unless a cotrustee remains in office, a report must be sent to the qualified beneficiaries by
the former trustee. A personal representative, [conservator], or [guardian] may send the qualified beneficiaries a report on behalf of a
deceased or incapacitated trustee.
 (d) A beneficiary may waive the right to a trustee’s report or other information otherwise required to be furnished under this section. A
beneficiary, with respect to future reports and other information, may withdraw a waiver previously given. . . .
1. Affirmative 1. Affirmative Disclosure: Statutes in many states, and UTC § 813(c), impose on a trustee a duty of periodic
Disclosure accounting to the beneficiaries regarding the administration of the trust. In the absence of such statute, a beneficiary
may compel a trustee to render an accounting. In addition to these accounting obligations, the common law has come to
recognize a duty to make affirmative disclosure to the beneficiaries of significant developments or intended
transactions.
 Duty to Notify Before Acting: Ordinarily, periodic statements are sufficient to satisfy a trustee’s duty to
beneficiaries of transactions affecting the trust property. The trustee must inform beneficiaries, however, of all
material facts in connection with a nonroutine transaction which significantly affects the trust estate and the
interests of the beneficiaries prior to the transaction taking place. Allard.
o Nonroutine Transaction: a trustee’s duty to provide information to beneficiaries may apply when
there are (inter alia): significant changes in trustee circumstances, including changes in the
identities, number, or roles of trustees or in methods of determining trustee compensation; decisions
regarding delegation of important fiduciary responsibilities or significant changes in arrangements
for delegation; important adjustments being considered in investment or other management
strategies; significant actions under consideration involving hard-to-value assets or special
sensitivity to beneficiaries (such as liquidating or selling shares of a closely held business or a sale
or long-term lease of a major real-estate holding); plans being made for distribution on termination
or partial termination (or perhaps subdivision) of the trust. RS3d Trusts, § 82.
 Trustee should err on the side of report.
 Duty to Obtain Maximum Price: Courts generally require that a trustee when selling trust assets try to
obtain the maximum price for the asset by either obtaining an independent appraisal of the property or by
“testing the market” to determine what a willing buyer would pay. Allard.
Allard v. Pacific National Bank (WA): J.T. and Georgina Stone
established testamentary trusts for the benefit of their children and the
issue of their children, with Pacific Bank as trustee. The sole asset of the
trust was a piece of real estate in downtown Seattle subject to a 99-year
lease entered into by the settlors with Seafirst Bank. The Stones gave the
trusts gave Pacific Bank “full power . . . to manage, improve, sell, lease,
mortgage, pledge, encumber, and exchange the whole part of the assets of
the trust estate.” Seafirst assigned its leasehold interest to Credit Union,
and Credit Union approached Pacific Bank about purchasing the real
estate for $139,000. Pacific Bank countered with $200,000. Credit Union
agreed, and Pacific Bank sold the real property to Credit Union. The trust
beneficiaries filed an action against Pacific Bank, alleging breach of
fiduciary duty by failure to inform them of the possible sale. The court
held that because the trust instrument gave the trustee complete power over the trust property, including the power to sell, Pacific Bank did not
need to obtain consent of the trust beneficiaries before selling the property. However, the court found that Pacific Bank breached its fiduciary
duty in its management of the trusts by failing to inform the beneficiaries of the impending sale (nonroutine transaction) so that they could
protect their interests (by bidding on it). Pacific Bank further breached its fiduciary duties by failing to ensure that the sale was for the highest
possible price by obtaining an independent appraisal or by “testing the market” before selling.
2. Responding to a 2. Responding to a Request for Information: A trustee is under a duty, as under UTC § 813, to respond promptly “to
Request for a beneficiary’s request for information related to the administration of the trust,” including a request to furnish a
Information complete copy of the trust instrument. {McL}
a. Settlor Authorizes Withholding Info: Where the settlor expressly provides in the trust that the terms of the trust or
information about the trust are to be withheld from the beneficiary, the law is not clear.
 Virginia: Where a trust is created for several beneficiaries, each of them is entitled to information as to the
trust. Even though the terms of the trust may regulate the amount of information which the trustee must give
and the frequency with which it must be given, the beneficiary is always entitled to such information as is
reasonably necessary to enable him to enforce his rights under the trust or to prevent or redress a breach of
trust. Fletcher. {McL}
o Solution: Use separate trust instruments
o Revocable Trust as Secret Will: One policy question is whether a settlor should be able to make
what is in effect a secret will by using a revocable trust. Implicitly, Fletcher holds the answer is No.
 UTC Approach: The UTC provides that the default rule is that upon request a trustee must “promptly furnish
to the beneficiary a copy of the trust instrument,” but this provision is not included in the schedule of
mandatory rules in § 105, which means a settlor is free to provide otherwise in the terms of the trust. {McL}
o Disagreement—check each state
 RS3d: Such a term is enforceable so long as it does not impair the ability of a beneficiary to protect his
76/87
interest in the trust: “By the terms of the trust . . . the settlor can limit the trustee’s duty to disclose trust
provisions or information on a reasonable basis, in order, for example, to lessen the risk of unnecessary or
unwarranted loss of privacy, or the risk of adverse effects upon youthful or troubled beneficiaries about
whose motivation or responsibility the settlor has concerns. Even limitations of these types, however, cannot
properly prevent beneficiaries . . . from requesting and receiving information to the extent currently necessary
for the protection of their interests. . . .” RS3d Trusts, § 82 cmt. e.
 California—Right to Receive: A statute provides that when a revocable trust becomes irrevocable because
of the death of the settlor, the trustee shall provide a complete copy of the trust to any beneficiary or heir of
the settlor who requests it.
 Secret Trust
o Under traditional law, a settlor may limit a beneficiary’s right to information but cannot override a
beneficiary’s right to information reasonably necessary for the protection of the beneficiary’s
interest in the trust.
o Under the UTC, a settlor can prevent a beneficiary from learning of a trust’s existence [indefinitely]
if there is a trust protector or other surrogate to whom information must be given and who has
standing to bring suit against the trustee for breach of trust. UTC § 105(b)(8)–(9).
Fletcher v. Fletcher (VA): Elinor Fletcher (settlor) created a revocable inter vivos
trust that provided that upon her death, three separate trusts would be created,
including one for her adult son James. Following Elinor’s death, one of Elinor’s
other sons and a corporate trustee were appointed trustees. James sued the trustees
after they refused to provide him with a copy of the trust instruments in their
possession. The trustees replied that Elinor had expressly indicated (orally) that the
terms and dealings were to be kept confidential, even from beneficiaries. The
trustees also argued that that the trust instrument established three separate trusts
and that the segregation of a trust into separate trusts for different beneficiaries not
only segregated the assets, but also segregated the trustee’s duties to the different
beneficiaries. The trustees stated that they had provided the beneficiary with all
information relevant to him. The court held the trial court had properly ordered the
trustees to provide to James all trust instruments in their possession because all the
trusts were part of one cohesive trust with one corpus and the information not
disclosed could have a material bearing on the administration of the trust agreement as far as James was concerned.
3. Accountings & 3. Accountings & Repose: “A trustee is not liable to a beneficiary for a breach of trust if (1) the facts of the breach are
Repose fairly disclosed in an accounting filed with the court, (2) notice of the accounting is properly served on the beneficiary,
and (3) the beneficiary does not timely object. A beneficiary who does not object is barred by res judicata from later
bringing a claim against the trustee that could have been brought in the accounting proceeding. {McL}
 Rationale: Offer the trustee a carrot (repose) to encourage the trustee to make regular and substantial
accountings to the beneficiary—to be transparent and open. {McL}
a. Misrepresentations by Fiduciaries: Entries in the accounts honestly made, after reasonable efforts to determine the
truth or falsity of the representations therein have failed through no fault of the trustee, will not be deemed fraudulent or
provide grounds for reopening otherwise properly allowed accounts. Nat’l Acad. of Scis.
 Exception—Constructive or Technical Fraud: If a person make a representation of a fact, as of his own
knowledge, in relation to a subject matter susceptible of knowledge, and such representation is not true; if the
party to whom it is made relies and acts upon it, as true, and sustains damage by it, it is fraud and deceit, for
which the party making it is responsible. Nat’l Acad. of Scis. {McL}
o Fraud contemplates the standard of constructive fraud at least to the extent that the fiduciary has
made no reasonable efforts to ascertain the true state of the facts it has misrepresented in the
accounts. This rule is not a strict liability standard, nor does it make a trustee an insurer against the
active fraud of all parties dealing with the trust. Nat’l Acad. of Scis.
o Discretionary Distribution: Does not apply to discretionary distribution that the remainder
beneficiary does not object to. {McL}
o Facial Errors: Does not apply to errors in distribution that are evidnece on the face of the
accounting and to which the remainder beneficiary does not object. {McL}
b. Misdelivery: Under traditional law, a trustee is liable for a mistaken delivery of trust property to an ineligible person
even if the “trustee . . . reasonably believe[d] that the person” was a property recipient. The modern trend is to protect a
trustee who makes such a mistake as long as the trustee has made “diligent, good-faith efforts or . . . reasonable
reliance” on the terms of the trust.
 UTC Approach: “If the happening of an event, including marriage, divorce, performance of educational
requirements, or death, affects the administration or distribution of a trust, a trustee who has exercised
reasonable care to ascertain the happening of the event is not liable for a loss resulting from the trustee’s lack
of knowledge.” UTC § 1007.
c. Informal Accounting: To hold down the costs of administration, a settlor sometimes waives the rule that a
beneficiary can compel a trustee to render an accounting to a court, providing instead that an informal accounting
approved by some or all of the beneficiaries shall have the same effect as a judicial accounting. Whether such waivers
are enforceable is uncertain and varies largely by jurisdiction and case-specific circumstances.
 Release from Beneficiary: A release of liability given by the beneficiary to a trustee is enforceable only if
the beneficiary knew of her rights and of the material facts of any breach covered by the release. UTC § 1009.
77/87
Nat’l Academy of Scis. v. Cambridge Trust Co. (MA): In his will, Leonard Troland (settlor) created a trust, to be administered by Cambridge,
for the benefit of his wife, Florence, as long as she did not remarry. Furthermore, the will provided that if Florence remarried or died then the
remainder of the trust was to be paid to National Academy. After Leonard passed away, Cambridge started to make trust payments to Florence.
Unbeknownst to Cambridge, Florence remarried soon after Leonard’s death and continued to receive trust payments until death. It was not until
Florence’s death that Cambridge discovered that she had remarried. On appeal, the court noted that Cambridge made the disputed payments for
22 years and during that time made no effort to ascertain if Florence had remarried. The trustee argued that the payments had been disclosed in
its annual accounting that were filed with the probate court, provided to the remainder beneficiary, and approved by the probate court. The court
held that the representation in the accountings that Florence remained unmarried (“Florence R. Troland”) was a factual representation that
Cambridge had never undertaken to verify, not even asking her to certify it, and implicitly represented that Cambridge had undertaken
reasonable care in ascertaining whether Florence had remarried. Therefore, the court held the representations constituted a constructive or
technical fraud on both National Academy and the court. The court reopened the accountings and held Cambridge liable for the payments made
to Florence after she had remarried.

10. Trusts: Alienation & Modification


A. Alienation of the Beneficial Interest
2. Spendthrift Trusts 2. Spendthrift Trusts: A settlor can modify a beneficiary’s ability to transfer his or her interest by including what is
known as a spendthrift clause in the trust that expressly limits the beneficiary’s power to transfer her interest. A
spendthrift interest prohibits both (1) voluntary alienation and (2) involuntary alienation by the beneficiary. {McL}
 Voluntarily: A beneficiary of a spendthrift clause cannot voluntarily alienate her interest in the trust (i.e., sell
the interest). {McL}
 Involuntary Alienation: Nor can her creditors attach her interest—even if the beneficiary is entitled to
mandatory distributions from the trust.
o Exception: Children & Spouses
 Example: “The interests of beneficiaries in principal or income shall not be subject to the claims of any
creditor, or to legal process, and may not be voluntarily or involuntarily alienated or encumbered.”
 Presumptions: Under traditional law, a trust is not spendthrift unless the settlor includes a spendthrift clause
in the trust instrument. However, a growing number of jurisdictions (e.g., Delaware) no longer require an
express spendthrift provisions.
i. Validity: There is an ongoing debate over the validity and scope of spendthrift clauses.
 Arguments Against: Unfair to creditors—especially spouses and children or tort victims; Unfairly favors the
rich over the poor
 Arguments in Support: Freedom of disposition/testation of donor. A creditor can still reach the beneficiary’s
interest once the property is distributed to the beneficiary; the creditor just cannot reach the property while it
is in the trust.
ii. General Rule: Today the spendthrift trust is recognized throughout the United States.
 UTC § 502. Spendthrift Provision.
 (a) A spendthrift provision is valid only if it restrains both voluntary and involuntary transfer of a
beneficiary’s interest.
 (b) A term of a trust providing that the interest of a beneficiary is held subject to a “spendthrift trust,” or
words of similar import, is sufficient to restrain both voluntary and involuntary transfer of the beneficiary’s
interest.
 (c) A beneficiary may not transfer an interest in a trust in violation of a valid spendthrift provision and, except
as otherwise provided in this [article], a creditor or assignee of the beneficiary may not reach the interest or a
distribution by the trustee before its receipt by the beneficiary.
iii. Exceptions: Although spendthrift clauses are generally valid, most jurisdictions have either statutorily or judicially
adopted doctrines that limit their application and effect.
a. UTC § 503. Exceptions to the Spendthrift Provision.
 (a) In this section, “child” includes any person for whom an order or judgment for child support has been
entered in this or another State.
 (b) A spendthrift provision is unenforceable against:
o (1) a beneficiary’s child, spouse, or former spouse who has a judgment or court order against the
beneficiary for support or maintenance;
o (2) a judgment creditor who has provided services for the protection of a beneficiary’s interest in
the trust; and
o (3) a claim of this State or the United States to the extent a statute of this State or federal law so
provides.
 (c) A claimant against which a spendthrift provision cannot be enforced may obtain from a court an order
78/87
attaching present or future distributions to or for the benefit of the beneficiary. The court may limit the award
to such relief as is appropriate under the circumstances.
b. Tort Victims: All states (except Georgia) have rejected an exception for tort victims. See Scheffel.
 UTC Approach: UTC § 503 does not recognize an exception for tort creditors.
 RS3d Approach: RS3d Trusts does not recognize an exception for tort creditors in the blackletter, but a
comment observes that “evolving policy” might “justify recognition of other exceptions.”
c. Children and Spouses: Judgments for child or spousal support, or both, can be enforced against the debtor’s interest
in a spendthrift trust in a majority of states and under UTC § 503(b)(1).
 Rationale: Preference for making good on support orders obtained by a child or spouse against the
beneficiary.
d. Necessary Support & Other Exceptions: A claimant who provided services necessary to protect a beneficiary’s
interest in a trust may recover against the beneficiary’s interest. UTC § 503(b)(2). Another longstanding exception is for
a claimant who provided a beneficiary with necessities such as a physician or grocer.
 Rationale: Ensure a beneficiary can obtain legal representation and necessary goods and assistance
e. Station-in-Life Rule: In NY and a few other states, a beneficiary’s creditors can reach spendthrift trust income in
excess of the amount needed for the support of the beneficiary. In determining what is necessary for the support of the
beneficiary, courts developed the station-in-life rule. Creditors can reach only the amount in excess of what is needed to
maintain the beneficiary in his station in life.
f. Federal Bankruptcy Law
g. Pension Trusts
Scheffel v. Krueger (NH): Krueger’s grandmother set up a trust for his
benefit. The trust had a spendthrift clause. Thereafter, Krueger sexually
assaulted a minor. In addition to being criminally convicted, a tort
judgment in the amount of more than $500,000 was entered against
him. The mother of the victim sought to attach to Krueger’s interest in
the trust to satisfy the judgment. The court ruled that the spendthrift
provision barred the mother’s claim against the trust because (1) N.H.
Rev. Stat. Ann. § 564:23(I) provided two exceptions to the
enforceability of spendthrift provisions, neither of which was
applicable; (2) nothing in the language of the statute suggested that the
legislature intended that a tort creditor should be exempted from a spendthrift provision; (3) and N.H. did not recognize a public policy tort
creditor exception to the statute.
11. Trusts: Charitable Purposes, Cy Pres, & Supervision
11. Trusts: Charitable Purposes, Cy Pres & Supervision: In general, the same rules that apply to the formation and administration of a private
trust also apply to a charitable trust. There are four significant exceptions that differentiate a charitable trust from a private trust:
 (1) a charitable trust must be for a recognized charitable purpose rather than for one or more ascertainable beneficiaries;
 (2) because a trust is exempt from the RAP and so may last “in perpetuity,” it is more freely modified under the cy pres doctrine;
 (3) because a charitable trust does not require an ascertainable beneficiary, traditional law relies on the state attorney general to enforce
the trust; and
 (4) charitable trusts are generally exempt from taxation. {McL}
A. Charitable Purposes
A. Charitable Purposes: Unlike a private trust, which must be for the benefit of one or more ascertainable beneficiaries, a charitable trust must be
for one or more charitable purposes. The question is whether the settlor intended a recognized charitable purpose and, if so, whether the trust can
reasonably be expected to further that purpose.
 Valid Charitable Purposes: A charitable trust is created only if the settlor properly manifests an intention to create a charitable trust.
Charitable purposes include:
o (a) the relief of poverty;
o (b) the advancement of education;
o (c) the advancement of religion;
o (d) the promotion of health;
o (e) governmental or municipal purposes; and
 public parks, museums, libraries
o (f) other purposes the accomplishment of which is beneficial to the community. Taylor.
 This category evolves over time with social norms
 Benevolent Trusts: Benevolent or philanthropic trusts are mere exhibitions of liberality and generosity, without regard to their effect
upon the donees. They are trusts from which the income is to be paid at stated intervals to each member of a designated segment of the
public, without regard to whether or not the recipients are poor or in need, is not for the relief of poverty. Nor are they paid for a social
benefit to the community. They may not be upheld as a charitable trust. Taylor (trusts to providing gifts to children, regardless of their
need, or to make Christmas gifts to members of a certain class, without consideration of need or effect, are not charitable) Taylor.
 Examples—Charitable: Trust to benefit such of employees who are sick or needy (promote health/relieve poverty); Trust to pay salary
of single law professor (furthers educational purpose); Trust to provide scholarships or prizes for educational achievement (furthers
educational purpose); Trust for the improvement of the structure and methods of government, in a manner advocated by a particular
political party; Trust to advance the principles of socialism including supporting those candidates that have socialist views; Trust for the
purpose of bringing about a change in the law (i.e., equal rights amendment); Trust to promote rifle shooting (train members for army);
Trust to promote temperance in use of intoxicating liquors; Trust to promote vegetarianism {McL}
 Examples—Not Charitable: Trust for general benefit of 5,000 employees; Trust for the benefit of lawyers generally; Trust to educate a
79/87
particular purpose or the descendants of the settlor; Trust to promote a political party; Trust for students of a particular school; Trust to
fund marching band from town hall to settlor’s grave and back annually on anniversary of T’s death {McL}; Trust to train poodles to
dance {McL}
 Examples—Maybe Charitable: Trust to collect & public T’s writings/artwork (charitable if court determines it to be of literary value);
Erect monument to T (charitable if someone like George Washington)
 The RAP: A charitable trust is exempt from the rule against perpetuities. Taylor.
 Tax Deduction: A purpose that is charitable under state law may not qualify as charitable under federal tax law.
 Mortmain Statutes: Most states once had statutes permitting spouses and children to set aside deathbed wills making gifts to charity.
They also restricted the share of an estate that could be left to charity even if the will was executed years before death. These statutes
have been repealed or declared unconstitutional as a denial of equal protection of the law or on substantive due process grounds. (Try
undue influence, duress, or fraud.)
 Failed Charitable Trusts—Trusts for Noncharitable Purposes: Most states allow a trust for a noncharitable purpose that is not
capricious. Many courts recognize an honorary trust, in which the trustee is not under a legal obligation to carry out the settlor’s stated
purpose, but if the transferee declines or neglects to do so, she holds the property upon a resulting trust and the property reverts to the
settlor or the settlor’s successors.
Shenandoah Valley Nat’l Bank v. Taylor (VA): Suit was brought to determine the validity of the provisions of Henry’s will which undertook
to create a charitable trust by providing that yearly on the last school day before Easter and Christmas the trustee should divide the net income
earned on the trust estate into as many equal parts as there were children in the first three grades of a given public school and that he should pay
one of such equal parts to each child in such grade, “to be used by such child in the furtherance of his or her obtainment of an education.” The
Bank (trustee) contended that the will created a charitable trust for educational purposes and was, therefore, not invalidated by the rule against
perpetuities. The court held that contention was without merit because the language used in Henry’s will and the occasion specified for payment
of the funds to the children definitely indicated that there was no intent or belief in Henry that the funds would be put to any use other than such
as youthful impulse and desire might dictate, and by the terms of the instrument the Bank was given no control or discretion over the funds
received by the child and nothing toward the advancement of the education could have been attained by the ultimate performance by Bank of its
full duty. Neither was the trust a charitable trust because it was to be paid to each member of a designated segment of the public, without regard
to whether or not the recipients were poor or in need, and so was not for the relief of poverty; nor was it a social benefit to the community.

B. Cy Pres & Deviation


1. Cy Pres 1. Cy Pres (as nearly as possible): Under the cy pres doctrine, if a charitable trust’s specific purpose becomes illegal,
impossible, or impracticable, the court may direct the application of the trust property to another charitable purpose that
approximates the settlor’s general charitable intent.
 Analysis
 Has it become illegal, impossible, impracticable, or wasteful to carry out the donor’s charitable purpose?
o If yes, did the donor have “general charitable intent”? (In some jurisdictions, there is a presumption
of General Charitable Intent, or the requirement has been abolished.)
 If yes, cy pres applies. Assets will be applied to charitable purposes “as near as possible”
to purpose specified by donor.
 If no, (“specific” intent), gift/trust “fails,” assets pass by resulting trust to donor’s residual
beneficiaries or intestate heirs.

80/87
a. Illegal, a. Illegal, Impossible, or Impractical: A court may apply cy pres if a designated charitable purpose becomes illegal,
Impossible, or impossible or impractical. See In re Neher’s Will
Impractical  Rationale: Because cy pres ultimately seeks to further the settlor’s general intent, it has been justified as
intent-implementing.
 Specific vs. General Charitable Intent: Under traditional law, a precondition to applying cy pres is a finding
that the donor had a general rather than a specific charitable intent.
o UTC Approach: UTC § 413(a) modifies the doctrine of cy pres by establishing a presumption that
the donor had a general charitable intent.
 Approximating the Donor’s Intent: Some courts have held that the new purpose to which the property is
applied must approximate as nearly as possible the donor’s original state purpose.
o RS3d Trusts: Increasingly, however, courts have recognized (as does the rule of this Section) that
the substitute or supplementary purpose need not be the nearest possible but one reasonably similar
or close to the settlor’s designated purpose, or "falling within the general charitable purpose" of the
settlor. This is especially so when the particular purpose becomes impossible or impracticable of
accomplishment long after the creation of the trust or when, among purposes reasonably close to
the original, one has a distinctly greater usefulness than the others that have been identified. This
more liberal application of cy pres is appropriate both because settlors’ probable preferences are
almost inevitably a matter of speculation in any event and because it is reasonable to suppose
that among relatively similar purposes charitably inclined settlors would tend to prefer those most
beneficial to their communities. RS3d Trusts § 67 cmt. d.
o UTC: UTC § 413(a)(3) provides that in applying cy pres the court may “modify or terminate the
trust by directing that the trust property be applied or distributed, in whole or in part, in a manner
consistent with the settlor’s charitable purposes.”
 Example: [I]f a testator devises property in trust to establish and maintain an institution of a particular type
but a similar institution already exists and is sufficiently effective that the testator’s plan would serve no
useful purpose, the intended purpose will not be enforced. . . . If property is given in trust for a particular
charitable purpose and the amount given is insufficient to accomplish the intended purpose in a socially
useful manner, the specified purpose fails and may be modified cy pres RS3d Trusts § 67 cmt. c.
 Beneficiary Causes Impracticability: The Supreme Court of Iowa does not believe that a per se rule against
the cy pres doctrine exists when the trustee or donee causes the impracticability, impossibility, or illegality.
But there are circumstances when a natural and unavoidable change in conditions or circumstances causes the
trustee or donee to act. In such cases, a proper disregard for the grant or devise may be maintained by the
donee or trustee that does not preclude the application of cy pres. The supreme court refuses to adopt a per se
rule that a trustee or donee may not invoke the doctrine of cy pres when he or she causes the impossibility,
impracticability, or illegality. Kolb.
In re Neher’s Will (NY): Red Hook village was bequeathed property that was to be
used as a hospital for the benefit of the people of the village. The village accepted the
bequest but later filed a petition for a decree construing and reforming the will provision
so that the village could use the property for the village’s administrative building as the
village did not have the resources to establish a hospital and there was a new hospital
near the village. On appeal, the court held that the gift was not a gift to a particular
institution, that there was no singular object of the bounty because it was to a whole
community (“to the incorporated Village of Redhook”), and that there was no hint for
any certain type of medical or surgical care. The court concluded that, in reading the will
provision as a whole, the true construction was that the intention was to give the
property for a general charitable purpose rather than a particular charitable purpose and
that, as grafted direction, could be ignored when compliance was impracticable under the doctrine of cy pres.
Kolb v. City of Storm Lake (IA): T devised a fund in trust to the city of Storm Lake to maintain a memorial flower garden in a designated
location in a city park. Thirty years later, the city removed the garden to make room for a vacation resort and petitioned the court to apply cy
pres to allow the city to recreate the flower garden elsewhere in the park. The supreme court concluded that the settlors’ general charitable
intentions were to memorialize a family member by maintaining a flower garden for the enjoyment of the public, and that that charitable
purpose was superior to the specific language of the trust regarding the location of the garden. In addition, because changed circumstances made
it impracticable, if not impossible, to continue to carry out the trust at its original location, the supreme court concluded that the trust could be
modified to fund the garden at its new location. The court found that T would have wanted the memorial to continue, even in another location.
b. Wasteful b. Wasteful: Under traditional law, a court may apply cy pres only if a designate charitable purpose becomes illegal,
impossible, or impracticable. Today, however, many states and UTC § 413(a) also permit application of cy pres if the
particular charitable purpose becomes wasteful. If a charitable trust holds property well in excess of what is
needed for its particular charitable purpose, the UTC and Restatement permit the application of cy pres to
direct the use of the surplus funds to a reasonably similar purpose.
 RS3d: Another type of case appropriate to the application of cy pres, when not precluded by the terms of the
trust, is a situation in which the amount of property held in the trust exceeds what is needed for the particular
charitable purpose to such an extent that the continued expenditure of all of the funds for that purpose,
although possible to do, would be wasteful. Faced with circumstances of the type required for cy pres
intervention in a surplus-funds case, a court might broaden the purposes of the trust, direct application of the
surplus funds to a like purpose in a different community, or otherwise direct the use of funds not reasonably
needed for the original purpose to a different but reasonably similar charitable purpose. RS3d Trusts § 67 cmt.

81/87
c.
The Buck Trust: Decided under traditional law, the court held that concepts of inefficiency and
ineffectiveness were not relevant to decide whether a settlor’s charitable purpose had become impractical (a
testator had devised stock worth $9 million in trust to support the relief of poverty and other charitable
purposes in Martin County—one of the richest counties in the country—and within a decade, the stock was
worth more than $300 million).
 Wasteful: Efficiency or Surplus Funds?
 Was the Buck Trust Wasteful?
2. Doctrine of 2. Deviation: Deviation is applicable to all trusts (both private and charitable). A court may direct or permit a trustee to
Administrative deviate from the administrative terms of a trust if owing to circumstances not known to the settlor and not anticipated
(Equitable) Deviation by him compliance would defeat or substantially impair the accomplishment of the purpose of the trust. RS2d. {McL}
(applies to Private &  Deviation vs. Cy Pres:
Charitable Trusts) o The doctrine of cy pres is applied by courts to modify the charitable purpose of a gift/trust. {McL}
 *must show donor’s designated charitable purpose has become
illegal/impossible/impractical/[wasteful] (high threshold)
o The traditional doctrine of administrative deviation is applied by courts to modify an administrative
term (not the purpose) of a trust to better carry out that purpose {McL}
 Example—Pulitzer Case: T bequeathed shares of stock of the Press Publishing Company in trust for his
three sons and provided further that the shares should not be sold under any circumstances (administrative
term). It was shown that great losses were being incurred in the publication of the newspaper with the result
that the newspaper would soon be of no value. Under these circumstances the court authorized the trustees to
sell the assets of the company to better carry out the purpose of the trust
 UTC § 412(a)-(b): (a) The court may modify the administrative or dispositive terms of a trust . . . if, because
of circumstances not anticipated by the settlor, modification or termination will further the purposes of the
trust. To the extent practicable, the modification must be made in accordance with the settlor’s probable
intention. (b) The court may modify the administrative terms of a trust if continuation of the trust on its
existing terms would be impracticable or wasteful or impair the trust’s administration.
 The Barnes Foundation: The Barnes Foundation is a world-class art collection, amassed in the first half of
the twentieth century by Dr. Albert C. Barnes. In establishing the Foundation, Barnes set forth detailed and
comprehensive restrictions on the use of, management of, and access to the trust that funded it. The
Foundation’s bylaws formalized both his aesthetic and social values by setting forth detailed restrictions on
the composition and placement of the collection, the use of the buildings and grounds, the admission policy,
and the Foundation’s management.
o Issue—extent to which we may vary from settlor’s intent
 The Stieglitz
C. Enforcement of Charitable Trusts
C. Enforcement of Charitable Trusts: The law of fiduciary administration applicable to a private trust applies also to a charitable trust.
1. Traditional Law 1. Traditional Law: Under traditional law, a settlor does not have standing to enforce a charitable trust unless she
retains an interest in it. Instead, the state attorney general, as paren patrie, has primary responsibility for enforcing
charitable trusts. In most states, the trustee of charitable trust must make regular reports to the attorney general, who is a
necessary party in any litigation involving the trust.
 Common Law Rule: A suit can be maintained for the enforcement of a charitable trust by the Attorney
General . . . , or by a co-trustee, or . . . by a person who has a special interest in the enforcement of the
charitable trust, but not by persons who have no special interest or by the settlor or his heirs, personal
representatives or next of kin. RS2d Trusts § 391. {McL}
o Rationale: Prevent lawsuits against charities
 Improper Inducement: Donors have standing to allege the fraudulent or negligent inducement of their
donations. Siebach (“claims alleging the improper inducement of a charitable donation are distinguishable
from claims seeking to enforce donative intent…improper-inducement claims do not fall within the common-
law [no] donor-standing rule”).

A. Given common-law “no-donor-standing” rule, how can donors ensure their intent is honored?
 If state has adopted UTC either settlor-standing rule, make gift expressly “in trust.”
o Problem: Donor dies
 Reserve standing for donor/descendants in gift agreement.
o Problems: Donor dies; Descendants don’t gain anything from enforcing gift.
 Require that charity agree not to object to donor/descendant standing in gift agreement.
o Problem: Court may raise standing issue sua sponte
 Pay gift in installments during life; direct Executor/Trustee to pay installments after death.
o Problem: Unless really big, there executor/trustee will not always be there
 Reversion to donor/descendants (not typically done because it can render a gift nondeductible)
 Gift over to another charity if initial charity fails to comply with donor restrictions
o Problem: Charities might collude.
2. Settlor Standing 2. Settlor Standing: Today more than half the states allow the settlor of a charitable trust to enforce the trust.
o Rationale: “The donor of a charitable gift is in a better position than the Attorney General to be

82/87
vigilant and to enforce his or her own intent.” “[T]he Attorney General’s interest in enforcing gift
terms is not necessarily congruent with that of the donor.” Smithers. “‘The [AG] may not be in a
position to become aware of wrongful conduct . . . sufficiently familiar with the situation to
appreciate its impact, and the various responsibilities of his office may also tend to make it
burdensome for him to institute legal actions except in situations of serious public detriment.’” LB
Research & Education Foundation.
 AG is too influenced by politics to protect charities.
o Criticism: Dead hand control; flood of litigation—consume charitable trust assets
 UTC: The settlor of a charitable trust, among others, may maintain a proceeding to enforce the trust. UTC §
405(c). {McL}
 Restatement: A suit for the enforcement of a charitable trust may be maintained only by the Attorney
General . . . or by a co-trustee or successor trustee, by a settlor, or by another person who has a special interest
in the enforcement of the trust. RS3d Trusts § 94. {McL}
Smithers v. St. Luke’s-Roosevelt Hospital Center (NY): Adele Smithers (widow) filed a claim to enforce the terms of her deceased husband’s
$ 10 million gift to the Hospital which was dedicated for the express purpose of setting up a rehabilitation center for alcoholics. Adele
contended that the trust precluded the hospital from selling the building where the rehab center was located. Before Adele sued, the state
attorney general discovered that the Hospital had misappropriated funds to fund other hospital projects. Adele sought an accounting of the funds
of the gift and sought an injunction preventing the Hospital from selling the building in which the rehabilitation center was located or
alternatively, to prevent disbursement of the funds from the sale. On appeal, the court found that Adele had standing to bring the suit because
N.Y. Est. Powers & Trusts Law § 8-1.1 did not designate the attorney general as the exclusive representative of donors of charitable
dispositions. The general rule barring beneficiaries from suing charities did not apply to Adele because she was bringing the action on behalf of
her husband’s estate—not on her own behalf or on behalf of beneficiaries.
4. Persons with a 4. Persons with a Special Interest in the Trust: Even under traditional law, “a person who has a special interest in the
Special Interest in the enforcement of the trust” may be able to enforce a charitable trust. To have special interest standing, a person must
Trust show that he is entitled to receive a benefit under the trust that is not available to the public at large or to an average
beneficiary. Special interest standing extends only so far as the person’s special interest (i.e., a senior citizen center has
special interest standing to enforce a pledge to contribute funds from a charitable trust, but not to enforce the terms of
the trust generally).
 Example—Standing:
o An elderly, indigent widow living in a charitable home for the aged has been held to have standing
to sue the board of trustees who, because of the costs of operating an obsolete facility, proposed to
relocate the residents elsewhere.
o A parishioner can sue to enforce a trust for the benefit of his church
o A minister can sue to enforce a trust to pay the salary of the clergy of his church
 Example—No Standing:
o A person who is merely eligible within the trustee’s discretion for a benefit from a charitable trust
o The alumni association of the Hershey School did not have standing to enforce the Hershey Trust
o A student does not have standing to sue college trustees
Siebach v. BYU (UT): The donors to BYU’s fund for academic research in philosophy, whose son was employed by BYU as a philosophy
professor, brought action against BYU for breach of fiduciary duty, accounting, breach of contract, and revocation of gift, among other claims,
after BYU audited the fund and concluded that donors’ contributions violated federal tax laws. BYU froze the accounts in the fund but
continued accepting donations. The district court properly dismissed the donors’ claims against BYU to enforce their donations to a research
account for lack of standing because they relinquished all pecuniary interest in the donated funds at the time they completed their gifts. The
donors’ argument—that they qualified for the special-interest exception because after a dispute over the use and management of the funds arose,
BYU offered to return the unspent portion—was not valid. However, the donors did have standing to allege they were induced by fraud or
negligent misrepresentation.
12. Trusts: Powers of Appointment
12. Trusts: Powers of Appointment: The settlor of a trust may give to someone other than the trustee a nonfiduciary power to distribute the trust
property. Such a power is known as a power of appointment.
A. Purposes, Terminology, and Types of Powers
A. Purposes: Powers of appointment provide (1) flexibility and are also commonly used for (2) tax planning and (3) asset protection.
a. Flexibility: The donor empowers the donee to deal flexibly with changing circumstances in the interim between their deaths, such as births,
deaths, marriages, divorces, the evolving ability of children and more remote descendants to manage property, changes in the economy and
investment returns, changes in the law, and other circumstances that the donor could not have foreseen but that the donee will observe firsthand.
b. Tax Planning: The property subject to the power will not be treated as belonging to the donee for tax purposes. No income, estate, or gift tax
will be levied on the donee or her estate.
c. Asset Protection: The property subject to the power will not be treated as belonging to the donee for claims of the donee’s creditors. No creditor
of the donee will be able to reach the appointive property.
1. Terminology &  Power of Appointment: “A power of appointment is a power that enables the donee of the power to
Relationships designate recipients of beneficial ownership in . . . the appointive property.” RS3d Prop. § 17.2
a. The Parties a. The Parties
 Donor: A person who creates the power of appointment
 Donee: A person who holds a power
 Objects of the Power/Permissible Appointees: The persons in whose favor a power may be exercised
 Appointee: When a power is exercised in favor of an object, that person becomes an appointee.

83/87
 Takers in Default of Appointment: The persons who will receive property if a donee fails to exercise a
power of appointment.
 Appointive Property: The property subject to a power of appointment
b. Example: T devises property to X in trust to pay the income to A for life, or until such time as A appoints, and to
distribute the principal to such person or persons as A shall appoint by deed or by will. If A does not exercise the power
of appointment, at A’s death X is to distribute the principal to B. T is the donor. A is the donee. B is the taker in default
of appointment. The class of objects is limitless.
b. Creation b. Creation: To create a power of appointment, the donor must manifest the intent do so, either expressly or by
implication. It is not necessary that the words “power of appointment” or “appoint” be used. “Any words or phrases are
sufficient to create a power of appointment if they establish that the transferor so intended.” RS3d. § 18.1.
 Discretion: A power of appointment confers discretion on the donee, who may choose to exercise the power
or not, and is to be distinguished from a nondiscretionary, direct disposition by the donor.
 Precatory Words: Words that express a mere wish or desire are precatory and do not create a power of
appointment (i.e., “with the request but not the legal obligation that she give some of the property to my other
relatives)
c. General & c. General & Nongeneral Powers: All powers of appointment can be divided into general powers and nongeneral
Nongeneral Powers powers. The validity of an exercise of a power, the consequences of a failure to exercise a power, the rights of the
creditors of the donor and the donee to reach appointive property, and the tax consequences of holding or exercising a
power often turn on whether the power is general or nongeneral.
 General Power: A power to appoint assets that is exercisable in favor of (i) the holder of the power [donee],
(ii) his estate, (iii) his creditors, or (iv) the creditors of his estate.” Internal Revenue Code § 2041(b).
o Example: T devises property to X in trust to distribute the income and principal to such of A’s
creditors as A shall appoint by deed.
o Settlor of Revocable Trust: The settlor of a revocable trust has a general power of appointment—a
power to revoke the trust and take back the trust property is a power to appoint oneself [the holder
of the power].
 Nongeneral Power/Special Power/Limited Power: A power that is not exercisable in favor of the donee, his
estate, his creditors, or the creditors of his estate.
o Example: T devises property to X in trust to pay the income and principal to any person whom B
appoints by will except that B may not appoint herself, her estate, her creditors, or creditors of her
estate.

84/87
85/87
86/87
B. Administration of Probate Estate

1. Proving a will

a) Witness must appear in court or sign affidavit


b) Nowadays the witnesses are usually paralegals or secretaries
c) With developed was a self-proving will/affidavit  document signed by witnesses before a notary

87/87

Potrebbero piacerti anche